Anda di halaman 1dari 340

16 Electrical Measuring

Instruments

Chapter Overview

1. Types of Electrical Instruments


Electrical measuring instruments may be classified according to their functions as (i) indicating
instruments (ii) integrating instruments and (iii) recording instruments.
(i) Indicating instruments. Those instruments which directly indicate the value of the
electrical quantity at the time when it is being measured are called indicating instruments e.g.
ammeters, voltmeters and wattmeters. In such instruments, a pointer moving over a graduated
scale directly gives the value of the electrical quantity being measured. For example, when an
ammeter is connected in the circuit, the pointer of the meter directly indicates the value of cur-
rent flowing in the circuit at that time.
(ii) Integrating instruments. Those instruments which measure the total quantity of electric-
ity (in ampere-hours) or electrical energy (in watt-hours) in a given time are called integrating
instruments e.g. ampere-hour meter and watt-hour meter. In such instruments, there are sets of
dials and pointers which register the total quantity of electricity or electrical energy supplied to
the load.
(iii) Recording instruments. Those instruments which give a continuous record of the vari-
ations of the electrical quantity to be measured are called recording instruments. A recording
instrument is merely an indicating instrument with a pen attached to its pointer. The pen rests
lightly on a chart wrapped over a drum moving with a slow uniform speed. The motion of the
drum is in a direction perpendicular to the direction of the pointer. The path traced out by the
pen indicates the manner in which the quantity, being measured, has varied during the time of
the record. Recording voltmeters are used in supply stations to record the voltage of the supply
mains during the day. Recording ammeters are employed in supply stations for registering the
current taken from the batteries.
2. Principles of Operation of Electrical Instruments
An electrical instrument essentially consists of a movable element and a scale to indicate or
register the electrical quantity being measured. The movable element is supported on jewelled
bearings and carries a pointer or sets of dials. The movement of the movable element is caused
by utilising one or more of the following effects of current or voltage.
Electrical Measuring Instruments 451
............................................................................................................................................................................................................................................................................................................................................................

1. Magnetic effect ..... Moving-iron instruments


2. Electrodynamic effect ..... (i) Permanent-magnet moving coil
(ii) Dynamometer type
3. Electromagnetic-induction ..... Induction type instruments
4. Thermal effect ..... Hot-wire instruments
5. Chemical effect ..... Electrolytic instruments
6. Electrostatic effect ..... Electrostatic instruments

S. No. Type Effect Suitable for Instrument


1. Moving-iron Magnetic effect d.c. and a.c. Ammeter, Voltmeter
2. Permanent-magnet Electrodynamic effect d.c. only Ammeter, Voltmeter
moving coil
3. Dynamometer type Electrodynamic effect d.c. and a.c. Ammeter, Voltmeter,
Wattmeter
4. Induction type Electro-magnetic a.c. only Ammeter, Voltmeter,
induction effect Wattmeter, Energy-
meter
5. Hot-wire Thermal effect d.c. and a.c. Ammeter, Voltmeter
6. Electrolytic meter Chemical effect d.c. only Ampere-hour meter
7. Electrostatic type Electrostatic effect d.c. and a.c. Voltmeter only
The principles of operation of electrical instruments are given in the above table for facility
of reference.
3. Essentials of Indicating Instruments
An indicating instrument essentially consists of moving system pivoted in jewel bearings. A
pointer is attached to the moving system which indicates on a graduated scale, the value of the
electrical quantity being measured. In order to ensure proper operation of indicating instruments,
the following three torques are required :
(i) Deflecting (or operating) torque
(ii) Controlling (or restoring) torque
(iii) Damping torque
(i) Deflecting torque. One important requirement in indicating instruments is the arrange-
ment for producing deflecting or operating torque (Td) when the instrument is connected in the
circuit to measure the given electrical quantity. This is achieved by utilising the various effects
of electric current or voltage mentioned in section 2. The deflecting torque causes the moving
system (and hence the pointer attached to it) to move from zero position to indicate on a gradu-
ated scale the value of electrical quantity being measured. The actual method of producing the
deflecting torque depends upon the type of instrument and shall be discussed while dealing with
particular instrument.
(ii) Controlling torque. If deflecting torque were acting alone, the pointer would continue
to move indefinitely and would swing over to the maximum deflected position irrespective of the
magnitude of current (or voltage or power) to be measured. This necessitates to provide some
form of controlling or opposing torque (TC). This controlling torque should oppose the deflecting
torque and should increase with the deflection of the moving system. The pointer will be brought
452 Objective Electrical Technology
...........................................................................................................................................................................................................................................................................................................................................................

to rest at a position where the two opposing torques are equal i.e. Td = TC. The controlling torque
performs two functions :
(a) It increases with the deflection of the moving system
so that the final position of the pointer on the scale
will be according to the magnitude of current (or Balance
voltage or power) to be measured. Weight
(b) It brings the pointer back to zero position when the
deflecting torque is removed. If it were not provided,
the pointer once deflected would not return to zero
Control
position on removing the deflecting torque. Spring
The controlling torque in indicating instruments may be
Spindle
provided by one of the following two methods :
(a) By one or more springs ... Spring control
(b) By weight of moving parts ... Gravity control
The most common method of providing controlling torque
is by the use of one or more springs as shown in Fig. 16.1.
One or two spiral hair springs made of some nonmagnetic Pivot
material are attached to the moving system of the instrument.
With the deflection of the pointer, the spring is twisted in
the opposite direction. This twist in the spring provides the Fig. 16.1
controlling torque. In the gravity control, a small adjustable
weight W is attached to the moving system [See Fig. 16.2 (i)] which provides the necessary controlling
torque. In the deflected position [See Fig. 16.2 (ii)], only the component W sin provides the
controlling torque.
TC = W sin l = W l sin
or TC sin (for fixed W and l)
60 80

40
10


0

20

l
 Balance
0 Weight

W
 
W W sin 
Control W cos 
Weight W
(i) (ii)
Fig. 16.2
(iii) Damping torque. If the moving system is acted upon by deflecting and controlling torques
alone, then pointer, due to inertia, will oscillate about its final deflected position for quite some
time before coming to rest. This is often undesirable because it makes difficult to obtain quick and
accurate readings. In order to avoid these oscillations of the pointer and to bring it quickly to its
final deflected position, a damping torque is provided in the indicating instruments. This damping
Electrical Measuring Instruments 453
............................................................................................................................................................................................................................................................................................................................................................

torque acts only when the pointer is in motion and always opposes the motion. The position of the
pointer when stationary is, therefore, not *affected by damping. The damping torque in indicating
instruments can be provided by (a) air friction (b) fluid friction and (c) eddy currents.
ation of Spindle
Rot

Spindle

Vanes

Vane

Damping Oil

Sector-shaped Box

Fig. 16.3. Air friction damping Fig. 16.4. Fluid friction damping

Spindle

Disc S
Damping
Magnet

Fig. 16.5. Eddy current damping


In air friction damping (See Fig. 16.3), one or two light aluminium vanes are attached to the
same spindle that carries the pointer. The vanes are permitted to swing in a sector-shaped closed
box that is just large enough to accommodate the vanes. As the pointer moves, the vanes swing in the
box, compressing the air in front of them. The pressure of compressed air in the vanes provides
the necessary damping torque. In fluid friction damping [See Fig. 16.4], discs or vanes attached to
the spindle of the moving system are kept immersed in a pot containing oil of high viscosity. As the
pointer moves, the friction between the oil and vanes opposes motion of the pointer and thus necessary
damping torque is provided. In eddy current damping [See Fig. 16.5], a thin **aluminium or copper
disc attached to the moving system is allowed to pass between the poles of a permanent magnet. As
the pointer moves, the disc cuts across the magnetic field and eddy currents are produced in the disc.
These eddy currents react with the field of the magnet to produce a force which opposes motion (Lenzs
law). In this way, eddy currents provide damping torque to reduce the oscillations of the pointer.

* The damping torque acts only when the pointer is in motion. When the pointer is in a particular deflected
position, though deflecting and controlling torques are acting on the moving system but the damping torque
is zero. It is because the pointer is steady and there is no movement of the moving system.
** The disc must be a conductor but non-magnetic.
454 Objective Electrical Technology
...........................................................................................................................................................................................................................................................................................................................................................

4. Ammeters and Voltmeters


The devices which measure current and potential difference in a circuit are called ammeters
and voltmeters respectively. An ammeter is connected in series with the circuit element whose
current we wish to measure as shown in Fig. 16.6. A voltmeter is connected in parallel with the
circuit element across which potential difference is to be measured as shown in Fig. 16.7.

E E R V

Fig. 16.6 Fig. 16.7


The basic principle of the ammeter and of the voltmeter is the same. Both are current operated
devices i.e., deflecting torque is produced when current flows through their operating coils. In
the ammeter, the deflecting torque is produced by the current we wish to measure, or a certain
fraction of that current. In the voltmeter, the deflecting torque is produced by a current which is
proportional to the potential difference we wish to measure. Thus, the same instrument can be
used as an ammeter or voltmeter with proper design.
The following types of instruments are used for making voltmeters and ammeters :
(i) Permanent-magnet moving coil type (ii) Dynamometer type
(iii) Moving-iron type (iv) Hot-wire type
(v) Electrostatic type (for voltmeters only) (vi) Induction type
The instrument at Sr. No. (i) can be used for d.c. work only whereas instrument at Sr. No.
(vi) is employed for a.c. work only. However, instruments from Sr. No. (ii) to (v) can be used
for both d.c. and a.c. measurements.
5. Permanent-Magnet Moving Coil (PMMC) Instruments
(Ammeters and Voltmeters)
These are suitable for d.c. work only. This type of instrument is based on the principle that
when a current carrying coil is placed in a magnetic field, torque acts on the coil. Fig. 16.8
shows the various parts of a permanent-magnet moving coil instrument. It consists of a light
rectangular coil of many turns of fine wire wound on an aluminium former inside which is an
iron core as shown in Fig. 16.8 (i). The coil is delicately pivoted upon jewel bearings and is
mounted between the poles of a permanent horse-shoe magnet. Attached to these poles are two
soft-iron pole pieces which concentrate the magnetic field. The current is led into and out of the
coil by means of two control hair-springs, one above and the other below the coil, as shown in
Fig. 16.8 (ii). These springs also provide the controlling torque. The damping torque is provided
by eddy currents induced in the aluminium former as the coil moves from one position to another.
Working. When the instrument is connected in the circuit to measure current or voltage,
the operating current flows through the coil. Since the coil is carrying current and is placed
in the magnetic field of the permanent magnet, a mechanical force acts on it. As a result, the
pointer attached to the moving system moves in a clockwise direction over the graduated scale to
indicate the value of current or voltage being measured. If the current in the coil is reversed, the
deflecting torque will also be reversed since the direction of the field of the permanent magnet
is the same. Consequently, the pointer will try to deflect below zero. Deflection in this direction
Electrical Measuring Instruments 455
............................................................................................................................................................................................................................................................................................................................................................

(i.e., reverse direction) is prevented by a spring stop. Since the deflecting torque reverses with
the reversal of current in the coil, such instruments can be used to measure direct currents and
voltages *only.

Fig. 16.8
The magnetic field in the air-gap is radial due to the presence of soft-iron core. Therefore,
when operating current flows through the coil, a constant deflecting torque Td acts on the coil
given by ;
Td = BINA newton metre
where B = Flux density in Wb/m2 ; I = Current through the coil
N = Number of turns in coil ; A = Area of coil
Since the values of B, N and A are fixed,
Td I
The instrument is spring controlled so that TC . The pointer will come to rest where
Td = TC.
I
Thus the deflection is directly proportional to operating current. Hence such instruments
have uniform scale.

6. Range Extension of PMMC Instruments


The resistance of PMMC instrument and the current needed to produce full-scale deflection
(f.s.d.) are very small. Typical values of these parameters are :
Full-scale deflection current, Ig = 1 mA
Resistance of the meter, G = 20
This means that this instrument can read currents upto 1mA and voltages upto 20 mV
(= 1mA 20 ). In order to measure large currents and voltages (i.e. to extend the range of the
instrument) suitable means are provided.
* The instrument can be used to measure alternating currents and voltages by using a rectifier. The given
alternating quantity is converted to d.c. by using a bridge type rectifier. This d.c. or average value is measured
by the permanent-magnet moving coil instrument. Since a.c. meters are calibrated to indicate effective
or r.m.s. values, we must multiply the average value registered on this meter by 1.11 (the form factor of
sinusoidal wave) to obtain the r.m.s. values. Some instruments are scaled to read r.m.s. values directly.
456 Objective Electrical Technology
...........................................................................................................................................................................................................................................................................................................................................................

(i) Range extension of PMMC ammeter. In order to measure large currents, a suitable
low resistance S (called shunt) is connected in parallel with the instrument. The shunted moving
coil instrument is called an ammeter. The value of shunt is chosen according to the maximum
current we wish to measure. Suppose we want to measure I amperes at full-scale using a moving coil
instrument having full-scale deflection current Ig and resistance G. This means that when circuit
current is I, we want current Ig through the meter as shown in Fig. 16.9. For this purpose we
connect a shunt S of suitable value so that I Ig current flows through it. Since the potential
difference across the shunt is the same as across the meter,
(I Ig) S = Ig G
Ig
S = G
I Ig
GS
Resistance of ammeter, Rm = G y S =
G+S
Clearly, the value of Rm will be less than S. Since the value of S is very small, the ammeter
resistance will be very low. Thus shunt has not only extended the current range but it has also
lowered the resistance of the ammeter.
I G+S
Multiplying power of shunt, n = =
Ig S
Voltmeter

I  Ig Ig R
G
G
I Ig Ig

V
Ammeter

Fig. 16.9 Fig. 16.10


(ii) Range extension of PMMC voltmeter. In order to measure large voltages, a suitable
high resistance R (called multiplier) is used. A moving coil meter in series with a high resist-
ance is called a voltmeter. The value of series resistance is chosen according to the maximum
voltage we wish to measure. Suppose we want to read V volts at full-scale using a moving coil
meter having full-scale deflection current Ig and resistance G. This means that when potential
difference across the voltmeter is V volts, we want that current through the meter should be Ig
as shown in Fig. 16.10. For this purpose, we connect a suitable high resistance R in series with
the meter so that current through the meter is Ig. Referring to Fig. 16.10 and applying Ohms
law, we have,
V V
Ig = orG + R =
G+R Ig
V
R = G
Ig
Resistance of voltmeter, Rm = G + R
Electrical Measuring Instruments 457
............................................................................................................................................................................................................................................................................................................................................................

Since the value of R is large, the resistance of the voltmeter will be very high. Thus the
series resistance has not only extended the voltage range but it has also increased the resistance
of the voltmeter.
Voltage to be measured V
Voltage amplification = =
Voltage across meter Ig G
I g (G + R) G + R R
= = =1+
IgG G G
Applications. Permanent-magnet moving coil instruments are acknowledged to be the best
type for all d.c. measurements. They are very sensitive and maintain a high degree of accuracy
over long periods. The chief applications of such instruments are :
(a) In the measurement of direct currents and voltages.
(b) In d.c. galvanometers to measure small currents.
(c) In ballistic galvanometers used mainly for measuring changes of magnetic flux linkages.
7. Dynamometer Type Instruments (Ammeters and Voltmeters)
These instruments can be used as ammeters or voltmeters but are generally used as wattmeters.
They are suitable for d.c. as well as a.c. work. The operating principle of such instruments is
that mechanical force exists between the current carrying conductors.
Fig. 16.11 shows the simplified diagram of a dynamometer type instrument. It essentially
consists of a fixed coil and a moving coil. The fixed coil is split into two equal parts (F, F)
which are placed close together and parallel to each other. The moving coil (M) is pivoted in
between the two fixed coils and carries a pointer as shown in Fig. 16.11. The current is led into
and out of the moving coil by means of two spiral hair-springs which also provide the controlling
torque. *Air friction damping is provided by means of the aluminium vanes that move in the
sector shaped chamber at the bottom of the instrument.

Balance
Weight
Movable
Coil
M

F F

Fixed Coils
(i) (ii)
Fig. 16.11
Working. For use as an ammeter or voltmeter, the fixed coils FF and the moving coil
M are so connected that the same current flows through the two coils. Due to these currents,
mechanical force exists between the coils. The result is that moving coil M moves the pointer
over the scale. The pointer comes to rest at a position where deflecting torque is equal to the
controlling torque. Since the polarity of the fields produced by both fixed and moving coils is
* Since the coils are air-cored, the operating magnetic field is very weak. For this reason, eddy current damp-
ing cannot be provided.
458 Objective Electrical Technology
...........................................................................................................................................................................................................................................................................................................................................................

reversed by the reversal of current, the deflection of the moving I

system is always in the same direction regardless of the direction


of current through the coils. For this reason, dynamometer I M
instruments can be used for both d.c. and a.c. measurements.
F F
The force of attraction or repulsion between the fixed and
moving coils is directly proportional to the product of ampere- I
turns of fixed coils and the moving coil i.e.,
Deflecting torque, Td Nf If Nm Im Fig. 16.12
Now, Nm and Nf are constants, so that :
Td If Im
Since the instrument is spring-controlled, the controlling torque is proportional to the angular
deflection i.e.,
TC
In the steady position of deflection, Td = TC.
If Im
Thus deflection () is directly proportional to the product of currents in the fixed coils and
the moving coil.
(i) As ammeter. When the instrument is used as an ammeter, the fixed coils and the mov-
ing coil are connected in series so that the same current flows through the two coils as shown
in Fig. 16.12. In that case, If = Im = I so that :
I2
For measuring large currents, the moving coil is shunted; the shunt being in series with the
fixed coils as shown in Fig. 16.13. The fixed coils carry the main current while the moving coil
carries a current proportional to the main current.
Im

Shunt
M M
If I
F F If F F

RS I
Multiplier

Fig. 16.13 Fig. 16.14


(ii) As voltmeter. When the instrument is used as a voltmeter, both fixed coils and the
moving coil are connected in series together with a high resistance RS (called multiplier) having
a negligible temperature co-efficient as shown in Fig. 16.14. Therefore, current in both the coils
is the same and is proportional to the voltage V being measured.
V2
It may be seen that whether the instrument is used as an ammeter or voltmeter, the deflection
is directly proportional to the square of quantity (current or voltage) being measured. Hence the
scale of dynamometer ammeter and voltmeter is not uniform ; being crowded at the beginning
and open at the upper end of the scale.
Electrical Measuring Instruments 459
............................................................................................................................................................................................................................................................................................................................................................

8. Moving Iron Ammeters and Voltmeters


This type of instrument is principally used for the measurement of alternating currents and
voltages, though it can also be used for d.c. measurements. There are two types of moving-iron
instruments viz attraction type and repulsion type.
(i) Attraction type. Fig. 16.15 shows the Air Chamber
constructional details of an attraction type moving-iron
instrument. It consists of a cylindrical coil or solenoid
which is kept fixed. An oval-shaped soft-iron is attached
Control
to the spindle in such a way that it can move in and out Spring
of the coil. A pointer is attached to the spindle so that
it is deflected with the motion of the soft-iron piece.
The controlling torque is provided by one spiral spring
arranged at the top of the moving element. The damping
torque is provided by an aluminium vane, attached to the
spindle, which moves in a closed air chamber.
Balance
Working. When the instrument is connected in the Weights
circuit to measure current or voltage, the operating current Moving Iron
flowing through the coil sets up a magnetic field. In other
Fig. 16.15
words, the coil behaves like a magnet and, therefore, it
attracts the soft-iron piece towards it. The result is that the pointer attached to the moving system
moves from zero position. The pointer will come to rest at a position where deflecting torque is
equal to the controlling torque. If current in the coil is reversed, the direction of magnetic field
also reverses and so does the magnetism produced in the soft-iron piece. Hence, the direction
of the deflecting torque remains unchanged. For this reason, such instruments can be used for
both d.c. and a.c. measurements.
Deflecting torque. The force F pulling the soft-iron piece towards the coil is directly
proportional to
(a) field strength H produced by the coil
(b) pole strength m developed in the iron piece
F m H
H2 ( m H)
Instantaneous deflecting torque H2
If the permeability of iron is assumed constant, then,
H i where i is the instantaneous coil current
Instantaneous deflecting torque i2
Average deflecting torque, Td mean of i2 over a cycle
Since the instrument is spring controlled,
TC
In the steady position of deflection, Td = TC .
mean of i2 over a cycle
I2 ...for d.c.
I r2.m.s. ...for a.c.
Since the deflection is proportional to the square of coil current, the scale of such instruments
is non-uniform; being crowded in the beginning and spread out near the finish end of the scale.
460 Objective Electrical Technology
...........................................................................................................................................................................................................................................................................................................................................................

(ii) Repulsion type. Fig. 16.16 shows the


Balance Weights
constructional details of a repulsion type moving-
iron instrument. It consists of two soft-iron pieces
or vanes surrounded by a fixed cylindrical hollow
coil which carries the operating current. One
of these vanes is fixed and the other is free to
Control
move as shown in Fig. 16.16. The movable vane Spring
is of cylindrical shape and is mounted axially Damping
Paddle Coil
on a spindle to which a pointer is attached. The
fixed vane, which is wedge-shaped and has a Fixed Iron
larger radius, is attached to the stationary coil. Moving Iron
The controlling torque is provided by one spiral Spindle
spring at the top of the instrument. It may be Fig. 16.16
noted that in this instrument, springs do not
provide the electrical connections. Damping is provided by air friction due to the motion of a
piston in an air chamber.
Working. When current to be measured or current proportional to the voltage to be measured
flows through the coil, a magnetic field is set up by the coil. This magnetic field magnetises
the two vanes in the same direction i.e., similar polarities are developed at the same ends of
the vanes. Since the adjacent edges of the vanes are of the same polarity, the two vanes repel
each other. As the fixed vane cannot move, the movable vane deflects and causes the pointer to
move from zero position. The pointer will come to rest at a position where deflecting torque is
equal to the controlling torque provided by the spring. If the current in the coil is reversed, the
direction of deflecting torque remains unchanged. It is because reversal of the field of the coil
reverses the magnetisation of both iron vanes so that they repel each other regardless of which
way the current flows through the coil. For this reason, such instruments can be used for both
d.c. and a.c. applications.
Deflecting torque. The deflecting torque results due to the repulsion between the similarly
charged soft-iron pieces or vanes. If the two pieces develop pole strengths of m1 and m2 respectively,
then,
Instantaneous deflecting torque m1 m2 H2
If the permeability of iron is assumed constant, then,
H i where i is coil current
Instantaneous deflecting torque i2
Average deflecting torque, Td mean of i2 over a cycle
Since the instrument is spring-controlled,
TC
In the steady position of deflection, Td = TC .
mean of i2 over a cycle
I2 ... for d.c.
I r2.m.s. ... for a.c.
Thus, the deflection is proportional to the square of coil current as is the case with attraction
type moving-iron instrument. Therefore, the scale of such instruments is also non-uniform;
being crowded in the beginning and spread out near the finish end of the scale. However, the
non-linearity of the scale can be corrected to some extent by the accurate shaping (e.g. using
tongue-shaped vanes) and positioning of iron vanes in relation to the operating coil.
Electrical Measuring Instruments 461
............................................................................................................................................................................................................................................................................................................................................................

9. Extending Range of Moving-Iron Instruments


As explained above, moving-iron instruments are used mainly on a.c. circuits. Therefore,
range extension shall be discussed with reference to a.c. measurements.
(i) Ammeter. Shunts are not used to extend the range of moving-iron a.c. ammeters. It is
because the division of current between the operating coil and the shunt varies with frequency
(since reactance of the coil depends upon frequency). In practice, the range of moving-iron a.c.
ammeter is extended by one of the following two methods :
(a) By changing the number of turns of the operating coil. For example, suppose that
full-scale deflection is obtained with 400 ampere-turns. For full-scale reading with 100A, the
number of turns required would be = 400/100 = 4.
Similarly, for full-scale reading with 50 A, the number of turns required is = 400/50 = 8.
Thus the ammeter can be arranged to have different ranges by merely having different number
of turns on the coil. Since the coil carries the whole of the current to be measured, it has a few
turns of thick wire. The usual ranges obtained by this method are 0250 A.
(b) For ranges above 0 250 A, a current transformer is used in conjunction with
0 5 A a.c. ammeter as shown in Fig. 16.17. The current transformer is a step-up transformer
i.e., number of secondary turns is more than the primary turns. The primary of this transformer
is connected in series with the load and carries the load current. The a.c. ammeter is connected
across the secondary of the transformer. Since in Fig. 16.17, the current transformer ratio is
10 : 1, it means that the line (or load) current is equal to 10 times the reading on the a.c. meter.
Load current, IL = 3 10 = 30 A
IL
Load

VL

C. T. Ratio 10:1 P. T. Ratio 20:1

A V
3A 100 V

Fig. 16.17
(ii) Voltmeter. The range of a moving-iron a.c. voltmeter is extended by connecting a high
resistance (multiplier) in series with it. For ranges higher than 0 750 V, where power wasted
in the multiplier would be excessive, a 0 110 V a.c. voltmeter is used in conjunction with a
potential transformer as shown in Fig. 16.17. The potential transformer is a stepdown transformer
i.e. number of primary turns is more than the secondary turns. The primary of the transformer
is connected across the load across which voltage is to be measured. The a.c. voltmeter is con-
nected across the secondary. Since in Fig. 16.17, the potential transformer ratio is 20 : 1, the
load voltage is equal to 20 times the reading on the a.c. voltmeter.
Load voltage, VL = 100 20 = 2000 V
Note that both secondaries of the instrument transformers are grounded as a safety measure.
462 Objective Electrical Technology
...........................................................................................................................................................................................................................................................................................................................................................

10. Electrostatic Voltmeters


These instruments are based on the fact that an electric force (attraction or repulsion) exists
between charged plates or objects. An electrostatic voltmeter is essentially an air condenser;
one plate is fixed while the other, which is coupled to the pointer, is free to rotate on jewelled
bearings. When p.d. to be measured is applied across the plates, the electric force between the
plates gives rise to a deflecting torque. Under the action of deflecting torque, the movable plate
moves and causes the deflection of the pointer to indicate the voltage being measured. Such
instruments can be used to measure direct as well as alternating voltages.
There are three types of electrostatic voltmeters viz.:
(i) Attracted disc type usual range from 500 V to 500 kV
(ii) Quadrant type usual range from 250 V to 10 kV
(iii) Multicellular type usual range from 30 V to 300 V
Two things are worth noting about electrostatic voltmeters. First, the deflecting torque is very
small for low voltages. For this reason, they are not very sensitive to measure small voltages.
Secondly, the instrument is only available for the measurement of p.d., that is to say as voltmeter.
It cannot be used as an *ammeter.
(i) Attracted disc type voltmeter. Fig. 16.18 shows the simplified diagram of an attracted
disc electrostatic voltmeter. It consists of two mushroom-shaped plates A and B, each mounted
on insulated pedestal. The plate B is fixed while the plate A (negative, for direct voltage) has a
movable central portion-the attracted disc. The movable plate A is attached to a horizontal rod
which is suspended by two phosphor bronze strips.
A B
When p.d. to be measured is applied across the plates,
the plate A moves towards the fixed plate B and actu-
ates the pointer via a pulley or link mechanism. The
control force is provided by gravity and damping force
by air dash pot. If the plates are too close together or if
the applied voltage is too high, a spark discharge may
occur. In order to prevent such a possibility, a ballast
resistor is included in the circuit. The function of this
resistor is to limit the current if any sparking-over oc-
curs. If the applied voltage reverses in polarity, there
is a simultaneous change in the sign of charge on the
plates so that the direction of deflecting force remains  +

unchanged. Hence such instruments can be used for Fig. 16.18


both d.c. and a.c. measurements.
Theory. The force of attraction F between the charged plates is given by ;
1 dC 2
F = V
2 dx
where x = distance between the plates
C = capacitance between the plates
V = applied voltage
Since x is always small, dC/dx is practically constant.
F V2
Obviously, the scale of the instrument will be non-uniform.
* When used as an ammeter, there will be a few millivolts voltage-across the instrument. This extremely
small p.d. is insufficient to produce any deflecting torque.
Electrical Measuring Instruments 463
............................................................................................................................................................................................................................................................................................................................................................

(ii) Quadrant type voltmeter. Fig. 16.19 shows the simplified diagram of a quadrant
electrostatic voltmeter. It consists of a light aluminium vane A suspended by a phosphor-bronze
string mid-way between two inter-connected quadrant shaped brass plates BB. One terminal is
joined to fixed plates BB (positive for direct voltage) 
and the other to the movable plate A (negative for
B B
direct voltage). The controlling torque is provided
A
by the torsion of the suspension string. Damping is V
provided by air friction due to the motion of another
vane in a partially closed box.
Working. When the instrument is connected +
in the circuit to measure the p.d., an electric force
exists between the plates. Consequently, the movable Moving Vane
vane A moves inbetween the fixed plates and causes
the deflection of the pointer. The pointer comes to
rest at a position where deflecting torque is equal to
B B
the controlling torque. Since the force of attraction
between the movable plate A and the fixed plates BB
is directly proportional to (p.d.)2, the instrument can be
used to measure either direct or alternating voltages. Fixed Plates
When used in an a.c. circuit, it reads the r.m.s. values.
More robust but *less accurate voltmeters are made Fig. 16.19
by pivoting the moving system. In pivoted voltmeters, the controlling torque is provided by a
spiral spring.
Theory. The capacitance C between the plates depends upon deflection i.e., upon the
position of the movable plate (or vane) A. Suppose that at any instant, the applied alternating
voltage is v.
1
Electrostatic energy at that instant = Cv 2
2
Since the capacitance between the plates depends upon deflection , the instantaneous
deflecting torque T d is given by ;
1 dC 2
Td = v
2 d
Average deflecting torque, Td = Average of Td over a cycle
1 T 1 dC 2
=
T 0 2 d
v dt
1 dC 1 T
=
2 d T 0
v 2 dt

1 dC 2
V Td =
2 d
where V = r.m.s. value of alternating voltage
This equation equally applies to direct voltages. If dC/d were constant, then,
Td V2

* Due to pivot friction, the pivoted voltmeters are less accurate than the suspension type. For this reason,
low voltage electrostatic voltmeters are always of suspension type.
464 Objective Electrical Technology
...........................................................................................................................................................................................................................................................................................................................................................

Hence the instrument has non-uniform scale. The non-linearity in the scale can be corrected
by shaping the movable vane A in such a way as to increase dC/d for small deflections and to
make the scale nearly uniform for larger ones.
(iii) Multicellular electrostatic voltmeter. The major drawback of quadrant type voltmeter
is that deflecting torque is very *small for low voltages. Therefore, such an instrument cannot
measure accurately voltages below 250 V. This diffi- Torsion Head (H)
culty has been overcome in a multicellular electrostatic
voltmeter which can read as low as 30 volts.
Coach Spring (S)
Fig. 16.20 shows the constructional details of a
multicellular voltmeter. It is essentially a quadrant Phosphor Bronze
String
type voltmeter with the difference that it has ten
moving vanes instead of one and eleven fixed plates Safety Sleeve (E)
forming cells in and out of which the vanes move.
The moving vanes are fixed to a vertical spindle Guide Stop (G)
and suspended by a phosphor-bronze wire so that Fixed Plates
the vanes are free to move, each between a pair
of fixed plates. At the lower end of the spindle, an
aluminium disc hangs horizontally in an oil bath Moving Vanes
and provides damping torque due to fluid friction.
The controlling torque is provided by the torsion
of the suspension wire as the moving system rotates.
The upper end of the suspension wire is attached
through a coach spring S to a torsion head H. The
torsion head is provided with a tangent screw for
zero adjustment. The function of the coach spring is Damping Vane
to prevent the suspension wire from breaking when
accidentally jerked. Should the moving vanes be jerked
downward, then the coach spring yields sufficiently to
allow the safety sleeve E to come into contact with
Fig. 16.20
the guide stop G before the suspension wire is over
strained. The scale is horizontal if the pointer is straight but the indications can be given on a
vertical scale by bending the pointer at right angles.
The working principle of multicellular voltmeter is exactly similar to the quadrant type.
By using a number of inter-leaved stationary and moving plates, we are able to increase the
capacitance and hence the deflecting torque. Consequently, the multicellular voltmeter is much
more sensitive than the quadrant type and can accurately measure low voltages.
11. Range Extension of Electrostatic Voltmeters
The range of electrostatic voltmeters can be increased by the use of multipliers. Two types
of multipliers are employed for this purpose viz.
(i) Resistance potential divider for ranges upto 40 kV
(ii) Capacitance potential divider for ranges upto 1000 kV
The first method can be used for both direct and alternating voltages whereas the second
method is suitable only for alternating voltages.
1
* Deflecting torque also depends upon the capacitance between plates [i.e. Td = (dC/d) V2]. In a quadrant
2
voltmeter, the capacitance cannot be increased since the number of vanes is limited by space consideration.
Electrical Measuring Instruments 465
............................................................................................................................................................................................................................................................................................................................................................

(i) Resistance potential divider. This divider consists of a high resistance with tappings
taken off at intermediate points. The voltage V to be measured is applied across the whole of
the potential divider and the electrostatic voltmeter connected across part of it (resistance r in
this case) as shown in Fig. 16.21. Since the voltmeter *practically carries no current, the p.d. v
across it is in the same fraction of the applied voltage V as the resistance across it (i.e. r) is of
the whole resistance (i.e. R) i.e.,
V R
Multiplying factor, =
v r
Thus if the voltmeter is connected across 1/5 of the whole resistance (i.e. R/r = 5), then
voltage V to be measured is 5 times the reading of the voltmeter. The advantage of this method
is that there is no shunting effect of the voltmeter. The drawback is that there is power loss in
the resistance divider.

R
V V Cv
r v
v

Fig. 16.21 Fig. 16.22


(ii) Capacitance potential divider. In this method, a single capacitor of capacitance C is
connected in series with the voltmeter and the whole circuit is connected across the voltage V
to be measured as shown in Fig. 16.22. Let v volts be the reading of the voltmeter. Since the
voltage across a capacitor is inversely proportional to its capacitance,
*
C + Cv 1
V **andv
C Cv Cv
V C + Cv C
Multiplying factor, = = 1+ v
v C C
By using capacitors of different capacitances, different voltage ranges can be obtained. This
method has the advantage that the circuit consumes no power. However, the drawback is that
capacitance current taken is greatly increased.
12. Induction Type Instruments
This class of instruments is suitable only for a.c. measurements. These instruments may be
used either as ammeter or voltmeter or wattmeter or energy meter. Perhaps the widest application
of induction principle is in watt-hour or energy meter.
Principle. Fig. 16.23 illustrates the principle of induction type instruments. Two alternating
fluxes 1 and 2 (whose magnitudes depend upon the current or voltage to be measured) having

* An electrostatic voltmeter is essentially an air capacitor. For direct voltages, no current can flow through
it. For alternating voltages, the current through the voltmeter is extremely small.
C Cv
** Total circuit capacitance = because C and Cv are in series.
C + Cv
466 Objective Electrical Technology
...........................................................................................................................................................................................................................................................................................................................................................

a phase difference pass through a A B


metallic disc, usually of copper or
aluminium. These alternating fluxes
induce currents in the disc. The 1 2
current produced by one flux reacts Aluminium
with the other flux, and vice-versa, to Disc
produce the deflecting torque that acts
Fig. 16.23
on the disc. It can be proved that net
deflecting torque on the disc is given by ;
Td 1m 2m sin
where 1m = maximum value of alternating flux 1
2m = maximum value of alternating flux 2
Obviously, to obtain maximum deflecting torque, the angle (i.e. phase angle between 1
and 2) should be 90.
The induction type instruments are worked on single phase. The question arises how to obtain
two fluxes having a phase difference (= 90 as far as possible) from a single phase supply.
This can be achieved in two ways viz.
(i) Splitting the phase (i.e. Ferraris Principle) (ii) Shaded-pole arrangement
(i) Splitting the phase. In this method, two flux-producing windings are connected in
parallel across a single phase supply; an inductive coil L in series with one and a resistance R
in series with the other. The values of R and L are so selected that currents through the two
windings [i.e. IR and IL in Fig. 16.24 (i)] have a phase difference of nearly 90. The result is
that we have two alternating fluxes with a relative phase shift of 90. These fluxes pass through
the aluminium disc and induce currents in it to produce the necessary driving torque.
The fluxes produced by the two currents may be represented as :
1 = 1m sin t ; 2 = 2m sin (t + )
where is the phase angle by which 2 leads 1.
Aluminium
Disc
R

Winding
IR 2 i2
2
1
IL i1
Winding F2
1  1 F1
Supply

L
(i) (ii)
Fig. 16.24
The two fluxes 1 and 2 will induce e.m.f.s e1 and e2 respectively in the disc. Assuming
r to be the resistance offered by the disc to each induced e.m.f., the induced currents are given
by ;
e d / dt 1 d 1m cos t
i1 = 1 = 1 = (1m sin t ) =
r r r dt r
Electrical Measuring Instruments 467
............................................................................................................................................................................................................................................................................................................................................................

or i1 1m cos t .... as r and are constant


Similarly, i2 2m cos (t + )
The portion of the disc which is traversed by flux 1 and carries currents i2 experiences a
force F1 along the direction indicated. The magnitude of this force is given by ;
F1 1 i2
Similarly, F2 2 i1
Since the direction of both fluxes and both currents are the same, these forces will be in
the opposite directions. This can be easily ascertained by applying right hand rule.
Resultant force, F F2 F1 (2 i1 1 i2)
1m 2m [sin ( t + ) cos t sin t cos ( t + )]
1m 2m sin
This resultant force will produce the deflecting torque Td which is directly proportional to it.
Td 1m 2m sin
(a) If = 0 (i.e. the two fluxes are in phase), then deflecting torque is zero. The deflecting
torque will be maximum when = 90 i.e. when the alternating fluxes have a phase difference
of 90.
(b) The deflecting torque is the same at every instant since 1m , 2m and are fixed for a
given condition.
(c) The direction of deflecting torque depends upon which flux is leading the other. The
deflecting torque acts in such a direction so as to rotate the disc from the point where the leading
flux passes the disc towards the point where the lagging flux passes the disc.
(ii) Shaded-pole arrangement. The shaded-pole structure differs from the split-phase type
in that there is only one flux-producing winding connected to a.c. supply. The flux produced
by this winding is split into two portions 1 and 2 having a phase difference of by shaded-
pole arrangement as shown in Fig. 16.25. In this arrangement, one-half of each pole (on the
same side) embraces a thick short-circuited copper loop called a shading coil. The shading coil
acts as a short-circuited secondary and the main winding as a primary. Induced currents in the
*shading coil cause the flux 1 in the shaded portion to lag the flux 2 in the unshaded portion
by (= 40 to 50). This displacement between the two fluxes produce the necessary deflecting
torque given by ;
Td 1m 2m sin
3
1 2
0

 Shading
Coil
2 1

Disc
Disc
Damping
Magnet

Fig. 16.25 Fig. 16.26


* Note that shading coil serves the same purpose as connecting resistance and inductance in split-phase
arrangement.
468 Objective Electrical Technology
...........................................................................................................................................................................................................................................................................................................................................................

13. Induction Ammeters and Voltmeters


Induction ammeters and voltmeters can be used for a.c. measurement only and can be of
shaded-pole type or split-phase type.
I. Shaded-pole type. Fig. 16.26 shows the principal parts of a shaded-pole type instru-
ment. It consists of a specially shaped aluminium disc coupled to a pointer and suspended in
jewelled bearings. The disc passes through two air-gaps; the first located in an electromagnet
having a shading coil and the second in a permanent magnet. The permanent magnet provides
the necessary damping torque. The controlling torque is provided by a spiral spring attached to
the moving system. As shown above, the deflecting torque is given by ;
Td 1m 2m sin
(i) When used as an ammeter, the current to be measured or a part of it is passed through
the operating coil of the instrument. Since both the fluxes are produced by the same alternating
current I (r.m.s. value),
Td I2
As the instrument is spring controlled, TC .
I2
(ii) When used as a voltmeter, current proportional to the voltage to be measured is passed
through the operating coil.
Td V2
Again the instrument is spring controlled, TC .
V2
It is clear that shaded-pole instruments have uneven scale, being crowded in the beginning
and open near the end of the scale. However, the non-linearity in the scale can be corrected to
a considerable extent by modifying the shape of the disc i.e., by using cam-shaped disc.
II. Split-phase type. In this method, the windings of the two electromagnets A and B are
connected in parallel across a single phase sup-

ply; an inductive coil L in series with one and a Supply
resistance R in series with the other. The values A
of R and L are so selected that the currents IR
B
IL
through the two windings [See Fig. 16.27] have
a phase difference of nearly 90o. This produces L
R Disc
the deflecting torque on the aluminium disc. The
permanent magnet provides the necessary damp-
ing torque. The controlling torque is provided by
a spiral spring attached to the moving system. As
shown above, the deflecting torque is given by ;
Damping Magnet
Td 1m 2m sin
Both fluxes are proportional to current or Fig. 16.27
voltage to be measured.
Td I2 ...for ammeter
V2 ...for voltmeter
As the instrument is spring controlled, TC .
I2 ...for ammeter
V2 ...for voltmeter
Obviously, the scale of this type of instrument is also non-uniform.
Electrical Measuring Instruments 469
............................................................................................................................................................................................................................................................................................................................................................

14. Potentiometer
A potentiometer is an accurate device for measuring the e.m.f. of a cell or potential difference
(p.d.) between two points of an electric circuit.
Principle. If a wire of uniform area of cross-section is carrying a steady current, then fall
of potential across any portion of the wire is directly proportional to the length of that portion.
E Rh
A

K1

E Rh J
A B

l
A C I B
G
K2
V E1

Fig. 16.28 Fig. 16.29


Consider a wire AB of uniform area of X-section and carrying a steady current I as shown
in Fig. 16.28. Consider a portion AC (= l) of this wire. Suppose the p.d. across AC is V i.e. fall
of potential across length l of the wire is V. If A is the area of cross-section of the wire and
is the resistivity of the wire, then,
l
Resistance of portion AC, R =
A
l
Now, V = I R or V = I
A
Since I, and A are constant, V l.
Construction. Fig. 16.29 shows the potentiometer in its simplest form. It consists of a long
uniform wire AB made of manganin stretched on a wooden board. A graduated scale (not shown
in the figure) runs parallel to the wire. The end A of the wire is connected to the positive terminal
of the battery E (driver battery) whose negative terminal is connected to the end B through an
ammeter, rheostat Rh and key K1. The current in wire AB can be changed with the help of the
rheostat. The positive terminal of the cell whose e.m.f. E1 is to be measured is connected to the
end A of the wire. The negative terminal of this cell is connected through a galvanometer G
and key K2 to a jockey J which can slide along the wire AB and can make contact at any point
on the wire. First key K1 is closed and rheostat is set at one position to obtain steady current in
wire AB. Now key K2 is also closed and the jockey is moved over the wire to get the balance
point. The potentiometer is said to be balanced if the jockey is at such a position on the wire
AB that on pressing the jockey, there is no current in the galvanometer.
15. D. C. Potentiometer Measurements
The potentiometer basically measures the potential difference between two points but on
this basis, it can be used to make several other measurements. A few common applications of
potentiometer are given below :
(i) Determination of e.m.f. of a cell. Fig. 16.30 shows the arrangement for determining the
e.m.f. of a cell with the help of a potentiometer. The positive terminal of the cell whose e.m.f.
E1 is to be determined is connected to the positive terminal of the battery E. Following usual
procedure, null point is obtained at point J.
470 Objective Electrical Technology
...........................................................................................................................................................................................................................................................................................................................................................

At null point, the p.d. between points A and J is equal to the e.m.f. of the cell. If resistance
of 1 cm of wire AB is r and length AJ = l cm, then,
P.D. across A J = I (l r) or E.M.F. of cell, E1 = I l r or E1 = k l
where k (= I r) is the p.d. across 1 cm length of wire AB. Note that E1 l.
Note that potentiometer method is a null method for determining the e.m.f. of a cell.
E E Rh K1
Rh K1
A A

I
I

J J1 J2
A B A B
E1
1
G
3
G 2
K2 E2
E1

Fig. 16.30 Fig. 16.31


(ii) Comparison of e.m.f.s of two cells. Fig. 16.31 shows the arrangement for comparing
the e.m.f.s E1and E2 of two cells with the help of potentiometer. The positive terminals of the
cells (E1 and E2) are connected to the positive terminal of the battery E. The negative terminals
of the cells are connected to terminals 1 and 2 of a two-way key. The common terminal 3 of
the two-way key is connected to the jockey through a galvanometer G.
First null point J1 is obtained with cell E1 alone. Suppose AJ1 = l1. Then E1 l1. Now null
point is obtained with cell E2 alone at point J2. Suppose AJ2 = l2. Then E2 l2.
E1 l
\ = 1
E2 l2
Note. The e.m.f. E of the battery should be greater than the e.m.f. E1 or E2 otherwise null point will
not be obtained. Further, a shunt must be used with the galvanometer initially and near the null point, it
should be removed.
(iii) Determination of internal resistance of a cell. Fig. 16.32 shows the arrangement for
determining the internal resistance (r) of a cell of Battery
Rh K1
e.m.f. E with the help of a potentiometer. The posi- A
tive terminal of the cell is connected to the posi- I
tive terminal of the battery. The negative terminal
of the cell is connected to the jockey through a
galvanometer G. A resistance box R is connected A J2 J1
B
across the cell through key K2.
(a) The key K1 is closed and current in poten- G
E
tiometer wire AB is adjusted to a suitable
constant value (say I) with the help of
rheostat. The setting of the rheostat is not R
to be disturbed throughout the experiment. K2

(b) Keeping key K open, the position of Fig. 16.32


2
jockey is adjusted till null point is obtained, i.e., galvanometer reads zero. Let the null
point be obtained at point J1 on the potentiometer wire and let distance AJ1 = l1.
E.M.F. of cell, E l1
Electrical Measuring Instruments 471
............................................................................................................................................................................................................................................................................................................................................................

(c) Now suitable resistance R is inserted and key K2 is closed. Again the position of the null
point is obtained on the potentiometer wire. Now the null point corresponds to the p.d.
V across the terminals of the cell. If the null point is obtained at point J2 and distance
AJ2 = l2, then, P.D. across the cell, V l2.
E l
= 1
V l2

E V E l1 l1 l2
Internal resistance of the cell, r = R = V 1 R = l 1 R = l R
V 2 2
l l
r = 1 2 R
l2
Since the values of l1, l2 and R are known, the value of internal resistance r of the cell can
be determined.
16. Ohmmeter
A device that measures the resistance directly is called an ohmmeter. The simplest direct
reading ohmmeter is the basic series ohmmeter circuit shown in Fig. 16.33 (i). It consists of a
permanent magnet moving coil (PMMC) instrument in series with a battery and a rheostat R.
Note that A and B are the terminals of ohmmeter.
PMMC R
A
M
Infinite Zero
Resistance Ohms Resistance
E Rx 30 k
90 k 10 k
 0
Ohmmeter Scale

(i) B (ii)
Fig. 16.33
(i) With terminals A and B shorted together, R is adjusted for full-scale deflection (f.s.d.).
Since terminals A and B are shorted, the ohmmeter should read zero resistance. Thus
as shown in Fig. 16.33 (ii), the full-scale deflection should read zero resistance.
E
Full-scale deflection current, Ig = (neglecting meter resistance)
R
(ii) When terminals A and B are open-circuited, the pointer should indicate infinity. Therefore
zero deflection point on the scale is marked as infinite resistance [See Fig. 16.33 (ii)].
(iii) When an unknown resistance Rx is connected to terminals A and B, the meter current
Im is
E
Im =
R + Rx
Since the value of Im is more than zero and less than Ig (f.s.d.), the meter will give a
reading between zero and infinity. Thus the value of unknown resistance Rx can be determined.
Note. The circuit shown in Fig. 16.33 (i) relies upon battery voltage remaining absolutely constant.
When the battery terminal voltage falls (as they all do with use), the instrument scale is no longer accurate.
Thus some means of adjusting for battery voltage variations must be built in the circuit.
472 Objective Electrical Technology
...........................................................................................................................................................................................................................................................................................................................................................

17. Megger
The megohmmeter (or megger) is an instrument for measuring very high resistance such as
the insulation resistance of electrical cables. A megger is essentially an ohmmeter with its own
hand-cranked high voltage generator (See Fig. 16.34). The generated voltage may be anything
from 100 V to 2.5 kV. The high voltage source is required to pass a measurable current through
the high resistance to be measured.
Hand Crank

Cable Under Test 0

Fig. 16.34
As in the case of ohmmeter, the scale of the megger indicates infinity () when its terminals
are open-circuited and zero when the megger terminals are shorted. When the unknown resistance
is connected to megger terminals, it gives the value of resistance between zero and infinity. The
range of the instrument can be changed by switching different values of standard resistor in the
circuit.
18. Wattmeters
A wattmeter, as its name implies, measures electric power given to or developed by an
electric apparatus or circuit. A wattmeter is hardly ever required in a d.c. circuit because power (P =
V I) can be easily determined from voltmeter and ammeter readings. However, in an a.c. circuit,
such a computation is generaly *speaking impossible. It is because in an a.c. circuit, power
(P = V I cos ) depends not only on voltage and current but also on the phase shift between
them. Therefore, a wattmeter is necessary for a.c. power measurement. There are two principal
types of wattmeters viz;
(i) Dynamometer wattmeter for both d.c. and a.c. power
(ii) Induction wattmeter for a.c. power only
19. Dynamometer Wattmeter
A dynamometer wattmeter is almost universally used for the measurement of d.c. as well
as a.c. power. It works on the dynamometer prinicple i.e. mechanical force exists between two
current carrying conductors or coils.
Construction. When a dynamometer instrument is used as a wattmeter, the fixed coils
are connected in series with the load and carry the load current (I1) while the moving coil is
connected across the load through a series multiplier R and carries a current (I2) proportional to
the load voltage as shown in Fig. 16.35. The fixed coil (or coils) is called the current coil and
the movable coil is known as potential coil. The controlling torque is provided by two spiral
springs which also serve the additional purpose of leading current into and out of the moving
coil. Air friction damping is provided in such instruments. A pointer is attached to the movable
coil.
* Except for the case of pure resistance when P = VI ( cos is 1 for pure resistance).
Electrical Measuring Instruments 473
............................................................................................................................................................................................................................................................................................................................................................

Current Coil

Fixed Fixed
Coil Coil
I1
R
Potential
Coil
Movable I2
V Load
I1 Coil
I2

V R Multiplier Load

(i) (ii)
Fig. 16.35
Working. When the wattmeter is connected in the circuit to measure power (See Fig. 16.35),
the current coil carries the load current and potential coil carries current proportional to the load
voltage. Due to currents in the coils, mechanical force exists between them. The result is that
movable coil moves the pointer over the scale. The pointer comes to rest at a position where
deflecting torque is equal to the controlling torque. Reversal of current reverses currents in both
the fixed coils and the movable coil so that the direction of deflecting torque remains unchanged.
Hence, such instruments can be used for the measurement of d.c. as well as a.c. power.
Deflecting torque. We shall now prove that deflecting torque is proportional to load power
in a d.c. as well as a.c. circuit.
(i) Consider that the wattmeter is connected in a d.c. circuit to measure power as shown
in Fig. 16.35 (ii). The power taken by the load is V I1.
Deflecting torque, Td I1 I2
Since I2 is directly proportional to V,
Deflecting torque, Td V I1 load power
(ii) Consider that the wattmeter is connected in an a.c. circuit to measure power. Suppose at any
instant, current through the load is i and voltage across the load is v. Let the load power factor be
cos lagging. Then,
v = Vm sin ; i = m sin ( )
Instantaneous deflecting torque vi
The pointer cannot follow the rapid changes in the instantaneous power owing to the large
inertia of the moving system. Hence the instrument indicates the mean or average power.
Average deflecting torque, Td Average of vi over a cycle
1 2

2 0
Vm I m sin sin ( ) d

V I
m m cos V I cos
2
Td load power
Thus whether the instrument is used to measure d.c. or a.c. power, deflecting torque is
proportional to load power.
474 Objective Electrical Technology
...........................................................................................................................................................................................................................................................................................................................................................

Since the instrument is spring-controlled, TC .


In the steady position of deflection, Td = TC .
load power
Hence such instruments have uniform scale.
Errors. A wattmeter may not give true reading due to several errors such as (i) error due
to connection of potential coil circuit (ii) error due to inductance of potential coil (iii) error due
to capacitance in potential coil circuit (iv) error due to stray fields and (v) error due to eddy
currents.
20. Induction Wattmeters
The induction type wattmeter can be used to measure a.c. power only in contrast to
dynamometer wattmeter which can be used to measure d.c. as well as a.c. power. The principle
of operation of an induction wattmeter is the same as that of induction ammeter and voltmeter
i.e. induction principle.
However, it differs from induction ammeter or voltmeter in so far that two seperate coils are
used to produce the rotating magnetic field in place of one coil with split phase arrangement.
Fig. 16.36 shows the physical arrangement of the various parts of an induction wattmeter.
Construction. Fig. 16.36 shows the principal parts of an induction wattmeter.
(i) It consists of two laminated electromagnets. One electromagnet, called shunt magnet is
connected across the supply and carries current proportional to the supply voltage. The coil of
this magnet is made highly inductive so that the current (and hence the flux produced) in it lags
behind the supply voltage by 90. The other electromagnet, called series magnet is connected
in series with the supply and carries the load current. The coil of this magnet is made highly
non-inductive so that angle of lag or lead is wholly determined by the load.

Shunt Magnet
Copper Shading Ring

IV

Supply Load
Aluminium
Disc

IC
Series
Magnet

Fig. 16.36
(ii) A thin aluminium disc mounted on the spindle is placed between the two magnets so
that it cuts the flux of both the magnets. The controlling torque is provided by spiral springs.
The damping is electro-magnetic and is usually provided by a permanent magnet embracing the
aluminium disc (See Fig. 16.37). Two or more closed copper rings (called shading rings) are
provided on the central limb of the shunt magnet. By adjusting the position of these rings, the
shunt magnet flux can be made to lag behind the supply voltage by exactly 90.
Electrical Measuring Instruments 475
............................................................................................................................................................................................................................................................................................................................................................

Working. When the wattmeter


is connected in the circuit (See Fig. Spring
16.36) to measure a.c. power, the shunt
Permanent
magnet carries current proportional Magnet
to the supply voltage and the series
magnet carries the load current. The
two fluxes produced by the magnets
induce eddy currents in the aluminium
Shunt
disc. The interaction between the Magnet
fluxes and eddy currents produces the
deflecting torque on the disc, causing
the pointer connected to the moving Aluminium Disc
system to move over the scale. The Series
Magnet
pointer comes to rest at a position
where deflecting torque is equal to the
controlling torque. Fig. 16.37
Let V = supply voltage
IV = current carried by shunt magnet
IC = current carried by series magnet
cos = lagging power factor of the load
The phasor diagram is shown in Fig. 16.38. The current IV V
in the shunt magnet lags the supply voltage V by 90 and so does
the flux V produced by it. The current IC in the series magnet is IC

the load current and hence lags behind the supply voltage V by
. The flux C produced by this current (i.e. IC) is in phase with it.
It is clear that phase angle between the two fluxes is 90 i.e.
C

= 90o
Mean deflecting torque, Td V C sin ...See section 12 90 
VI sin (90 ) IV
V
[ V V and C I]
V I cos Fig. 16.38
a.c. power
Since the instrument is spring controlled, TC .
For steady deflected position, Td = TC .
a.c. power
Hence such instruments have uniform scale.
Application. Due to low accuracy and high power consumption, the characteristics of induction
wattmeters are inferior to those of dynamometer wattmeters. For this reason, dynamometer
wattmeters are almost universally used for the measurement of a.c. as well as d.c. power. However,
induction wattmeters have their chief application as panel instruments where the variations in
frequency are not too much.
476 Objective Electrical Technology
...........................................................................................................................................................................................................................................................................................................................................................

21. Single Phase Induction Watthour Meter


Single phase induction watthour meters (or energy meters) are extensively used for the
measurement of electrical energy in a.c. circuits. One can find such meters installed in homes.
An induction watthour meter is essentially an induction wattmeter with control spring and pointer
removed but brake magnet and counting mechanism provided.
Construction. Fig. 16.39 shows the various parts of a single-phase induction watthour meter.
(i) It consists of (a) two a.c. electromagnets; the series magnet and shunt magnet (b) an
aluminium disc or rotor placed between the two electromagnets (c) brake magnet and
(d) counting mechanism.

Shunt Magnet Recording


Mechanism

P. F. Compen-
sator

Load
Supply
Brake
Magnet

Disc

Series Magnet

Fig. 16.39
(ii) The shunt magnet is wound with a fine wire of many turns and is connected across the
supply so that it carries current proportional to the supply voltage. Since the coil of
shunt magnet is highly *inductive, the current (and hence the flux) in it lags the supply
voltage by 90.
The series magnet is wound with a heavy wire of few turns and is connected in series with
the load so that it carries the load current. The coil of this magnet is highly non-inductive so
that angle of lag or lead is determined wholly by the load.
(iii) A thin aluminium disc mounted on the spindle is placed between the shunt and series
magnets so that it cuts the fluxes of both the magnets.
(iv) The braking torque is obtained by placing a permanent magnet near the rotating disc
so that the disc rotates in the field established by the permanent magnet. Eddy currents
induced in the disc produce a braking or retarding torque that is proportional to the disc
speed.
(v) A short-circuited copper loop (also known as power factor compensator) is provided on
the central limb of the shunt magnet. By adjusting the position of this loop, the shunt
magnet flux can be made to lag behind the supply voltage exactly by 90.
Frictional compensation is obtained by means of two adjustable short-circuited loops placed
in the leakage gaps of the shunt magnet. Geared to the rotating element is counting mechanism
which indicates the energy consumed directly in kilowatthours (kWh).

* The coil has a very large number of turns and the reluctance of its magnetic circuit is very small due to the
presence of small air gaps. This makes the coil highly inductive.
Electrical Measuring Instruments 477
............................................................................................................................................................................................................................................................................................................................................................

Theory. When induction watthour meter is connected in the circuit to measure energy, the
shunt magnet carries current proportional to the supply voltage and the series magnet carries the
load current. Therefore, expression for the driving torque is the same as for induction wattmeter.
Referring back to the phasor diagram in Fig. 16.38,
Driving torque, Td V C sin
V I sin (sin 90 )
VI cos
power
The braking torque is due to the eddy currents induced in the aluminium disc. Since the
magnitude of eddy currents is proportional to the disc speed, the braking torque will also be
proportional to the disc speed i.e.,
Braking torque, *TB n (i.e. disc speed)
For steady speed of rotation, Td = TB.
Power n
Multiplying both sides by t, the time for which power is supplied,
Power t n t
or Energy N
where N (= nt) is the total number of revolutions in time t.
The counting mechanism is so arranged that the meter indicates kilowatthours (kWh) directly
and not the revolutions.
Meter Constant. We have seen above that :
N Energy
or N = K Energy
where K is a constant called meter constant.
N No. of revolutions
Meter constant, K = =
Energy kWh
Hence the number of revolutions made by the disc for 1 kWh of energy consumption is called
meter constant.
The meter constant is always written on the name plates of the energy meters installed in
homes, commercial and industrial establishments. If the meter constant of an energy meter is
1500 rev/kWh, it means that for consumption of 1 kWh, the disc will make 1500 revolutions.
22. Errors in Induction Watthour Meter
The users of electrical energy are charged according to the readings of the energy meters
installed in their premises. It is, therefore, very important that construction and design of energy
meters should be such as to ensure long-time accuracy i.e., they should give correct readings over
a period of several years under normal use conditions. Some of the common errors in energy
meters and their remedial measures are discussed below :
(i) Phasor error. The meter will read correctly only if the shunt magnet flux lags behind
the supply voltage by exactly 90. Since the shunt magnet coil has some resistance and is not
* Let = flux of permanent magnet ; disc speed = n
E.M.F. induced in the disc, e n
If R is the resistance of the eddy current path, then, i = e/R n/R.
Braking torque, TB i 2 n/R
The braking mechanism is so designed that and R are constant. Therefore, TB n.
478 Objective Electrical Technology
...........................................................................................................................................................................................................................................................................................................................................................

completely reactive, the shunt magnet flux does not lag the supply voltage by exactly 90o. The
result is that the meter will not read correctly at all power factors.
Adjustment. The flux in the shunt magnet can be made to lag behind the supply voltage by
exactly 90 by adjusting the position of the shading coil placed around the lower part of the
central limb of the shunt magnet. A current is induced in the shading coil by the shunt magnet
flux and causes a further displacement of the flux. By moving the shading coil up or down the
limb, the displacement between shunt magnet flux and the supply voltage can be adjusted to
90. This adjustment is known as lag adjustment or power factor adjustment.
(ii) Speed error. Sometimes the speed of the disc of the meter is either fast or slow, result-
ing in the wrong recording of energy consumption.
Adjustment. The speed of the disc of the energy meter can be adjusted to the desired value
by changing the position of the brake magnet. If the brake magnet is moved towards the centre
of the spindle, the braking torque is reduced and the disc speed is *increased. Reverse would
happen should the brake magnet be moved away from the centre of the spindle.
(iii) Frictional error. Frictional forces at the rotor bearings and in the counting mechanism
cause noticeable error especially at light loads. At light loads, the torque due to friction adds
considerably to the braking torque. Since friction torque is not proportional to the speed but is
roughly constant, it can cause considerable error in the meter reading.
Adjustment. In order to compensate for this error, it is necessary to provide a constant addition
to the driving torque that is equal and opposite to the friction torque. This is produced by means
of two adjustable short-circuited loops placed in the leakage gaps of the shunt magnet. These
loops upset the symmetry of the leakage flux and produce a small torque to oppose the friction
torque. This adjustment is known as light-load adjustment. The loops are adjusted so that when
no current is passing through the current coil (i.e. exciting coil of the series magnet), the torque
produced is just sufficient to overcome the friction in the system, without actually rotating the disc.
(iv) Creeping. Sometimes the disc of the meter makes slow but continuous rotation at no
load i.e., when potential coil is excited but with no current flowing in the load. This is called
creeping. This error may be caused due to overcompensation for friction, excessive supply volt-
age, vibrations, stray magnetic fields etc.
Adjustment. In order to prevent this creeping, two diametrically opposite holes are drilled in
the disc. This causes sufficient distortion of the field. The result is that the disc tends to remain
stationary when one of the holes comes under one of the poles of the shunt magnet.
(v) Temperature error. Since watthour meters are frequently required to operate in outdoor
installations and are subject to extreme temperatures, the effects of temperature and their com-
pensation are very important. The resistance of the disc, of the potential coil and characteristics
of magnetic circuit and the strength of brake magnet are affected by the changes in temperature.
Therefore, great care is exercised in the design of the meter to eliminate the errors due to tem-
perature variations.
(vi) Frequency variations. The meter is designed to give minimum error at a particular
frequency (generally 50 Hz). If the supply frequency changes, the reactance of the coils also
changes, resulting in a small error. Fortunately, this is not of much significance because com-
mercial frequencies are held within close limits.
(vii) Voltage variations. The shunt magnet flux will increase with an increase in voltage. The
driving torque is proportional to the first power of flux whereas braking torque is proportional
* Moving the brake magnet inwards means that speed of the part of disc under the pole face of brake magnet
will be less. This results in the lesser induced voltage in the disc and hence reduced braking torque.
Electrical Measuring Instruments 479
............................................................................................................................................................................................................................................................................................................................................................

to the square of the flux. Therefore, if the supply voltage is higher than the normal value, the
braking torque will increase much more than the driving torque and vice-versa. The result is
that the meter has the tendency to run slow at higher than normal voltages and fast at reduced
voltages. However, the effect is small for most of the meters and is not more than 0.2% to 0.3%
for a voltage change of 10% from the rated value. The small error due to voltage variations can
be eliminated by the proper design of the magnetic circuit of the shunt magnet.

SHORT ANSWER QUESTIONS


Q. 1. When is indicating instrument said to be dead-beat?
Ans. If the degree of damping is adjusted to such a value that the pointer comes up to the correct
reading quickly without passing it or oscillating about it, the instrument is said to be dead-
beat or critically damped.
Q. 2. What do you mean by a linear scale and squared scale?
Ans. (i) Linear scale is not necessarily straight as the name implies but is usually in the form of
an arc. It is called linear (or uniform) to denote that it is evenly graduated over the whole
length of the arc with equal sub-divisions.
(ii) When the deflecting torque is directly proportional to the square of the quantity (current
or voltage) to be measured and the instrument is spring controlled, we get a squared
scale. The characteristic of this scale is that it is crowded in the beginning and spread
out near the finish end of the scale.
Note. Except permanent-magnet moving coil instrument and dynamometer wattmeters, all other
indicating instruments have squared scale.
Q. 3. Why should an ammeter have low resistance?
Ans. An ammeter is used to measure current in a circuit. It is thus connected in series with the
circuit under test so that current to be measured or a fraction of it passes through the instrument
itself. Its resistance must therefore be as small as possible :
(i) since power wasted in the instrument is given by I m2 Rm where Im is the meter current
and Rm is the meter resistance.
(ii) so that it does not increase the resistance of the circuit into which it is inserted.
Q. 4. Why should a voltmeter have high resistance?
Ans. A voltmeter is used to measure the potential difference between two points of a circuit. It is
thus connected in parallel with the circuit or some part of the circuit. A voltmeter must have
high resistance so that :
(i) it is not injured by the current that flows through it.
(ii) power wasted is small as the same is given by V2/R.
(iii) it will not materially affect the current in the circuit to which it is connected.
Q. 5. What is DArsonval movement?
Ans. This movement was first conceived by French Physicist Arsene dArsonval in 1881. This
movement is due to force acting on a current carrying conductor placed in the magnetic field
of permanent magnet. This movement is so sensitive and accurate that engineers always try
to devise methods to use this movement in measurements.
Q. 6. Why is eddy current damping not provided in dynamometer type instruments?
Ans. Since the coils of dynamometer instrument are air-cored, the operating magnetic field is very
weak. For this reason, eddy current damping cannot be provided. We provide air friction
damping in such instruments.
480 Objective Electrical Technology
...........................................................................................................................................................................................................................................................................................................................................................

Q. 7. Why is dynamometer type instrument chiefly used as a wattmeter?


Ans. Dynamometer instruments are very costly. When used as ammeters and voltmeters, their
performance is inferior to moving coil and moving-iron instruments. For this reason,
dynamometer ammeters and voltmeters are rarely used. However, dynamometer wattmeter
is universally used for the measurement of d.c. as well a.c. power. The main reason is that
the scale is uniform and high degree of accuracy can be achieved by careful design. Keeping
in view these features, extra cost is justified.
Q. 8. Can you list any situation where dynamometer ammeter or voltmeter is used?
Ans. Dynamometer instruments are common as wattmeters but are seldom used as general purpose
voltmeters or ammeters. The principal use of dynamometer ammeters and voltmeters is as
transfer instruments i.e. when the instrument is required to read both direct and alternating
currents as in the case of a.c. potentiometer.
Q. 9. Why are moving-iron instruments not used for d.c. work?
Ans. The moving-iron instruments are principally used for alternating current and voltage
measurements. They can also be used for d.c. work but then they are liable to small errors
due to residual magnetism in iron. The residual magnetism can yield either an additive or a
subtractive error. Thus a second reading should be made with leads interchanged when the
meter is used on d.c. circuits. The average of the two readings gives the true value. For this
reason, moving-iron instruments are primarily used in a.c. circuits at power line frequencies.
Q. 10. What is the advantage of a squared scale over a linear scale?
Ans. In a squared scale, the deflection is proportional to the square of the coil current. If the full-
scale deflection is 100 V, the centre scale calibration is 70.7 V instead of 50 V found on a
linear scale. This spread out scale in the upper region is often very useful when the values
of current and voltage tend to fluctuate about particular values. For instance, if line voltage
fluctuates between 220 V and 230 V, it can be monitored more accurately on a squared scale
meter than on a linear scale if both meters have a full-scale deflection of 250 V.
Q. 11. Why is eddy current damping not provided in moving-iron instruments?
Ans. Eddy current damping is not provided in moving-iron instruments because the presence
of permanent magnet required for the purpose would affect the magnetic field due to the
operating coil of the instrument. This will, in turn, affect the reading of the instrument. Air
friction damping is provided in such instruments.
Q. 12. Why are shunts not used to extend the range of moving-iron ammeters?
Ans. Shunts are not used to extend the range of moving-iron ammeters. It is because the division
of current between the operating coil and the shunt varies with frequency (since reactance of
the coil depends upon frequency). In practice, the range of a moving-iron ammeter is extended
by one of the following two methods :
(i) By changing the number of turns of the operating coil.
(ii) By using a current transformer in conjunction with an a.c. ammeter.
Q. 13. Why are dynamometer instruments insensitive?
Ans. In a dynamometer type instrument, fixed coils as well as moving coil produces magnetic
field. Since energy must be used to create two magnetic fields, such instruments are generally
insensitive. Moreover, the power required for a full-scale deflection in such instruments is
much greater than that required for permanent magnet moving-coil instruments.
Q. 14. Why is the pointer of a dynamometer wattmeter in the form of a triangular truss?
Ans. The pointer of a dynamometer wattmeter is generally a triangular truss with a thin tip mounted
at the end. This construction has the following advantages :
Electrical Measuring Instruments 481
............................................................................................................................................................................................................................................................................................................................................................

(i) It makes the pointer strong without making it heavy.


(ii) The natural frequency of vibration is kept well outside that of the power at any commercial
frequency, and so steady readings are always obtainable.
Q. 15. What is the practical application of a hot-wire instrument?
Ans. Hot-wire instruments are used for high-frequency alternating currents (e.g. in wireless work)
because the inductance of hot wire is very small. Dynamometer or moving-iron instruments
are unsuitable at such high frequencies.
Q. 16. Why is an electrostatic instrument not used as an ammeter?
Ans. An electrostatic instrument is based on the principle that an electric force (attraction or
repulsion) exists between two charged plates or objects. It is essentially an air condenser ;
one plate is fixed while the other, which is coupled to the pointer, is free to rotate on jewelled
bearings. When p.d. to be measured is applied across the plates, the electric force between
the plates gives rise to a deflecting torque.
When used as an ammeter, there will be a few millivolts voltage across the instrument. This
extremely small p.d. is insufficient to produce any deflecting torque. For this reason, an
electrostatic instrument cannot be used as an ammeter. It is only used as a voltmeter.
Q. 17. Why does an electrostatic voltmeter give very accurate reading?
Ans. An electrostatic voltmeter draws negligible current when connected in the circuit. Hence
such a voltmeter does not alter the conditions of the circuit to which it is connected. In other
words, an electrostatic voltmeter gives very accurate readings.
Q. 18. Why are wattmeters shielded?
Ans. A wattmeter is generally of dynamometer type. Since dynamometer movement employs
air-cored coils, the operating magnetic field is quite weak and is easily affected by the stray
magnetic fields. Stray field errors can be reduced by enclosing the wattmeter in an iron case.
The iron case has a magnetic screening effect.
Q. 19. Why are dynamometer wattmeters always preferred to induction wattmeters for a.c. power
measurements?
Ans. Due to low accuracy and high power consumption, the characteristics of induction wattmeters
are inferior to that of dynamometer wattmeters. For this reason, dynamometer wattmeters
are almost universally used for the measurement of a.c. as well as d.c. power. However,
induction wattmeters have their chief application as panel instruments where the variations
in frequency are not too much.
Q. 20. What is the chief advantage of hot wire-instruments?
Ans. The chief advantage of the hot-wire instruments is that the deflection depends upon the r.m.s.
value of current in the hot-wire whatever the waveform and frequency. Thus, a hot-wire
ammeter can be calibrated with d.c. but used for measuring alternating currents. The same
applies to a hot-wire voltmeter provided its series resistance (i.e. multiplier) is non-inductive.
Q. 21. Why are moving-iron and dynamometer instruments not suitable for high-frequency
applications?
Ans. At high frequencies, the reactance of the operating coil/coils may become so high that the
instrument does not draw any current. Consequently, readings cannot be taken.
Q. 22. All indicating instruments except permanent-magnet moving coil and dynamometer wattmeter
have squared scale. Why?
Ans. (i) Permanent-magnet moving coil instruments have linear scale as they incorporate a
permanent magnet having a constant field which reacts with the current passing through
482 Objective Electrical Technology
...........................................................................................................................................................................................................................................................................................................................................................

the moving coil and, therefore, the torque or motive power required to rotate the coil
varies directly with the value of this current.
(ii) A dynamometer wattmeter has two coils viz fixed coil and a moving coil. The fixed coil is
designed to carry the line current or a fraction of it while the moving coil carries current
proportional to load voltage. Since one flux is proportional to load current and the other
is proportional to load voltage, the torque on the pointer is directly proportional to load
power.
(iii) In all other types of indicating instruments, the scale is non-linear. It is because these
meters have no constant auxiliary source of energy, such as the permanent magnet of the
moving coil meter and, therefore, the torque at any instant is proportional to the energy
in the coil (I2Rt) i.e. it is proportional to the square of current in the coil. Hence, such
instruments have a squared scale; being crowded in the beginning and spread out at the
finishing end of the scale.
Q. 23. What is a multirange ammeter?
Ans. A multirange ammeter is constructed simply by employing several values of shunt resistors,
with a rotary switch to select the desired range. Fig. 16.40 (i) shows the circuit of a multirange
switch.
M

Rs1

B
C
Rs2 C B

D
Rs3 D A
E

Rs4 E A

(i) (ii)
Fig. 16.40
While using a multirange ammeter, care must be taken to ensure that shunt does not become
open-circuited, even for a brief instant. If the shunt is open-circuited even for a brief instant,
a large current will flow through the meter and will result in its destruction. For this reason,
make-before-break switch is used [See Fig. 16.40 (ii)]. The wide-ended moving contact
connects to the next terminal before it loses contact with the previous terminal. Thus during
switching time, there are two shunts in parallel and an open-circuited shunt is avoided.
Q. 24. What is a multirange voltmeter? R1
Ans. Fig. 16.41 shows the circuit of a
multirange voltmeter. Any one of the A
B
C
R2 rm
several multiplier resistors (R1, R2 and M
R3) is selected by means of a rotary D R 3
switch.
Unlike the case of the ammeter, the
rotary switch used with the voltmeter V
should be a break-before-make type i.e.
the moving contact should disconnect Fig. 16.41
from one terminal before connecting to the next terminal.
Electrical Measuring Instruments 483
............................................................................................................................................................................................................................................................................................................................................................

Q. 25. Consider two meters A and B of the same physical size that have full-scale deflection current
of 20 A. The resistance of meter A is 1500 and that of B is 200 . Which meter will be
costlier?
Ans. The full-scale deflection voltage of A = 20 A 1500 = 30 mV and that of B = 20 A
200 = 4 mV. The power input required for full deflection of A = 0.03 V 20 A = 0.6 W
and that for B = 0.004 V 20 A = 0.08 W. Meter B requires about one-eighth of power
needed to operate meter A. Accordingly meter B is more sensitive than meter A. Since the
operating power requirements are different, the construction of meter B is necessarily more
delicate than that of meter A. As a result, the cost of meter B is greater.
Q. 26. A potentiometer is equivalent to a voltmeter of infinite resistance. Discuss.
Ans. When the e.m.f. of a cell is measured by a potentiometer, then at null point, the current through
the cell is zero, i.e. the cell is on open circuit. Therefore, we get the actual value of e.m.f.
of the cell. If the above condition is to be realised with a voltmeter, it should have inifinite
resistance.
Q. 27. A potentiometer measures exact p.d. while voltmeter does not. Explain.
Ans. The potentiometer method is a null method. At null point, no current flows in the circuit
under measurement. Hence, potentiometer measures the exact p.d. On the other hand, when
a voltmeter is connected to measure p.d. across a component (say a resistor), a part of current
is drawn by the voltmeter. As a result, current through the component decreases. Hence, p.d.
being measured by the voltmeter is slightly less than the actual value.
Q. 28. Sometimes balance point is not obtained on potentiometer wire. Why?
Ans. It is because the p.d. under measurement is greater than the potential drop across the
potentiometer wire. In that case, the driver battery of large e.m.f. should be used.
Q. 29. For measuring p.d. by a voltmeter, deflection of the instrument is noted. When measuring
p.d. by a potentiometer, null point position on the potentiometer wire is noted. In which case
error of measurement will be less?
Ans. The error will be less in case of potentiometer. Let us explain this point. When p.d. is measured
by a voltmeter, there can be some error in reading the deflection. However, in reading the
null-point position in a potentiometer, there can be a maximum error of 1mm. Suppose the
potentiometer wire is 2m long and is connected to a cell of e.m.f. 2V. Then potential gradient
along the wire per mm will be 2/2000 = 0.001 V. Thus, the maximum error in measuring the
p.d. will be 0.001 V. This error can be further reduced by using a longer potentiometer wire.
Q. 30. Why should the cross-section of potentiometer wire be uniform?
Ans. If the cross-section of potentiometer wire is not uniform, then potential gradient will not be
same at all places on the potentiometer wire. Consequently, the measured value of potential
difference will not be correct.

Objective Questions
1. An ammeter is ............... instrument. (iii) increases (iv) none of the above
(i) an indicating (ii) an integrating 3. When the pointer of an indicating instru-
(iii) a recording (iv) none of the above ment comes to rest in the final deflected
2. The controlling torque of an indicating position, ...............
instrument ............... as the deflection of (i) only controlling torque acts
the moving system increases. (ii) only deflecting torque acts
(i) remains unchanged (iii) both deflecting and controlling torques
(ii) decreases act
484 Objective Electrical Technology
...........................................................................................................................................................................................................................................................................................................................................................

(iv) none of the above (iii) it is spring controlled


4. When the pointer of an indicating instru- (iv) it has no hysteresis loss
ment is in motion, then deflecting torque 11. Shunts are generally made of ...............
is opposed by .............. (i) copper (ii) aluminium
(i) controlling torque only (iii) silver (iv) manganin
(ii) damping torque only 12. The range of a permanent-magnet moving
(iii) both damping & controlling torques coil instrument is 0-10 A. If the full-scale
(iv) none of the above deflection current of the meter is 2 mA,
5. The pointer of an indicating instrument is then multiplying power of the shunt is
generally made of ............... ...............
(i) copper (ii) aluminium (i) 2500 (ii) 10000
(iii) silver (iv) soft steel (iii) 5000 (iv) none of the above
6. When the pointer of an indicating instru- 13. A moving coil instrument having meter
ment is in the final deflected position resistance of 5 is to be used as a volt-
,............... meter of range 0-100 V. If the full-scale
(i) deflecting torque is zero deflection current is 10 mA, then required
(ii) controlling torque is zero series resistance is ...............
(iii) damping torque is zero (i) 20 (ii) 1000
(iv) both deflecting & controlling torques (iii) 9995 (iv) none of the above
are zero 14. The multiplying power of the shunt of a
7. In eddy current damping, disc or former milliammeter is 8. If the circuit current is
is made of a material that is a ............... 200 mA, then current through the meter
(i) conductor but non-magnetic is ...............
(ii) conductor but magnetic (i) 200 mA (ii) 25 mA
(iii) non-conductor and non-magnetic (iii) 1600 mA (iv) none of the above
(iv) non-conductor but magnetic 15. The material of the shunt should have
8. In general, fluid friction damping is not ............... temperature co-efficient of re-
employed in indicating instruments al- sistance.
though one can find its use in ............... (i) negligible (ii) positive
(i) dynamometer wattmeter (iii) negative (iv) none of the above
(ii) hot-wire ammeter 16. A small swamping resistance is put in
(iii) induction type energy meter series with operating coil of a moving
(iv) Kelvin electrostatic voltmeter coil ammeter in order to compensate for
the effects of ...............
9. Permanent-magnet moving coil instrument
can be used for ............... (i) temperature variations
(i) a.c. work only (ii) d.c. work only (ii) external magnetic fields
(iii) both d.c. and a.c. work (iii) hysteresis loss (iv) none of the above
(iv) none of the above 17. A moving coil voltmeter gives full-scale
deflection of 100 V for a meter current of
10. The scale of a permanent-magnet mov-
1 mA. For 45 V reading, the meter current
ing coil instrument is uniform because
will be ...............
...............
(i) of effective eddy current damping (i) 0.45 mA (ii) 1.45 mA
(ii) external magnetic fields have no effect (iii) 2.22 mA (iv) none of the above
Electrical Measuring Instruments 485
............................................................................................................................................................................................................................................................................................................................................................

18. Dynamometer type instruments can be 27. The full-scale voltage across a moving
used for ............... coil voltmeter is about ...............
(i) a.c. work only (ii) d.c. work only (i) 10 V (ii) 5 V
(iii) both d.c. and a.c. work (iii) 100 V (iv) 50 mV
(iv) none of the above 28. Moving-iron instruments have ...............
19. A dynamometer instrument is chiefly used scale.
as a ............... (i) uniform (ii) squared
(i) d.c. ammeter (ii) d.c. voltmeter (iii) log (iv) none of the above
(iii) wattmeter (iv) none of the above 29. The range of a moving-iron a.c. ammeter
20. In a dynamometer type instrument, damp- is extended by ...............
ing is provided by ............... (i) a shunt (ii) a multiplier
(i) air friction (ii) eddy currents (iii) changing number of turns of operating
(iii) fluid friction (iv) none of the above coil
21. Dynamometer type ............... has uniform (iv) none of the above
scale. 30. To measure high-frequency currents, we
(i) ammeter (ii) wattmeter mostly use ............... ammeter.
(iii) voltmeter (iv) none of the above (i) hot-wire (ii) dynamometer
22. The instrument in which springs provide (iii) moving-iron (iv) thermocouple
the controlling torque as well as serve to 31. For the measurement of high direct volt-
lead current into and out of the operating age (say 10 kV), one would use ...............
coil is ............... instrument. voltmeter.
(i) moving-iron (ii) hot-wire (i) permanent-magnet moving coil
(iii) permanent-magnet moving coil (ii) electrostatic
(iv) none of the above (iii) hot-wire (iv) moving iron
23. If current through the operating coil of 32. ............... movement is most expensive.
a moving-iron instrument is doubled, the (i) DArsonval (ii) Moving-iron
operating force becomes ............... (iii) Dynamometer
(i) two times (ii) four times (iv) None of the above
(iii) one-half time (iv) three times 33. Electrostatic instruments are used as
24. The full-scale deflection current of a mov- ............
ing coil instrument is about ............... (i) voltmeters only (ii) ammeters only
(i) 50 mA (ii) 1 A (iii) both ammeters and voltmeters
(iii) 3 A (iv) 2 A (iv) wattmeters only
25. For measuring high values of alternating 34. An electric pyrometer is an instrument
current with a dynamometer ammeter, we used to measure ...............
use a ...............
(i) phase (ii) frequency
(i) shunt (ii) multiplier
(iii) high temperatures
(iii) potential transformer
(iv) none of the above
(iv) current transformer
35. The best type of meter movement is
26. Hot-wire instruments have ............... scale. ............... movement.
(i) uniform (ii) log (i) iron-vane (ii) DArsonval
(iii) squared (iv) none of the above (iii) dynamometer (iv) none of the above
486 Objective Electrical Technology
...........................................................................................................................................................................................................................................................................................................................................................

36. ...............instruments are most sensitive. (i) 0 A (ii) 5 A


(i) Moving-iron (ii) Hot-wire (iii) 2.5 A (iv) none of above
(iii) Dynamometer 46. In the above question, if the meter remains
(iv) Permanent-magnet moving coil connected in the circuit for some time,
37. In induction type ammeter, ............... (i) meter pointer gives full-scale deflection
damping is provided. (ii) meter pointer starts oscillating
(i) air friction (ii) eddy current (iii) meter coil is burnt
(iii) fluid friction (iv) none of the above (iv) none of above
38. The most commonly used induction type 47. If in question 45, the frequency of a.c. is
instrument is ............... 0.1 Hz, the pointer will
(i) induction voltmeter (i) rise from zero and then fall back
(ii) induction wattmeter (ii) read zero
(iii) induction watt-hour meter (iii) read full-scale (iv) none of the above
(iv) induction ammeter 48. On a simple ohmmeter, the infinity
39. All voltmeters except ............... voltmeters mark is ............... of the scale.
are operated by the passage of current. (i) far left (ii) far right
(i) moving-iron (ii) electrostatic (iii) in the middle (iv) none of above
(iii) dynamometer 49. The instrument used in an ohmmeter is
(iv) permanent-magnet moving coil generally
40. The watt-hour meter is ............... instru- (i) moving-iron type
ment. (ii) hot-wire type
(i) an integrating (ii) an indicating (iii) permanent magnetic moving coil type
(iii) a recording (iv) a transfer (iv) dynamometer type
41. Indicating instruments are assumed to be 50. When the terminals of a series ohmmeter
most accurate ............... part of scale. are open-circuited, the pointer reads
(i) at beginnig (ii) at finishing (i) zero (ii) infinity
(iii) at half of full (iv) none of the above (iii) a high resistance
42. ............... meter will be the most sensitive. (iv) none of the above
(i) 50 mA (ii) 100 A 51. Out of the following, the most accurate
(iii) 50 A (iv) 1 A measurement of unknown resistance will
43. On a simple ohmmeter, the 0 mark is be by
............... of the scale. (i) potentiometer (ii) ohmmeter
(i) far left (ii) far right (iii) voltmeter and ammeter
(iii) in the middle (iv) none of the above (iv) Wheatstone bridge
44. If a wattmeter connected in circuit gives 52. A 50 V range voltmeter has a sensitivity
down scale reading, then we normally of 20 k/V. The total resistance of the
change connections of ........... voltmeter is
(i) current coil (ii) potential coil (i) 2.5 k (ii) 0.4 k
(iii) both current and potential coils (iii) 10 k (iv) 1 M
(iv) none of above 53. Fig. 16.42 shows voltmeter-ammeter
45. A permanent magnet moving coil ammeter method of measuring the value of resis-
is connected in 50 Hz a.c. circuit in which tance R. This method is used to measure
5A current is flowing. The meter will read R if its value is
Electrical Measuring Instruments 487
............................................................................................................................................................................................................................................................................................................................................................

(i) 20 k (ii) 10 k
(iii) 30 k (iv) 40 k
Rv V R 57. In Fig. 16.44, the resistor r is for
(i) zero adjustment (ii) meter protection
A
(iii) battery protection
Ra
(iv) none of above
Fig. 16.42 58. The scale of an ohmmeter is
(i) very low (i) linear (ii) non-linear
(ii) very high and moderate (iii) linear and non-linear
(iii) cannot say (iv) none of above (iv) none of above
54. Fig. 16.43 shows voltmeter-ammeter 59. Fig. 16.45 shows a permanent magnet
method of measuring the value of resis- moving coil (PMMC) voltmeter having
tance R. This method is used to measure range of 100 V. The coil resistance is
R if its value is 100 and full-scale deflection current
is 50 A. What is the value of voltmeter
sensitivity?
Rv V R R G = 100 
M
A
Ra Ig = 50 A

Fig. 16.43
V
(i) very low
Fig. 16.45
(ii) high and moderate
(i) 10 k/V (ii) 30 k/V
(iii) cannot say (iv) none of above
(iii) 40 k/V (iv) 20 k/V
55. The series ohmmeter shown in Fig. 16.44 is
made up of a 3 V battery, a 100 A meter 60. In the above question, what is the voltmeter
and a resistance r which has a fixed value sensitivity if the range is 50 V?
of 30 k. What is the value of unknown (i) 10 k/V (ii) 30 k/V
resistance R, when the pointer indicates (iii) 20 k/V (iv) 40 k/V
half full-scale deflection? 61. In the circuit shown in Fig. 16.46, what is
the voltage across R2 without the voltmeter
in the circuit?
I2 + Iv

R1 100 k
Iv
E = 100 V A
I2
Range = 2 V
R2 1k V Sensitivity
Fig. 16.44
= 1 k / V
(i) 20 k (ii) 60 k
B
(iii) 30 k (iv) 40 k
Fig. 16.46
56. In the above question, what is the value
of R if pointer indicates 3/4th full-scale (i) 1.2 V (ii) 0.77 V
deflection? (iii) 0.36 V (iv) 0.99 V
488 Objective Electrical Technology
...........................................................................................................................................................................................................................................................................................................................................................

62. In the above question, what is the voltage (iii) more than actual load power
across R2 with voltmeter connected in the (iv) none of the above
circuit?
(i) 0.66 V (ii) 0.36 V
(iii) 0.75 V (iv) 0.84 V A

63. In the circuit shown in Fig. 16.47, what



is the value of unknown resistor R? The WM

voltmeter reads 4 V. Line V Load

R 10 k
V
4V
I Fig. 16.48
120 V 69. In Fig. 16.48, the ammeter reading will
be
(i) equal to the load current
Fig. 16.47
(ii) more than the load current
(i) 110 k (ii) 290 k
(iii) less than the load current
(iii) 134 k (iv) 245 k
(iv) cannot be predicted
64. In the above question, a voltmeter is used
70. A dynamometer wattmeter with its voltage
instead of ammeter. It is because
coil connected across the load side reads
(i) voltmeter has high resistance 192 W (See Fig. 16.49). The load voltage
(ii) voltmeter sensitivity is low is 208 V and the resistance of potential coil
(iii) the value of R is very large is 3825 . What is the true load power?
(iv) none of the above Wattmeter
65. Resistance of an ammeter having range
0-5A is 1.8 . It is shunted by a resistor
of 0.2 . What is the effective current
when the pointer reads 2 A?
(i) 10 A (ii) 30 A
208 V
(iii) 15 A (iv) 20 A 3825 
66. In a dynamometer wattmeter, the moving
coil is the
(i) current coil (ii) potential coil
(iii) current coil or potential coil Fig. 16.49
(iv) none of the above (i) 125.8 W (ii) 180.7 W
67. Fig. 16.48 shows an ammeter, a voltmeter (iii) 224.6 W (iv) 352.8 W
and a wattmeter connected in the circuit. 71. In the above question, what is the percent-
The wattmeter will read age error due to wattmeter connection?
(i) upscale (ii) down scale (i) 6.25% (ii) 4.85%
(iii) data insufficient (iv) none of above (iii) 11.34% (iv) 7.86%
68. In the circuit shown in Fig. 16.48, the 72. In Fig. 16.50, the resistances of the two
wattmeter reading will be coils of wattmeter are 0.01 and 1000
(i) equal to actual load power respectively and both are non-inductive.
(ii) less than actual load power The load is taking a current of 20 A at
Electrical Measuring Instruments 489
............................................................................................................................................................................................................................................................................................................................................................

200 V and 0.8 p.f. lagging. What is the 77. To measure a.c. as well as d.c. power, we
reading of wattmeter? use
(i) induction wattmeter
0.01  I = 20 A (ii) dynamometer wattmeter
(iii) sometimes induction and sometimes
dynamometer wattmeter
(iv) none of the above
200 V
1000  78. In a single phase energy meter, braking
torque is provided by
(i) permanent magnet
(ii) air friction
Fig. 16.50 (iii) fluid friction (iv) none of the above
(i) 1602 W (ii) 6408 W 79. The meter constant of an energy meter
(iii) 1122 W (iv) 3204 W is 1500 rev/kWh, the disc makes 3000
73. In the above question, what is the revolutions in a given time. The energy
percentage error in the reading of the consumed is
wattmeter? (i) 4 kWh (ii) 1 kWh
(i) 0.5% (ii) 1.25% (iii) 3 kWh (iv) 2 kWh
(iii) 0.125% (iv) 0.25% 80. A 230 V, 50 Hz single-phase energy meter
74. Fig. 16.51 shows another way of connect- has a load current of 10 A at p.f. of 0.8
ing the wattmeter in the circuit. If the load lagging. The energy consumed by the load
p.f. is 0.8 lagging, what is the reading of in 2 minutes is
the wattmeter? (i) 1.2 kWh (ii) 0.06 kWh
(iii) 2.4 kWh (iv) 4.2 kWh
0.01  I = 20 A 81. In the above question, if the disc makes
72 revolutions per minute, what is the
meter constant?
(i) 600 rev/kWh (ii) 900 rev/kWh
200 V
1000  (iii) 1200 rev/kWh (iv) none of the above
82. In a single phase energy meter, if the brake
magnet is moved towards the centre of the
spindle, the disc speed
Fig. 16.51 (i) increases (ii) decreases
(i) 1620 W (ii) 3240 W (iii) remains same (iv) none of the above
(iii) 1976 W (iv) 2382 W 83. An energy meter whose meter constant is
75. In the above question, what is the percent- 1500 rev/kWh makes 20 revolutions in 30
age error in the wattmeter reading? seconds. The load in kW is
(i) 0.125% (ii) 2.52% (i) 3.2 kW (ii) 0.8 kW
(iii) 1.25% (iv) 2.65% (iii) 6.4 kW (iv) 1.6 kW
76. Induction wattmeter can measure 84. A 230 V single phase energy meter has
(i) a.c. power only a constant load current of 4 A passing
(ii) d.c. power only through if for 5 hours at unity power fac-
(iii) both a.c. and d.c. power tor. If the meter makes 1104 revolutions
(iv) none of the above during this period, the meter constant is
490 Objective Electrical Technology
...........................................................................................................................................................................................................................................................................................................................................................

(i) 480 rev/kWh (ii) 240 rev/kWh (i) 0.4 (ii) 0.2
(iii) 320 rev/kWh (iv) 960 rev/kWh (iii) 0.1 (iv) 0.3
85. In the above question, if the load p.f. is 92. In a potentiometer experiment, it is found
0.8, the number of revolutions the disc will that no current flows through the galva-
make in above time (5 hours) is about nometer when the terminals of the cell are
(i) 464 (ii) 572 connected across 125 cm potentiometer
(iii) 883 (iv) 774 wire. On shunting the cell by 2 resistor,
the balancing length is reduced to half.
86. The full-scale deflection current of a
The internal resistance of the cell is
permanent magnet moving-coil (PMMC)
meter is 1 mA and the coil resistance is (i) 3 (ii) 6
50. The least voltage that can be mea- (iii) 1 (iv) 2
sured with this meter is 93. A moving coil galvanometer has a sensitiv-
(i) 50 mV (ii) 25 mV ity of 60 divisions/amp. When a shunt is
used, its sensitivity becomes 10 div/amp.
(iii) 100 mV (iv) none of the above
What is the value of the shunt used if the
87. If the full-scale current of a meter is resistance of the galvanometer is 20 ?
50 A, then its sensitivity is
(i) 2 (ii) 6
(i) 1000 /V (ii) 20,000 /V (iii) 4 (iv) 1
(iii) 10,000 /V (iv) data insufficient 94. Which of the following is likely to have
88. A 100 V, full-scale, 100 /volt meter has the largest resistance?
a full-scale deflection current of (i) moving coil galvanometer
(i) 0.5 mA (ii) 2 mA (ii) voltmeter of range 10 V
(iii) 3.5 mA (iv) 1 mA (iii) ammeter of range 1 A
89. With a potentiometer, null points are ob- (iv) a copper wire of length 1m and diam-
tained at 140 cm and 180 cm with cells eter 3 mm
of e.m.f. 1.1 V and one of the unknown 95. If 2% of the main current is to be passed
e.m.f. respectively. The unknown e.m.f. is through a moving coil galvanometer of
(i) 1.1 V (ii) 1.8 V resistance G, the resistance of the shunt
(iii) 1.41 V (iv) 1.6 V required is
90. In a potentiometer experiment, it is found (i) G/49 (ii) G/50
that no current flows through the galva- (iii) 49 G (iv) 50 G
nometer when the terminals of the cell are 96. A galvanometer has a resistance of G ohms.
connected across 52 cm of potentiometer It is shunted by a resistance of S ohms.
wire. If the cell is shunted by a resistance How much resistance should be added so
of 5 , the balance point is found at 40 that the main current remains unchanged?
cm of the wire from the same end. The S G
internal resistance of the cell is (i) (ii)
S +G G+S
(i) 1.5 (ii) 2.5 SG G2
(iii) (iv)
(iii) 3 (iv) 4.5 G+S G+S
91. A potentiometer wire is 10 m long. It has 97. In a moving coil galvanometer, the
a resistance of 20 . It is connected in deflection becomes one-half when the
series with a battery of e.m.f. 3 V and galvanometer is shunted by a 20 resistor.
negligible resistance and a resistance of The galvanometer resistance is
10 . The potential gradient along the
(i) 10 (ii) 40
wire in volt/meter is
(iii) 20 (iv) 5
Electrical Measuring Instruments 491
............................................................................................................................................................................................................................................................................................................................................................

98. It is required to convert a moving coil 104. To send 10% of main current through a
galvanometer of current range 15 mA and moving coil galvanometer of resistance
voltage range 750 mV into an ammeter of 99 , the value of shunt resistance
range 2.5 A. What is the value of shunt required is
resistance required? (i) 33 (ii) 9.9
(i) 0.3 (ii) 1.5 (iii) 22 (iv) 11
(iii) 3.4 (iv) 0.5 105. The sensitivity of a moving coil galva-
99. A moving coil galvanometer of resistance nometer is 60 divisions/ampere. When
25 is connected to a source of e.m.f. a shunt is used, the sensitivity becomes
2V with a resistance of 3 k. Full-scale 10 divisions/ampere. If the galvanometer
deflection of 30 units is obtained. When resistance is 20 , the value of shunt is
3 k resistance is replaced by R, the de- (i) 15 (ii) 20
flection becomes 20 units. The value of (iii) 4 (iv) 5
R is approximately
106. A 20 A, 200 mV meter movement is
(i) 2.3 k (ii) 4.2 k used to make a 25 V voltmeter. What is
(iii) 3.8 k (iv) 4.5 k the sensitivity of the meter?
100. A moving coil galvanometer with a scale (i) 25 k/V (ii) 50 k/V
divided into equal divisions has a current (iii) 100 k/V (iv) 75 k/V
sensitivity of 10 divisions per mA and
107. In the above question, what is the value
voltage sensitivity of 50 divisions per mV.
of multiplier resistance?
To convert it into an ammeter of range 5
(i) 620 k (ii) 1240 k
A, the necessary shunt resistance required
is (iii) 830 k (iv) none of the above
(i) 0.1 (ii) 5/499 108. In question 106, what is the total resistance
of the voltmeter?
(iii) 0.03 (iv) 3/283
(i) 1250 k (ii) 625 k
101. Deflection of a moving coil galvanometer
falls from 50 divisions to 20 divisions (iii) 2500 k (iv) 150 k
when a 12 shunt is applied. Galvanom- 109. A series ohmmeter circuit uses a 3 V bat-
eter resistance is tery and 1 mA meter movement. What is
(i) 18 (ii) 6 the half scale resistance for this ohmmeter?
(iii) 9 (iv) 24 (i) 6 k (ii) 1.5 k
102. A voltmeter of resistance 998 is con- (iii) 12 k (iv) 3 k
nected in series with a cell of e.m.f. 2 V 110. A 10 V range voltmeter is rated for 50 A
and internal resistance 2 . The percentage full-scale current. The total resistance of
error in the reading of voltmeter is the voltmeter is
(i) 1% (ii) 0.2% (i) 100 k (ii) 200 k
(iii) 2% (iv) 0.1% (iii) 300 k (iv) 400 k
103. In an ammeter, 5 percent of main cur- 111. The resolution of an instrument is the
rent passes through the galvanometer. If (i) minimum quantity it can measure
resistances of the galvanometer and shunt (ii) maximum quantity it can measure
are G and S respectively, then, (iii) maximum non-linearity
G 1 (iv) none of the above
(i) S = (ii) S G =
19 19
112. The torque/weight ratio of an instrument
1 S indicates
(iii) G S = (iv) G =
19 19
492 Objective Electrical Technology
...........................................................................................................................................................................................................................................................................................................................................................

(i) selectivity (ii) accuracy The value of R1 is


(iii) sensitivity (iv) fidelity R4 R3 R2 R1

113. A current i = (10 + 10 sin t) A is passed


through an ideal moving iron type am- + 300 V 100 V 30 V 10 V
meter. Its reading will be Meter

(i) zero (ii) 10 A


(iii) 2 A (iv) 150 A Fig. 16.52
114. A high frequency a.c. signal is applied to (i) 9.9 kW (ii) 10 kW
a PMMC instrument. If the r.m.s. value
(iii) 100 W (iv) 0.0 W
of a.c. signal is 2 V, the reading of the
instrument will be 120. Which of the following instrument/instru-
(i) 2 V (ii) zero ments has/have linear scale?
(iii) 2 2 V (iv) 4 2 V (i) moving iron (ii) PMMC
115. Which of the following instrument is free (iii) induction meter (iv) electrostatic
from hysteresis and eddy current errors? 121. For the voltmeter circuit shown in
(i) Moving iron instrument Fig. 16.53, the basic DArsonval move-
(ii) PMMC instrument ment meter used has full-scale current of
(iii) Electrostaic instrument 1 mA and meter resistance Rm = 100 W.
The values of series resistances R1 and
(iv) Dynamometer instrument
R2 required for 10 V range and 50 V
116. What is the correct sequence of the fol-
range will be
lowing types of ammeters and voltmeters
R2 R1
with increasing accuracy?
(a) moving iron (b) PMMC meter
(c) induction instrument.
Rm
(i) c, a, b (ii) a, c, b
Unknown
(iii) b, a, c (iv) a, b, c Voltage
117. A moving coil instrument gives full-scale
deflection for 1 mA and has a resistance Fig. 16.53
of 5 W. If a resistance of 0.55 W is con-
(i) 10 kW and 50 kW
nected in parallel with the instrument, what
is the maximum current it can measure? (ii) 20 kW and 30 kW
(i) 5 mA (ii) 50 mA (iii) 200 kW and 250 kW
(iii) 100 mA (iv) 10 mA (iv) 9.9 kW and 40 kW
118. A DArsonval movement with internal 122. An energy meter having a meter constant
resistance R = 100 W and full-scale cur- of 1200 rev. per kWh is found to make 5
rent of 1 mA is to be converted into 010 revolutions in 75 seconds. The load power
V range voltmeter. What is the required is
resistance? (i) 500 W (ii) 100 W
(i) 10 kW (ii) 9900 W (iii) 200 W (iv) 1000 W
(iii) 10,100 W (iv) 12000 W 123. A spring-controlled moving iron voltmeter
119. A basic DArsonval movement with inter- draws a current of 1 mA for full-scale
nal resistance 100 W and full-scale current value of 100 V. If it draws a current of
of 1 mA is to be converted into a multi 0.5 mA, the meter reading is
range d.c. voltmeter with ranges 010 V, (i) 25 V (ii) 50 V
030 V, 0100 V and 0300 V. The cir-
(iii) 100 V (iv) 200 V
cuit arrangement is shown in Fig. 16.52.
Electrical Measuring Instruments 493
............................................................................................................................................................................................................................................................................................................................................................

124. Which one of the following decides the 125. A galvanometer with a full-scale current
time response of an indicating instrument? of 10 mA has a resistance of 1000 W. The
(i) deflecting system multiplying power of a 100 W shunt with
(ii) damping system this galvanometer is
(iii) controlling system (i) 110 (ii) 10
(iv) pivot and jewel bearings (iii) 100 (iv) 11

Answers to Objective Questions


1. (i) 2. (iii) 3. (iii) 4. (iii) 5. (ii)
6. (iii) 7. (i) 8. (iv) 9. (ii) 10. (iii)
11. (iv) 12. (iii) 13. (iii) 14. (ii) 15. (i)
16. (i) 17. (i) 18. (iii) 19. (iii) 20. (i)
21. (ii) 22. (iii) 23. (ii) 24. (i) 25. (iv)
26. (iii) 27. (iv) 28. (ii) 29. (iii) 30. (iv)
31. (ii) 32. (iii) 33. (i) 34. (iii) 35. (ii)
36. (iv) 37. (ii) 38. (iii) 39. (ii) 40. (i)
41. (iii) 42. (iv) 43. (ii) 44. (ii) 45. (i)
46. (iii) 47. (i) 48. (i) 49. (iii) 50. (ii)
51. (iv) 52. (iv) 53. (ii) 54. (i) 55. (iii)
56. (ii) 57. (i) 58. (ii) 59. (iv) 60. (iii)
61. (iv) 62. (i) 63. (ii) 64. (iii) 65. (iv)
66. (ii) 67. (i) 68. (iii) 69. (ii) 70. (ii)
71. (i) 72. (iv) 73. (iii) 74. (ii) 75. (iii)
76. (i) 77. (ii) 78. (i) 79. (iv) 80. (ii)
81. (iii) 82. (i) 83. (iv) 84. (ii) 85. (iii)
86. (i) 87. (ii) 88. (iv) 89. (iii) 90. (i)
91. (ii) 92. (iv) 93. (iii) 94. (ii) 95. (i)
96. (iv) 97. (iii) 98. (i) 99. (iv) 100. (ii)
101. (i) 102. (ii) 103. (i) 104. (iv) 105. (iii)
106. (ii) 107. (ii) 108. (i) 109. (iv) 110. (ii)
111. (i) 112. (iii) 113. (iv) 114. (ii) 115. (iii)
116. (i) 117. (iv) 118. (ii) 119. (i) 120. (ii)
121. (iv) 122. (iii) 123. (i) 124. (ii) 125. (iv)

Hints to Selected Objective Questions


5. Because aluminium is light and does not increase the weight of the moving system.
6. Damping torque acts only when the pointer is in motion and always opposes motion.
8. Most of the indicating instruments are portable. Fluid friction damping is not suitable
because of the oil contained in the instrument.
10. In a permanent-magnet moving coil instrument, Td I and TC .
In the final deflected position, Td = TC.
I
494 Objective Electrical Technology
...........................................................................................................................................................................................................................................................................................................................................................

Hence such instruments have uniform scale.


11. Since manganin has negligible temperature co-efficient of resistance.
I 10 A
12. Multiplying power of shunt = = = 5000
Im 2 mA
where I is the circuit current and Im is the meter current. In the present case, I is taken
equal to 10 A so that Im will be the full-scale meter current.
V 100 V
13. Series resistance, Rs = Rm = 5 = 10000 5 = 9995
Im 10 mA
17. For full scale deflection (i.e. 100 V), the meter current is 1 mA. For 45 V reading, the
meter current will be = (1/100) 45 = 0.45 mA.
23. In a moving-iron instrument, the deflecting torque is
Td I2 ----- for d.c.
I2r.m.s. ----- for a.c.
If current is doubled, operating torque is increased four times.
25. For measuring alternating currents larger than 5 A, shunts are not practicable with a
dynamometer ammeter. It is because the division of current between the shunt and the
coils varies with frequency (since reactance of coils depends upon frequency). Therefore,
the instrument will be accurate only at the frequency at which it is calibrated. Instead,
we use current transformer.
35. Because it is very light and accurate. Moreover, the sensitivity of the instrument can
be greatly increased by increasing the number of turns of the operating coil.
38. The single phase energy meter used in homes is the induction watt-hour meter.
44. It is easier to change connections of potential coil than current coil connections.
45. At 50 Hz, the meter pointer would have to rise and fall 50 times in every second. The
inertia of the moving system prevents the moving coil from moving this fast. The meter
pointer settles at the average level of alternating current. The average value of alternating
current is zero.
46. The meter coil is burnt due to excessive heat.
51. It is because Wheatstone bridge method is independent of the constancy of supply or
the calibration of an indicating instrument.
52. The voltmeter sensitivity is an important constant and is usually printed on the face of
all voltmeters. To find the total voltmeter resistance, the resistance per volt (i.e. sensitiv-
ity) is multiplied by the voltmeter range.
Total voltmeter resistance = (20 k/V) (50 V) = 1000 k = 1 M
53. Let voltmeter read V and ammeter read I. Then,
V = I Ra + I RorR = V/I Ra
Since R is high, the value of Ra can be neglected. Therefore, R = V/I.
54. Let voltmeter read V and ammeter read I. Then ammeter current is equal to sum of
current through voltmeter and current through R.
V V
I = +
R RV
Since R is very low as compared to RV, V/RV can be neglected.
I = V/RorR = V/I
Electrical Measuring Instruments 495
............................................................................................................................................................................................................................................................................................................................................................

55. When R = 0, the pointer deflects to full-scale and meter current is 100 A.
At half full-scale deflection, Im = 100/2 = 50 A
E 3V
Now, R + r = = = 60 k
I m 50 A
R = 60 r = 60 30 = 30 k
59. The total resistance per volt of the voltmeter is called voltmeter sensitivity. In the given
question,
V 100 V
Total resistance of voltmeter = = = 2 M
I g 50 A
2 M 2000 k

Voltmeter sensitivity = = = 20 k/V
100 V 100 V
V 50 V
60. Total resistance of voltmeter = = = 1 M
I g 50 A
1 M 1000 k

Voltmeter sensitivity = = = 20 k/V
50 V 50 V
Note that for a given voltmeter, voltmeter sensitivity is constant.
61. Voltage across R2 without voltmeter
E 100 V
= R2 = 1 k = 0.99 V
R1 + R2 100 k + 1 k
62. Voltmeter resistance, RV = range sensitivity = 2 V 1 k/V = 2 k
Resistance between A and B, RAB = R2 | | RV = 1 k || 2 k = 666.7
E 100 V
Voltage across R2 = RAB = 666.7 = 0.66 V
R1 + RAB 100 103 + 666.7
Note that voltmeter loading effect can cause error in measurement. For this reason, a
voltmeter should have very high resistance so that the current it draws is extermely
small.
4
63. Circuit current, I = 3
= 4 104 V
10 10
Supply voltage = I (R + RV)
or 120 = 4 104 (R + 10 103)
R = 290 103 = 290 k
64. Since the value of R is very large (290 k), the circuit current is very small (4 104 A).
If ammeter is connected, the ammeter reading will be too small to be measured. This
is an unusual use of a voltmeter to measure the value of a high resistance.
G+S G 1.8
65. Multiplying power of shunt, n = = +1= + 1 = 10
S S 0.2
Therefore, the range of ammeter increases 10 times. When the ammeter indicates 2 A,
the circuit current = 2 10 = 20 A.
67. Fig. 16.48 shows the correct connections of a wattmeter to measure the load power.
Current comes into the terminal of the current coil from the line. The terminal of
the potential coil is connected to the load side of the current coil. With this connection,
the wattmeter reads upscale.
496 Objective Electrical Technology
...........................................................................................................................................................................................................................................................................................................................................................

68. The wattmeter reads high by the amount of power consumed by the voltmeter and by
the potential coil of the wattmeter.
69. The ammeter reads high by the amount of current taken by the voltmeter and by the
potential coil of the wattmeter.
70. Since the voltage coil of the wattmeter is connected on the load side, the power con-
sumed by it is also included in the reading of the wattmeter.
Wattmeter reading = 192 W

Power consumed in potential circuit = = 11.3 W


( 208)2
3825
True load power = 192 11.3 = 180.7 W
192 180.7
71. Percentage error = 100 = 6.25%
180.7
72. Load power = V I cos = 200 20 0.8 = 3200 W
Power loss in current coil = I2 RC = (20)2 0.01 = 4 W
Wattmeter reading = 3200 + 4 = 3204 W.
4
73. Percentage error = 100 = 0.125%
3200
74. Load power = V I cos = 200 20 0.8 = 3200 W
2
V 2 ( 200 )
Power loss in voltage coil = = = 40 W
Rp 1000
Wattmeter reading = 3200 + 40 = 3240 W
40
75. Percentage error = 100 = 1.25%
3200
77. Induction wattmeters have low accuracy and high power consumption. Therefore, they
are inferior to dynamometer wattmeter. For this reason, dynamometer wattmeters are
almost universally used for the measurement of a.c. as well as d.c. power.
80. Energy consumed by the load in 2 minutes
V I cos 230 10 0.8 2
= t = = 0.06 kWh
1000 1000 60
No. of rev. 72
81. Meter constant = = = 1200 rev/kWh
Energy consumed 0.06
82. Moving the brake magnet inwards means that speed of the part of the disc under the
pole face of brake magnet will be less. This results in the lesser induced voltage and
hence the reduced braking torque. Consequently, the disc speed increases.
83. Energy consumed when disc makes 20 revolutions
1 1
= 20 = kWh
1500 75
Now energy consumed is equal to load in kW multiplied by time in hours i.e.
3 1
Load =
3600 75
1 3600
Load = = 1.6 kW
75 30
Electrical Measuring Instruments 497
............................................................................................................................................................................................................................................................................................................................................................

VI cos 230 4 1
84. Energy supplied = t = 5 = 4.6 kWh
1000 1000
No. of revolutions 1104
Meter constant = = = 240 rev/kWh
Energy supplied 46
85. Energy supplied when load p.f. is 0.8
V I cos 230 4 0.8
= t = 5 = 3.68 kWh
1000 1000
No. of revolutions = meter constant energy supplied
= 240 3.68 = 883 revolutions.
86. The least voltage that a movement can measure is the full-scale current multiplied by
coil resistance. Least voltage that can be measured = 1 mA 50 = 50 mV.
87. We can find the sensitivity if 1 V is divided by full-scale current of the meter.
1V 1V 1
Sensitivity = = = M/V = 20,000 /V
f.s.d. current 50 A 50
1V
88. Sensitivity =
f.s.d. current
1V 1V 1

f.s.d. current = = = A = 1 mA
Sensitivity 1000 1000
l 52
90. Internal resistance, r = 1 1 R = 1 5 = 1.5
l2 40
91. Total circuit resistance, R = 20 + 10 = 30
Current in potentiometer wire, I = E/R = 3/30 = 1/10 A
1
P.D. across the wire = 20 = 2 V
10
Potential gradient = 2/10 = 0.2 V/m
l
92. Internal resistance of cell, r = 1 1 R = (2 1) 2 = 2
l2
Ig 10 1
93. = = ; G = 20
I 60 6
Ig S 1 S
Now = or = S = 4
I S +G 6 S + 20
GS
96. Original resistance = G ; New resistance =
G+S
Resistance to be added in series
GS G2 + G S G S G2
= G = =
G+S G+S G+S
97. Fig. 16.54 shows the conditions of the problem. S

I I
20 = G G = 20 I
2 2 2
750 mV G
98. Resistance of galvanometer, G = = 50 I I
15 mA 2
Fig. 16.54
498 Objective Electrical Technology
...........................................................................................................................................................................................................................................................................................................................................................

Value of shunt resistance, S =


Ig G
=
15 103 50
= 0.3
( )
I Ig 2.5 15 103
2 2
99. 30 = and 20 =
3000 + 25 R + 25
On solving, R = 4512 or nearly 4.5 k.
50 103
100. Resistance of galvanometer, G = =5
10 103
Ig G
Required value of shunt, S =
I Ig
Here Ig = 10 mA ; I = 5 A = 5000 mA ; G = 5
10 5 50 5
S = = =
5000 10 4990 499
102. The total circuit resistance = 998 + 2 = 1000 . Therefore, circuit current, I = 2/1000
= 0.002 A and voltage drop in the cell = Ir = 0.002 2 = 0.004 V.
0.004
Error = 0.004 V and % error = 100 = 0.2%
2
103. Fig. 16.55 shows the conditions of the problem. Let S

the main current be I. Then as per given conditions,


0.95 I
0.95 I flows through the shunt and the remaining
0.05 I passes through the galvanometer. As voltages G
across shunt and galvanometer are equal, I Ig = 0.05 I

S 1 Fig. 16.55
S 0.95 I = G 0.05 Ior =
G 19
Ig S I g 10 1
105. = Here = = ; G = 20
I S +G I 60 6
1 S
= orS = 4
6 S + 20

1 1 106
106. Sensitivity = = = /V = 50 k/V
I g 20 106 20
6
1 1 1 10
110. Sensitivity = = = /V = 20 k/V
I g 50 A 50
Total resistance of voltmeter = 20 10 = 200 kW
111. The resolution of an instrument is the smallest change in the input signal (quantity
under measurement) which can be detected by the instrument.
113. A moving iron ammeter reads r.m.s. value of current.
2
10
Ir.m.s. = 102 + = 150 A
2
114. Since the deflecting torque of PMMC instrument reverses with the reversal of current
in the coil, pointer fails to respond to rapid reversal of torque due to its high inertia.
Hence the instrument will read zero i.e. PMMC instrument will not work.
Electrical Measuring Instruments 499
............................................................................................................................................................................................................................................................................................................................................................

115. Since no iron is used in the construction of electrostatic instruments, they are free
from hysteresis and eddy current losses.
117. G = 5 W ; S = 0.55 W ; Ig = 1 mA ; I = ?
(I Ig)S = Ig Gor(I 1) 0.55 = 1 5 \ I = 10 mA
118. V = 10 volts ; Ig = 1 mA ; R = 100 W ; RS = ?
V 10
Required series resistance, RS = R= 100 = 9900 W
Ig 1 103
119. V = 10 volts ; Ig = 1 mA ; G = 100 W
V 10 V
\ R = G = 100 = 9900 W = 9.9 kW
Ig 1 mA
10 V 10 V
121. For 10 V range, R1 = Rm = 100 = 9900 W = 9.9 kW
Ig 1 mA
50 50 V
For 50 V range, R1 + R2 = Rm = 100 = 49900 W = 49.9 kW
Ig 1 mA

\ R2 = (R1 + R2) R1 = 49.9 9.9 = 40 kW
1 1
122. 1 rev. corresponds to kWh = 1000 3600 watt-sec = 3000 watt-sec
1200 1200
5 revolutions correspond to 3000 5 = 15,000 watt-sec
Load power = 15,000/75 = 200 W
123. For moving iron voltmeter, deflection q V2.
For the first case, 100 12 ; For the second case, V 0.52
V 0.52

\ = 2 = 0.25 \ V = 0.25 100 = 25 V
100 1
125. With shunt S = 100 W, suppose the maximum current that can be measured is I mA.
Now, S = 100 W ; G = 1000 W ; Ig = 10 mA ; I = ?
Now, (I Ig) S = Ig Gor(I 10)100 = 10 1000 \ I = 110 mA
I 110 mA
\ Multiplying power of shunt = = = 11
I g 10 mA
www.JobsCare.info
20
554  Principles of Electronics

Silicon
Controlled Rectifiers
20.1 Silicon Controlled Rectifier (SCR)
20.2 Working of SCR
20.3 Equivalent Circuit of SCR
20.4 Important Terms
20.5 V-I Characteristics of SCR
20.6 SCR in Normal Operation
20.7 SCR as a Switch
20.8 SCR Switching
20.9 SCR Half-Wave Rectifier
20.10 SCR Full-Wave Rectifier
20.11 Single-Phase SCR Inverter Circuit
20.12 Applications of SCR
20.13 Light-Activated SCR

INTRODUCTION

T
he silicon controlled rectifier (abbreviated as SCR) is a three-terminal semiconductor switching
device which is probably the most important circuit element after the diode and the transis
tor. Invented in 1957, an SCR can be used as a controlled switch to perform various functions
such as rectification, inversion and regulation of power flow. The SCR has assumed paramount
importance in electronics because it can be produced in versions to handle currents upto several
thousand amperes and voltages upto more than 1 kV.
The SCR has appeared in the market under different names such as thyristor, thyrode transistor.
It is a unidirectional power switch and is being extensively used in switching d.c. and a.c., rectifying
a.c. to give controlled d.c. output, converting d.c. into a.c. etc. In this chapter, we shall examine the
various characteristics of silicon controlled rectifiers and their increasing applications in power elec-
tronics.
www.JobsCare.info
Silicon Controlled Rectifiers 555
20.1 Silicon Controlled Rectifier (SCR)
A silicon *controlled rectifier is a semiconductor **device that acts as a true electronic switch. It can

Fig. 20.1
change alternating current into direct current and at the same time can control the amount of power
fed to the load. Thus SCR combines the features of a rectifier and a transistor.
Constructional details. When a pn junction is added to a junction transistor, the resulting three
pn junction device is called a silicon controlled rectifier. Fig. 20.1 (i) shows its construction. It is
clear that it is essentially an ordinary rectifier (pn) and a junction transistor (npn) combined in one
unit to form pnpn device. Three terminals are taken; one from the outer p-type material called anode
A, second from the outer n-type material called cathode K and the third from the base of transistor
section and is called gate G. In the normal operating conditions of SCR, anode is held at high positive
potential w.r.t. cathode and gate at small positive potential w.r.t. cathode. Fig. 20.1 (ii) shows the
symbol of SCR.
The silicon controlled rectifier is a solid state equivalent of thyratron. The gate, anode and
cathode of SCR correspond to the grid, plate and cathode of thyratron. For this reason, SCR is
sometimes called thyristor.

Typical SCR Packages

20.2 Working of SCR


In a silicon controlled rectifier, load is connected in series with anode. The anode is always kept at
positive potential w.r.t. cathode. The working of SCR can be studied under the following two heads:

* Why not germanium controlled rectifier ? The device is made of silicon because leakage current in
silicon is very small as compared to germanium. Since the device is used as a switch, it will carry leakage
current in the off condition which should be as small as possible.
** It got this name because it is a silicon device and is used as a rectifier and that rectification can be con-
trolled.
www.JobsCare.info
556  Principles of Electronics
(i) When gate is open. Fig. 20.2 shows the SCR circuit with gate open i.e. no voltage applied
to the gate. Under this condition, junction J2 is reverse biased while junctions J1 and J3 are forward
biased. Hence, the situation in the junctions J1 and J3 is just as in a npn transistor with base open.
Consequently, no current flows through the load RL and the SCR is cut off. However, if the applied
voltage is gradually increased, a stage is reached when * reverse biased junction J2 breaks down. The
SCR now conducts ** heavily and is said to be in the ON state. The applied voltage at which SCR
conducts heavily without gate voltage is called Breakover voltage.

Fig. 20.2
(ii) When gate is positive w.r.t. cathode. The SCR can be made to conduct heavily at smaller
applied voltage by applying a small positive potential to the gate as shown in Fig. 20.3. Now junction
J3 is forward biased and junction J2 is reverse biased. The electrons from n-type material start mov-
ing across junction J3 towards left whereas holes from p-type towards the right. Consequently, the
electrons from junction J3 are attracted across junction J2 and gate current starts flowing. As soon as
the gate current flows, anode current increases. The increased anode current in turn makes more
electrons available at junction J2. This process continues and in an extremely small time, junction J2
breaks down and the SCR starts conducting heavily. Once SCR starts conducting, the gate (the
reason for this name is obvious) loses all control. Even if gate voltage is removed, the anode current
does not decrease at all. The only way to stop conduction (i.e. bring SCR in off condition) is to reduce
the applied voltage to zero.

Fig. 20.3

* The whole applied voltage V appears as reverse bias across junction J2 as junctions J1 and J3 are forward
biased.
** Because J1 and J3 are forward biased and J2 has broken down.
www.JobsCare.info
Silicon Controlled Rectifiers 557
Conclusion. The following conclusions are drawn from the working of SCR :
(i) An SCR has two states i.e. either it does not conduct or it conducts heavily. There is no state
inbetween. Therefore, SCR behaves like a switch.
(ii) There are two ways to turn on the SCR. The first method is to keep the gate open and make
the supply voltage equal to the breakover voltage. The second method is to operate SCR with supply
voltage less than breakover voltage and then turn it on by means of a small voltage ( typically 1.5 V,
30 mA) applied to the gate.
(iii) Applying small positive voltage to the gate is the normal way to close an SCR because the
breakover voltage is usually much greater than supply voltage.
(iv) To open the SCR (i.e. to make it non-conducting ), reduce the supply voltage to zero.
20.3 Equivalent Circuit of SCR
The SCR shown in Fig. 20.4 (i) can be visualised as separated into two transistors as shown in

Fig. 20.4
Fig. 20.4 (ii). Thus, the equivalent circuit of SCR is composed of pnp transistor and npn transistor
connected as shown in Fig. 20.4. (iii). It is clear that collector of each transistor is coupled to the base
of the other, thereby making a positive feedback loop.
The working of SCR can be easily explained from
its equivalent circuit. Fig. 20.5. shows the equiva-
lent circuit of SCR with supply voltage V and load
resistance RL. Assume the supply voltage V is less
than breakover voltage as is usually the case. With
gate open (i.e. switch S open), there is no base cur-
rent in transistor T2. Therefore, no current flows in
the collector of T2 and hence that of T1. Under such
conditions, the SCR is open. However, if switch S is
closed, a small gate current will flow through the base
of T2 which means its collector current will increase.
The collector current of T2 is the base current of T1.
Therefore, collector current of T1 increases. But col- Fig. 20.5
lector current of T1 is the base current of T2. This
action is accumulative since an increase of current in
one transistor causes an increase of current in the other transistor. As a result of this action, both
www.JobsCare.info
558  Principles of Electronics
transistors are driven to saturation, and heavy current flows through the load RL. Under such condi-
tions, the SCR closes.
20.4 Important Terms
The following terms are much used in the study of SCR :
(i) Breakover voltage (ii) Peak reverse voltage
(iii) Holding current (iv) Forward current rating
(v) Circuit fusing rating
(i) Breakover voltage. It is the minimum forward voltage, gate being open, at which SCR
starts conducting heavily i.e. turned on.
Thus, if the breakover voltage of an SCR is 200 V, it means that it can block a forward voltage
(i.e. SCR remains open) as long as the supply voltage is less than 200 V. If the supply voltage is more
than this value, then SCR will be turned on. In practice, the SCR is operated with supply voltage less
than breakover voltage and it is then turned on by means of a small voltage applied to the gate.
Commercially available SCRs have breakover voltages from about 50 V to 500 V.
(ii) Peak reverse voltage (PRV). It is the maximum reverse voltage (cathode positive w.r.t.
anode) that can be applied to an SCR without conducting in the reverse direction.
Peak reverse voltage (PRV) is an important consideration while connecting an SCR in an a.c.
circuit. During the negative half of a.c. supply, reverse voltage is applied across SCR. If PRV is
exceeded, there may be avalanche breakdown and the SCR will be damaged if the external circuit
does not limit the current. Commercially available SCRs have PRV ratings upto 2.5 kV.
(iii) Holding current. It is the maximum anode current, gate being open, at which SCR is
turned off from ON conditions.
As discussed earlier, when SCR is in the conducting state, it cannot be turned OFF even if gate
voltage is removed. The only way to turn off or open the SCR is to reduce the supply voltage to
almost zero at which point the internal transistor comes out of saturation and opens the SCR. The
anode current under this condition is very small (a few mA) and is called holding current. Thus, if an
SCR has a holding current of 5mA, it means that if anode current is made less than 5mA, then SCR
will be turned off.
(iv) Forward current rating. It is the maximum anode current that an SCR is capable of
passing without destruction.
Every SCR has a safe value of forward current which it can conduct. If the value of current
exceeds this value, the SCR may be destroyed due to intensive heating at the junctions. For example,
if an SCR has a forward current rating of 40A, it means that the SCR can safely carry only 40 A. Any
attempt to exceed this value will result in the destruction of the SCR. Commercially available SCRs
have forward current ratings from about 30A to 100A.
2
(v) Circuit fusing (I t) rating. It is the product of square of forward surge current and the time
of duration of the surge i.e.,
Circuit fusing rating = I2t
The circuit fusing rating indicates the maximum forward surge current capability of SCR. For
2
example, consider an SCR having circuit fusing rating of 90 A s. If this rating is exceeded in the SCR
circuit, the device will be destroyed by excessive power dissipation.
Example 20.1. An SCR has a breakover voltage of 400 V, a trigger current of 10 mA and
holding current of 10 mA. What do you infer from it ? What will happen if gate current is made
15 mA ?
Solution. (i) Breakover voltage of 400 V. It means that if gate is open and the supply voltage is
www.JobsCare.info
Silicon Controlled Rectifiers 559
400 V, then SCR will start conducting heavily. However, as long as the supply voltage is less than
400 V, the SCR stays open i.e. it does not conduct.
(ii) Trigger current of 10 mA. It means that if the supply voltage is less than breakover voltage
(i.e. 400 V) and a minimum gate current of 10 mA is passed, the SCR will close i.e. starts conducting
heavily. The SCR will not conduct if the gate current is less than 10 mA. It may be emphasised that
triggering is the normal way to close an SCR as the supply voltage is normally much less than the
breakover voltage.
(iii) Holding current of 10 mA. When the SCR is conducting, it will not open (i.e. stop conduct-
ing) even if triggering current is removed. However, if supply voltage is reduced, the anode current
also decreases. When the anode current drops to 10 mA, the holding current, the SCR is turned off.
(iv) If gate current is increased to 15 mA, the SCR will be turned on lower supply voltage.
Example 20.2. An SCR in a circuit is subjected to a 50 A surge that lasts for 12 ms. Determine
whether or not this surge will destroy the device. Given that circuit fusing rating is 90 A 2s.
2 2 3 2
Solution. Circuit fusing rating = I t = (50) (12 10 ) = 30 A s
2
Since this value is well below the maximum rating of 90 A s, the device will not be destroyed.
2
Example 20.3. An SCR has a circuit fusing rating of 50 A s. The device is being used in a circuit
where it could be subjected to a 100 A surge. Determine the maximum allowable duration of such a
surge.
2
I t (rating)
Solution. tmax = where Is = known value of surge current
I s2

50 3
tmax = 2
= 5 10 s = 5 ms
(100)
Example 20.4. A 220 resistor is connected in series with the gate of an SCR as shown in Fig.
20.6. The gate current required to fire the SCR is 7mA. What is the input voltage (Vin) required to fire
the SCR ?
Solution. The input voltage must overcome the junction voltage
between the gate and cathode (0.7V) and also cause 7mA to flow through
the 220 resistor. According to Kirchhoffs voltage law, Vin is given by;
Vin = VGK + IGR
= 0.7V + (7mA) (220) = 2.24V
20.5 V-I Characteristics of SCR
It is the curve between anode-cathode voltage (V) and anode current (I)
of an SCR at constant gate current. Fig. 20.7 shows the V-I characteris- Fig. 20.6
tics of a typical SCR.
(i) Forward characteristics. When anode is positive w.r.t. cathode, the curve between V and I is
called the forward characteristic. In Fig. 20.7, OABC is the forward characteristic of SCR at IG = 0.
If the supply voltage is increased from zero, a point is reached (point A) when the SCR starts conduct-
ing. Under this condition, the voltage across SCR suddenly drops as shown by dotted curve AB and
most of supply voltage appears across the load resistance RL. If proper gate current is made to flow,
SCR can close at much smaller supply voltage.
(ii) Reverse characteristics. When anode is negative w.r.t. cathode, the curve between V and I is
known as reverse characteristic. The reverse voltage does come across SCR when it is operated with
a.c. supply. If the reverse voltage is gradually increased, at first the anode current remains small (i.e.
leakage current) and at some reverse voltage, avalanche breakdown occurs and the SCR starts con-
ducting heavily in the reverse direction as shown by the curve DE. This maximum reverse voltage at
which SCR starts conducting heavily is known as reverse breakdown voltage.
www.JobsCare.info
560  Principles of Electronics

Fig. 20.7

20.6 SCR in Normal Operation


In order to operate the SCR in normal operation, the following points are kept in view :
(i) The supply voltage is generally much less than breakover voltage.
(ii) The SCR is turned on by passing an appropriate amount of gate current (a few mA) and not
by breakover voltage.
(iii) When SCR is operated from a.c. supply, the peak reverse voltage which comes during
negative half-cycle should not exceed the reverse breakdown voltage.
(iv) When SCR is to be turned OFF from the ON state, anode current should be reduced to
holding current.
(v) If gate current is increased above the required value, the SCR will close at much reduced
supply voltage.

20.7 SCR as a Switch


The SCR has only two states, namely; ON state and OFF state and no state inbetween. When appro-
priate gate current is passed, the SCR starts conducting heavily and remains in this position indefi-
nitely even if gate voltage is removed. This corresponds to the ON condition. However, when the
anode current is reduced to the holding current, the SCR is turned OFF. It is clear that behaviour of
SCR is similar to a mechanical switch. As SCR is an electronic device, therefore, it is more appropri-
ate to call it an electronic switch.
Advantages of SCR as a switch. An SCR has the following advantages over a mechanical or
electromechanical switch (relay) :
(i) It has no moving parts. Consequently, it gives noiseless operation at high efficiency.
9
(ii) The switching speed is very high upto 10 operations per second.
(iii) It permits control over large current (30100 A) in the load by means of a small gate current
(a few mA).
(iv) It has small size and gives trouble free service.

20.8 SCR Switching


We have seen that SCR behaves as a switch i.e. it has only two states viz. ON state and OFF state. It
is profitable to discuss the methods employed to turn-on or turn-off an SCR.
Silicon Controlled Rectifiers 561
1. SCR turn-on methods. In order to turn on the SCR, the gate voltage VG is increased upto
a minimum value to initiate triggering. This minimum value of gate voltage at which SCR is turned
ON is called gate triggering voltage VGT. The resulting gate current is called gate triggering current
IGT. Thus to turn on an SCR all that we have to do is to apply positive gate voltage equal to VGT or pass
a gate current equal to IGT. For most of the SCRs, VGT = 2 to 10 V and IGT = 100 A to 1500 mA. We
shall discuss two methods to turn on an SCR.

Fig. 20.8
(i) D.C. gate trigger circuit. Fig. 20.8 shows a typical circuit used for triggering an SCR with
a d.c. gate bias. When the switch is closed, the gate receives sufficient positive voltage (= VGT) to turn
the SCR on. The resistance R1 connected in the circuit provides noise suppression and improves the
turn-on time. The turn-on time primarily depends upon the magnitude of the gate current. The higher
the gate-triggered current, the shorter the turn-on time.

Fig. 20.9
(ii) A.C. trigger circuit. An SCR can also be turned on with positive cycle of a.c. gate current.
Fig. 20.9 (ii) shows such a circuit. During the positive half-cycle of the gate current, at some point IG
= IGT, the device is turned on as shown in Fig. 20.9 (i).
2. SCR turn-off methods. The SCR turn-off poses more problems than SCR turn-on. It is
because once the device is ON, the gate loses all control. There are many methods of SCR turn-off but
only two will be discussed.
www.JobsCare.info
562  Principles of Electronics
(i) Anode current interruption. When the anode current is reduced below a minimum value
called holding current, the SCR turns off. The simple way to turn off the SCR is to open the line
switch S as shown in Fig. 20.10.
(ii) Forced commutation. The method of discharging a capacitor in parallel with an SCR to
turn off the SCR is called forced commutation. Fig. 20.11 shows the forced commutation of SCR
where capacitor C performs the commutation. Assuming the SCRs are switches with SCR1 ON and
SCR2 OFF, current flows through the load and C as shown in Fig. 20.11. When SCR2 is triggered on,
C is effectively paralleled across SCR1. The charge on C is then opposite to SCR1s forward voltage,
SCR1 is thus turned off and the current is transferred to RSCR2 path.

Fig. 20.10 Fig. 20.11

20.9 SCR Half-Wave Rectifier


One important application of an SCR is the controlled half-wave rectification. Fig. 20.12 (i) shows
the circuit of an SCR half-wave rectifier. The a.c. supply to be rectified is supplied through the
transformer. The load resistance RL is connected in series with the anode. A variable resistance r is
inserted in the gate circuit to control the gate current.

Fig. 20.12
Operation. The a.c. supply to be converted into d.c. supply is applied to the primary of the
transformer. Suppose the peak reverse voltage appearing across secondary is less than the reverse
www.JobsCare.info
Silicon Controlled Rectifiers 563
breakdown voltage of the SCR. This condition ensures that SCR will not break down during negative
half-cycles of a.c. supply. The circuit action is as follows :
(i) During the negative half-cycles of a.c. voltage appearing across secondary, the SCR does not
conduct regardless of the gate voltage. It is because in this condition, anode is negative w.r.t. cathode
and also PRV is less than the reverse breakdown voltage.
(ii) The SCR will conduct during the positive half-cycles provided proper gate current is made
to flow. The greater the gate current, the lesser the supply voltage at which SCR is turned ON. The
gate current can be changed by the variable resistance r as shown in Fig. 20.12 (i).
(iii) Suppose that gate current is adjusted to such a value that SCR closes at a positive voltage V1
which is less than the peak voltage Vm. Referring to Fig. 20.12 (ii), it is clear that SCR will start
conducting when secondary a.c. voltage becomes V1 in the positive half-cycle. Beyond this, the SCR
will continue to conduct till voltage becomes zero at which point it is turned OFF. Again at the start
of the next positive half-cycle, SCR will start conducting when secondary voltage becomes V1.
(iv) Referring to Fig. 20.12 (ii), it is clear that firing angle is i.e. at this angle in the positive
half-cycle, SCR starts conduction. The conduction angle is (= 180 ).
It is worthwhile to distinguish between an ordinary half-wave rectifier and SCR half-wave recti-
fier. Whereas an ordinary half-wave rectifier will conduct full positive half-cycle, an SCR half-wave
rectifier can be made to conduct full or part of a positive half-cycle by proper adjustment of gate
current. Therefore, an SCR can control power fed to the load and hence the name controlled rectifier.
Mathematical treatment. Referring to Fig. 20.12 (i), let v = Vm sin be the alternating volt-
age that appears across the secondary. Let be the firing angle. It means that rectifier will conduct
from to 180 during the positive half-cycles.
180 180
1 Vm
Average output, Vav =
2

Vm sin d =
2 sin d

Vm
=
2
[ cos ]180

Vm
= (cos cos 180)
2
Vm
Vav = (1 + cos )
2
Vav Vm
Average current, Iav = = (1 + cos )
RL 2 RL
The following points may be noted :
(i) If the firing angle = 0, then full positive half-cycle will appear across the load RL and the
output current becomes :
Vm V
Iav = (1 + cos 0) = m
2 RL RL
This is the value of average current for ordinary half-wave rectifier. This is expected since the
full positive half-cycle is being conducted.
(ii) If = 90, then average current is given by :
Vm Vm
Iav = (1 + cos 90) =
2 RL 2 RL
This shows that greater the firing angle , the smaller is the average current and vice-versa.
www.JobsCare.info
564  Principles of Electronics
Example 20.5. A half-wave rectifier circuit employing an SCR is adjusted to have a gate cur-
rent of 1mA. The forward breakdown voltage of SCR is 100 V for Ig = 1mA. If a sinusoidal voltage of
200 V peak is applied, find :
(i) firing angle (ii) conduction angle (iii) average current.
Assume load resistance = 100 and the holding current to be zero.
Solution. v = Vm sin
Here, v = 100 V, Vm = 200 V
(i) 100 = 200 sin

or sin = 100 = 0.5


200
= sin1 (0.5) = 30 i.e. Firing angle, = = 30
(ii) Conduction angle, = 180 = 180 30 = 150

(1 + cos ) = 200 (1 + cos 30) = 59.25 V


Vm
(iii) Average voltage =
2 2
Average voltage
Average current = = 59.25 = 0.5925 A
RL 100

Example 20.6. An SCR half-wave rectifier has a forward breakdown voltage of 150 V when a
gate current of 1 mA flows in the gate circuit. If a sinusoidal voltage of 400 V peak is applied, find:
(i) firing angle (ii) average output voltage
(iii) average current for a load resistance of 200 (iv) power output
Assume that the gate current is 1mA throughout and the forward breakdown voltage is more
than 400 V when Ig = 1 mA.
Solution. Vm = 400 V, v = 150 V, RL = 200
(i) Now v = Vm sin

or sin = v = 150 = 0.375


Vm 400
1
i.e. firing angle, (= ) = sin 0.375 = 22
(ii) Average output voltage is
V 400 (1 + cos 22) =
Vav = m (1 + cos 22) = 122.6 V
2 2
average output voltage
(iii) Average current, Iav = = 122.6 = 0.613 A
RL 200
(iv) Output power = Vav Iav = 122.6 0.613 = 75.15 W

Example 20.7. An a.c. voltage v = 240 sin314 t is applied to an SCR half-wave rectifier. If the
SCR has a forward breakdown voltage of 180 V, find the time during which SCR remains off.
Solution. The SCR will remain off till the voltage across it reaches 180 V. This is shown in Fig.
20.13. Clearly, SCR will remain off for t second.
Now v = Vm sin 314 t
Here v = 180 V; Vm = 240 V
180 = 240 sin (314 t)
www.JobsCare.info
Silicon Controlled Rectifiers 565
180 = 0.75
or sin 314 t =
240
1
or 314 t = sin (0.75)
= 48.6 = 0.848 radian
0.848
t = = 0.0027 sec
314
= 2.7 millisecond
Example 20.8. In an SCR half-wave rectifier circuit,
what peak-load current will occur if we measure an aver-
age (d.c.) load current of 1A at a firing angle of 30 ? Fig. 20.13
Solution. Let Im be the peak load current.
Vm
Now, Iav = 2R (1 + cos )
L
Im Vm
= (1 + cos ) ( Q Im = )
2 RL
2 I av
Im =
1 + cos
Here Iav = Idc = 1A ; = 30
2 1
Im = = 3.36A
1 + cos 30
Example 20.9. Power (brightness) of a 100W, 110 V tungsten lamp is to be varied by control-
ling the firing angle of an SCR in a half-wave rectifier circuit supplied with 110 V a.c. What r.m.s.
voltage and current are developed in the lamp at firing angle = 60?
Solution. The a.c. voltage is given by;
v = Vm sin
Let be the firing angle as shown in Fig. 20.14. This means that the SCR will fire (i.e. start
conducting) at = . Clearly, SCR will conduct from to 180.

1 V 2 sin 2 d
E2r.m.s. =
2 m

*
= Vm2 2 ( ) + sin 2
8
2( ) + sin 2
Er.m.s. = Vm
8 Fig. 20.14
Here, Vm = 2 110 = 156V; = 60 = /3
2 ( / 3) + sin 120
Er.m.s. = 156 = 70 V
8
2 2
V = (110)
Lamp resistance, RL = = 121
P 100
Ir.m.s. = r.m.s. = 70 = 0.58 A
E

RL 121

* On carrying out the integration.


www.JobsCare.info
566  Principles of Electronics
Comments. The load current can be decreased by increasing the firing angle. The larger the
value of , the smaller is the load current and vice-versa. This method of controlling power is very
efficient because other methods, such as added series resistance, waste much power in the added
control element.
20.10 SCR Full-Wave Rectifier
Fig. 20.15 (i) shows the circuit of SCR full-wave rectifier. It is exactly like an ordinary centre-tap circuit
except that the two diodes have been replaced by two SCRs. The gates of both SCRs get their

Fig. 20.15
supply from two gate controls. One SCR conducts during the positive half-cycle and the other during
the negative half-cycle. Consequently, full-wave rectified output is obtained across the load.
Operation. The angle of conduction can be changed by adjusting the gate currents. Suppose the
gate currents are so adjusted that SCRs conduct as the secondary voltage (across half winding) be-
comes V1. During the positive half-cycle of a.c. across secondary, the upper end of secondary is
positive and the lower end negative. This will cause SCR1 to conduct. However, the conduction will
start only when the voltage across the upper half of secondary becomes V1 as shown in Fig. 20.15 (ii).
In this way, only shaded portion of positive half-cycle will pass through the load.
During the negative half-cycle of a.c. input, the upper end of secondary becomes negative and
the lower end positive. This will cause SCR2 to conduct when the voltage across the lower half of
secondary becomes V1. It may be seen that current through the load is in the same direction (d.c.) on
both half-cycles of input a.c. The obvious advantage of this circuit over ordinary full-wave rectifier
circuit is that by adjusting the gate currents, we can change the conduction angle and hence the output
voltage.
Mathematical treatment. Referring to Fig. 20.15 (i), let v = Vm sin be the alternating voltage
that appears between centre tap and either end of secondary. Let be the firing angle.
180 180
1 Vm
Average output, Vav =


Vm sin d =
sin d

V 180 V
= m [ cos ] = m (cos cos 180)

www.JobsCare.info
Silicon Controlled Rectifiers 567
Vm
Vav = (1 + cos )

This value is double that of a half-wave rectifier. It is expected since now negative half-cycle is
also rectified.
Vav V
Average current, Iav = = m (1 + cos )
RL RL
Example 20.10. An SCR full-wave rectifier supplies to a load of 100 . If the peak a.c. voltage
between centre tap and one end of secondary is 200V, find (i) d.c. output voltage and (ii) load current
for a firing angle of 60.
Solution. Vm = 200 V; = 60; RL = 100

(1 + cos ) = 200 (1 + cos 60) = 95.5 V


Vm
(i) D.C. output voltage, Vav =

= 95.5 = 0.955 A
Vav
(ii) Load current, Iav =
RL 100
Example 20.11. Power (brightness) of a 100 W, 110 V lamp is to be varied by controlling firing
angle of SCR full-wave circuit; the r.m.s. value of a.c. voltage appearing across each SCR being
110 V. Find the r.m.s. voltage and current in the lamp at firing angle of 60.
Solution. Let v = vm sin be the alternating voltage that
appears between centre tap and either end of the secondary. Let
be the firing angle as shown in Fig. 20.16. This means that SCR
will conduct at = . Clearly, SCR circuit will conduct from to
180.

1 V 2 sin 2 d
2
Er.m.s. =
m
Fig. 20.16
*
2 ( ) + sin 2
= Vm2
4
2 ( ) + sin 2
Er.m.s. = Vm
4
Here Vm = 110 2 = 156V ; = 60
2 ( /3) + sin 120
Er.m.s. = 156 = 98.9V
4
2 2
(110)
Lamp resistance, RL = V = = 121
P 100
Er.m.s. 98.9
Ir.m.s. = = = 0.82 A
RL 121

20.11 Single-Phase SCR Inverter Circuit


SCR inverter provides an efficient and economical way of converting direct current or voltage into
alternating current or voltage. In this application, SCR acts as a controlled switch, alternately opening
and closing a d.c. circuit. Fig. 20.17 shows the basic inverter circuit. Here, a.c. voltage is generated
by alternately closing and opening switches S1 and S2. Replacing, the mechanical switches

* On carrying out the integration.


www.JobsCare.info
568  Principles of Electronics

Fig. 20.17
with SCRs, whose gates are triggered by an external pulse generator, we get the practical SCR inverter
as shown in Fig. 20.18.

Fig. 20.18
Circuit Action
The circuit action is as under :
(i) When conduction is initiated by applying a positive trigger pulse to SCR 1 (SCR 2 is as-
sumed OFF), the voltage across SCR decreases rapidly as the current through it increases. At the
same time, the capacitor C charges through SCR 1 in the polarity shown. The load current flows
through inductor L, upper half of the transformer primary winding and SCR1.
(ii) When a firing pulse is applied to the gate of SCR 2, this SCR turns on and conducts current.
At this instant, capacitor C begins to discharge through SCR 1 and SCR 2. This discharge current
flows through SCR 1 in a reverse direction. This reverse current turns off SCR1. At this time, with
SCR 1 turned off, the capacitor voltage (approximately*2E) appears across SCR 1 as a reverse
voltage, long enough for this SCR to recover for forward blocking.
Simultaneously, during this interval, conducting SCR 2 allows the capacitor to discharge through
the transformer primary winding and inductor L. The function of L is to control the discharge rate of
C to allow sufficient time for SCR 1 to turn OFF. Capacitor C discharges rapidly from 2E to zero
and then charges up in the opposite direction to + 2E. The load current is now carried through the
second half of the transformer primary winding and SCR 2.
(iii) When trigger pulse is applied to the gate of SCR 1, this device will conduct and SCR 2 will
turn off by the process just described. In this way, SCR 1 and SCR 2 alternately turn ON and OFF.
Consequently, a.c. output is obtained as shown in Fig. 20.18.

* The capacitor voltage will charge to double the suppply voltage (E) as a result of transformer action be-
tween the two primary windings.
www.JobsCare.info
Silicon Controlled Rectifiers 569
The a.c. waveform produced by a single-phase inverter is a poor version of sine wave and would
not be suitable for most industrial, commercial and domestic loads. More complex inverters using
multiple SCRs and sophisticated triggering circuits are capable of generating a.c. voltages that are
extremely close to a pure sine wave.
20.12 Applications of SCR
The ability of an SCR to control large currents in a load by means of small gate current makes this
device useful in switching and control applications. Some of the important applications of SCR are
discussed below :
(i) SCR as static contactor. An important application of SCR is for switching operations. As
SCR has no moving parts, therefore, when it is used as a switch, it is often called a static contactor.

Fig. 20.19
Fig. 20.19 shows the use of SCR to switch ON or OFF a.c. power to a load RL. Resistances R1 and R2
are for the protection of diodes D1 and D2 respectively. Resistance R3 is the gate current limiting
resistor. To start the circuit, switch is closed. During the positive half-cycle of a.c. supply, end A is
positive and end B is negative. Then diode D2 sends gate current through SCR1. Therefore SCR1 is
turned ON while SCR2 remains OFF as its anode is negative w.r.t. cathode. The current conduction
by SCR1 follows the path ARLK1BA. Similarly, in the next half-cycle, SCR2 is turned ON and con-
ducts current through the load. It may be seen that switch S handles only a few mA of gate current to
switch ON several hundred amperes in the load RL. This is a distinct advantage over a mechanical
switch.

Fig. 20.20
570  Principles of Electronics
(ii) SCR for power control. It is often necessary to control power delivered to some load such
as the heating element of a furnace. Series resistances or potentiometers cannot be used because they
waste power in high power circuits. Under such conditions, silicon controlled rectifiers are used
which are capable of adjusting the transmitted power with little waste. Fig. 20.20 shows a common
circuit for controlling power in the load RL. During the positive half-cycle of a.c. supply, end A is
positive and end B is negative. Therefore, capacitor C2 is charged through AD1 RC2 D4B. The charge
on the capacitor C2 depends upon the value of potentiometer R. When the capacitor C2 is charged
through a sufficient voltage, it discharges through the zener Z. This gives a pulse to the primary and
hence secondary of transformer T2. This turns on SCR2 which conducts currents through the load RL.
During negative half-cycle of supply, the capacitor C1 is charged. It discharges through the zener and
fires SCR1 which conducts current through the load.

Fig. 20.21
The angle of conduction can be controlled by the potenti-
ometer R. The greater the resistance of R, lesser is the voltage
across C1 or C2 and hence smaller will be the time during which
SCR1 and SCR2 will conduct in a full cycle. In this way, we
can control a large power of several kW in the load RL with the
help of a small potentiometer R.
(iii) SCRs for speed control of d.c. shunt motor. The
conventional method of speed control of d.c. shunt motor is to
change the field excitation. But change in field excitation
changes the motor torque also. This drawback is overcome in
SCR control as shown in Fig. 20.21. Diodes D1, D2, D3 and D4
form the bridge. This bridge circuit converts a.c. into d.c. and
supplies it to the field winding of the motor. During the posi-
tive half-cycle of a.c. supply, SCR1 conducts because it gets
gate current from bridge circuit as well as its anode is positive
w.r.t. cathode. The armature winding of the motor gets current. SCR Power Control
The angle of conduction can be changed by varying the gate
www.JobsCare.info
Silicon Controlled Rectifiers 571
current. During the negative half-cycle of a.c. sup-
ply, SCR2 provides current to the armature winding.
In this way, the voltage fed to the motor armature and
hence the speed can be controlled.
(iv) Overlight detector. Fig. 20.22 shows the
use of SCR for overlight detection. The resistor R is
a photo-resistor, a device whose resistance decreases
with the increase in light intensity. When the light
falling on R has normal intensity, the value of R is
high enough and the voltage across R1 is insufficient
to trigger the SCR. However, when R is in strong
light, its resistance decreases and the voltage drop
across R1 becomes high enough to trigger the SCR.
Consequently, the buzzer sounds the alarm. It may Fig. 20.22
be noted that even if the strong light disappears, the
buzzer continues to sound the alarm. It is because once the SCR is fired, the gate loses all control.
(v) SCR Crowbar. A crowbar is a circuit that is used to protect a voltage-sensitive load from
excessive d.c. power supply output voltages. Fig. 20.23 shows the SCR crowbar circuit. It consists of
a zener diode, a gate resistor RG and an SCR. It also contains a *snubber to prevent false triggering.

Fig. 20.23
Operation. The circuit action is as under:
(a) Under normal conditions, the zener diode and the SCR are OFF. With zener diode being in
cutoff, there is no current through RG and no voltage drop occurs across this resistor. This means that
the gate of SCR is at 0V so that the SCR is in the off state. Therefore, as long as zener diode is off, the
SCR behaves as an open and will not affect either the d.c. power supply or the load.
(b) Suppose the output voltage from the d.c. power supply suddenly
increases. This causes the zener diode to break down and conduct current.
As the current flows through the zener diode, voltage is developed across
resistor RG which causes the SCR to conduct current. When the SCR con-
ducts, the voltage source is shorted by the SCR. The supply voltage fuse
blows out and the load is protected from overvoltage.
20.13 Light-Activated SCR
The light-activated SCR (LASCR) is the light sensitive equivalent of the nor-
mal SCR and is shown in Fig. 20.24. As the name suggests, its state is con-
trolled by the light falling on depletion layers. In a normal SCR, gate current Fig. 20.24

* It is an RC circuit connected between the SCR anode and cathode to eliminate false triggering.
www.JobsCare.info
572  Principles of Electronics
turns on the device. In the *LASCR, instead of having the external gate current applied, light shinning
on the device turns it ON. Just as a normal SCR, the LASCR will continue to conduct even if the light
source is removed. The LASCRs find many applications including optical light controls, relays, phase
control, motor control and a large number of computer applications. The maximum current (r.m.s.)
and power (gate) ratings for LASCRs commercially available today are about 3A and 0.1W. It may be
noted that LASCR is most sensitive to light when the gate terminal is open. Its sensitivity can be
reduced and controlled by the insertion of a gate resistor.
Example 20.12. The SCR of Fig. 20.25 has gate trigger voltage VT = 0.7V, gate trigger current
IT = 7 mA and holding current IH = 6 mA.
(i) What is the output voltage when the SCR is off ?
(ii) What is the input voltage that triggers the SCR ?
(iii) If VCC is decreased until the SCR opens, what is the value of VCC ?
Solution.
(i) When the SCR is off (i.e. it is not conducting),
there is no current through the 100 resistor.
Vout = Supply voltage VCC = 15V
(ii) The input voltage Vin must overcome VT (=0.7V)
and also cause 7 mA to flow through 1 k resistor.
Vin = VT + IT R = 0.7 + (7 mA) (1 k)
= 7.7V
(iii) In order to open the SCR, the VCC must be re-
duced so that anode current is equal to IH.
VCC VT
IH = 100
Fig. 20.25
or VCC = (100) (IH) + VT
= (100) (6 mA) + 0.7 = 1.3V
Example 20.13. In Fig. 20.26, the SCR has a trigger voltage of 0.7 V. Calculate the supply
voltage that turns on the crowbar. Ignore zener diode resistance.

Fig. 20.26
Solution. The breakdown voltage of the zener is 5.6V. To turn on the SCR, the voltage across
68 has to be equal to VT (= 0.7V).
VCC = VZ + VT = 5.6 + 0.7 = 6.3V

* For maximum sensitivity to light, the gate is left open.


www.JobsCare.info
Silicon Controlled Rectifiers 573
When the supply voltage becomes 6.3 V, the zener breaks down and starts conducting. The
voltage VT (= 0.7V) across 68 forces the SCR into conduction. When the SCR conducts, the supply
voltage is shorted by the SCR and the fuse in the supply voltage burns out. Thus the load (100) is
protected from overvoltage.
Example. 20.14. The zener diode of Fig. 20.27 has a tolerance of 10% and the trigger voltage
can be as high as 1.5V. What is the maximum supply voltage where crowbarring takes place?

Fig. 20.27
Solution. The breakdown voltage of the zener diode is 12V and it has a tolerance of 10%. It
means that breakdown voltage of zener can vary from 10.8V to 13.2V. Since the trigger voltage of
SCR has a maximum value of 1.5 V,
Maximum value of supply voltage for crowbarring
= 13.2V + 1.5V = 14.7V
Example. 20.15. The circuit of Fig. 20.28 is in a dark room. When a
bright light is turned on, the LASCR fires. What is the approximate output
voltage? If the bright light is turned off, what is the output voltage?
Solution. Fig. 20.28 shows a light-activated SCR, also known as a
photo-SCR. When light falls on the device, it starts conducting and the
output voltage is ideally,
Vout = 0V
However, if we take into account anode-cathode drop, Vout = 0.7V.
When light is turned off, the LASCR stops conducting and the output Fig. 20.28
voltage is equal to the supply voltage VCC i.e.
Vout = VCC = + 25V
Example 20.16. Give a simple method for
testing an SCR.
Solution. Fig. 20.29 shows a simple cir-
cuit for testing an SCR. The test lamp serves two
purposes. First, it is a visual indicator of current
conduction. Secondly, it limits current through
the SCR.
(i) When switch S is closed, the lamp
should not light for the SCR to be good. It is
because voltage is *applied only between anode
and cathode but there is no trigger voltage. If Fig. 20.29
the lamp lights, the SCR is shorted.

* It is understood that the applied voltage is less than the breakover voltage of the SCR.
574  Principles of Electronics
(ii) Now touch R1 momentarily between gate and anode terminals. For the SCR to be good, the
lamp should light and *continue to light. If it does not, the SCR is open.
Note that the lamp will be on at half brilliance because the SCR conducts only every other half-
cycle.

MULTIPLE-CHOICE QUESTIONS
1. An SCR has ............... pn junctions. (iii) a rectifier and capacitor
(i) two (ii) three (iv) none of the above
(iii) four (iv) none of the above 11. The control element in an SCR is ...............
2. An SCR is a solid state equivalent of ............ (i) cathode (ii) anode
(i) triode (ii) pentode (iii) anode supply (iv) gate
(iii) gas-filled triode (iv) tetrode 12. The normal way to turn on an SCR is by
3. An SCR has ............... semiconductor layers. ...............
(i) two (ii) three (i) breakover voltage
(iii) four (iv) none of the above (ii) appropriate anode current
4. An SCR has three terminals viz. ............... (iii) appropriate gate current
(i) cathode, anode, gate (iv) none of the above
(ii) anode, cathode, grid 13. An SCR is turned off by ...............
(iii) anode, cathode, drain (i) reducing anode voltage to zero
(iv) none of the above (ii) reducing gate voltage to zero
5. An SCR behaves as a ............... switch. (iii) reverse biasing the gate
(i) unidirectional (ii) bidirectional (iv) none of the above
(iii) mechanical (iv) none of the above 14. An SCR is a ............... triggered device.
6. An SCR is sometimes called ............... (i) voltage
(i) triac (ii) current
(ii) diac (iii) voltage as well as current
(iii) unijunction transistor (iv) none of the above
(iv) thyristor 15. In an SCR circuit, the supply voltage is gen-
7. An SCR is made of ............... erally ............... that of breakover voltage.
(i) germanium (ii) silicon (i) equal to (ii) less than
(iii) carbon (iv) none of the above (iii) greater than (iv) none of the above
8. In the normal operation of an SCR, anode is 16. When an SCR is turned on, the voltage across
............... w.r.t. cathode. it is about ..............................
(i) at zero potential (i) zero (ii) 10 V
(ii) negative (iii) 0.1 V (iv) 1V
(iii) positive 17. An SCR is made of silicon and not germa-
nium because silicon ...............
(iv) none of the above
(i) is inexpensive
9. In normal operation of an SCR, gate is
............... w.r.t. cathode. (ii) is mechanically strong
(i) positive (iii) has small leakage current
(ii) negative (iv) is tetravalent
(iii) at zero potential 18. An SCR is turned off when ...............
(iv) none of the above (i) anode current is reduced to zero
10. An SCR combines the features of ............... (ii) gate voltage is reduced to zero
(i) a rectifier and resistance (iii) gate is reverse biased
(ii) a rectifier and transistor (iv) none of the above

* Recall that once the SCR is fired by the gate voltage, it continues to conduct current even if the gate voltage
is removed.
www.JobsCare.info
Silicon Controlled Rectifiers 575
19. In an SCR circuit, the angle of conduction (i) exactly zero
can be changed by ............... (ii) small leakage current
(i) changing anode voltage (iii) large leakage current
(ii) changing gate voltage (iv) none of the above
(iii) reverse biasing the gate 23. An SCR can exercise control over ...............
(iv) none of the above of a.c. supply.
20. If firing angle in an SCR circuit is increased, (i) positive half-cycles only
the output ............... (ii) negative half-cycles only
(i) remains the same (iii) both positive and negative half-cycles
(ii) is increased (iv) positive or negative half-cycles
(iii) is decreased 24. We can control a.c. power in a load by con-
(iv) none of the above necting ...............
21. If gate current is increased, then anode-cath- (i) two SCRs in series
ode voltage at which SCR closes ...............
(ii) two SCRs in parallel
(i) is decreased
(iii) two SCRs in parallel opposition
(ii) is increased
(iii) remains the same (iv) none of the above
(iv) none of the above 25. When SCR starts conducting, then ...............
22. When SCR is OFF, the current in the circuit loses all control.
is ............... (i) gate (ii) cathode
(iii) anode (iv) none of the above

Answers to Multiple-Choice Questions


1. (ii) 2. (iii) 3. (iii) 4. (i) 5. (i)
6. (iv) 7. (ii) 8. (iii) 9. (i) 10. (ii)
11. (iv) 12. (iii) 13. (i) 14. (ii) 15. (ii)
16. (iv) 17. (iii) 18. (i) 19. (ii) 20. (iii)
21. (i) 22. (ii) 23. (iv) 24. (iii) 25. (i)

Chapter Review Topics


1. Explain the construction and working of an SCR.
2. Draw the equivalent circuit of an SCR and explain its working from this equivalent circuit.
3. Explain the terms breakover voltage, holding current and forward current rating as used in connection
with SCR analysis.
4. Draw the V-I characteristics of an SCR. What do you infer from them ?
5. Explain the action of an SCR as a switch. What are the advantages of SCR switch over a mechanical
or electro-mechanical switch ?
6. Discuss some important applications of SCR.

Problems
1. An SCR has a breakover voltage of 450 V, a trigger current of 15 mA and holding current of 10 mA.
What do you infer from it?
2. An SCR in a circuit is subjected to a 50 A current surge that lasts for 10 ms. Determine whether or not
2
this surge will destroy the device. Given that circuit fusing rating of SCR is 90 A s.
[will not be destroyed]
2
3. An SCR has a circuit fusing rating of 70 A s. The device is being used in a circuit where it could be
subjected to a 100 A surge. Determine the limit on the duration of such a surge. [7ms]
4. An SCR has a circuit fusing rating of 60 A2s. Determine the highest surge current value that SCR can
withstand for a period of 20 ms. [54.77A]
www.JobsCare.info
576  Principles of Electronics
5. In Fig. 20.30, what value of input voltage would be required to cause the SCR to break down if the gate
current required for firing is 10 mA ? [3.7V]

Fig. 20.30 Fig. 20.31


6. In Fig. 20.31, if the trigger current of the SCR is 1.5 mA, what is the input voltage that triggers the SCR?
Given VT = 0.7V. [7.3V]
7. A 24V r.m.s. supply is connected to a half-wave SCR circuit that is triggered at 50. What is the d.c.
voltage delivered to the load ? [8.88V]

Discussion Questions
1. How does SCR differ from an ordinary rectifier ?
2. Why is SCR always turned on by gate current ?
3. Why SCR cannot be used as a bidirectional switch ?
4. How does SCR control the power fed to the load ?
5. Why are SCRs usually used in a.c. circuits?
6. Name three thyristor devices.
7. Why is SCR turned on by high-frequency radiation ?
www.JobsCare.info
22
Electronic Instruments 601

Electronic
Instruments
22.1 Electronic Instruments

22.3 Applications of Multimeter

22.5 Merits and Demerits of Multimeter

22.7 Electronic Voltmeters

22.9 Applications of VTVM

22.11 Transistor Voltmeter Circuit

22.13 Cathode Ray Oscilloscope

22.15 Deflection Sensitivity of CRT

22.17 Display of Signal Waveform on CRO

22.19 Various Controls of CRO

INTR ODUCTION
INTRODUCTION

I
n recent years, the rapid strides and remarkable advances in the field of electronics is partly due
to modern electronic instruments. By using these instruments, we can gather much information
regarding the performance of specific electronic circuit. Electronic instruments are also used for
trouble shooting since they permit readings to be taken so that circuit faults can be located by
ascertaining which component values do not coincide with the pre-established values indicated by the
manufacturer. In fact, electronic instruments are playing a vital role in the fast developing field of
electronics. It is with this view that they have been treated in a separate chapter.
22.1 Electronic Instruments
Those instruments which employ electronic devices for measuring various electrical quantities (e.g.
voltage, current, resistance etc.) are known as electronic instruments.
www.JobsCare.info
602  Principles of Electronics
There are a large number of electronic instruments available for completion of various tests and
measurements. However, in this chapter, we shall confine our attention to the following electronic
instruments :
(i) Multimeter
(ii) Electronic Voltmeters
(iii) Cathode ray oscilloscope
The knowledge of the manner in which each instrument is used plus an understanding of the
applications and limitations of each instrument will enable the reader to utilise such instruments
successfully.
22.2 Multimeter
A multimeter is an electronic instrument which can measure resistances, currents and voltages. It is an
indispensable instrument and can be used for measuring d.c. as well as a.c. voltages and currents.
Multimeter is the most inexpensive equipment and can make various electrical measurements with
reasonable accuracy.
Construction. A multimeter consists of an ordinary pivoted type of moving coil galvanometer.
This galvanometer consists of a coil pivoted on jeweled bearings between the poles of a permanent
magnet. The indicating needle is fastened to the coil. When electric current is passed through the
coil, mechanical force acts and the pointer moves over the scale.
Functions. A multimeter can measure voltages, currents and resistances. To achieve this objec-
tive, proper circuits are incorporated with the galvanometer. The galvanometer in a multimeter is
always of left zero type i.e. normally its needle rests in extreme left position as compared to centre
zero position of ordinary galvanometers.
(i) Multimeter as voltmeter. When a high resistance is connected in series with a galvanom-
eter, it becomes a voltmeter. Fig. 22.1 (i) shows a high resistance R connected in series with the
galvanometer of resistance G. If Ig is the full scale deflection current, then the galvanometer becomes
a voltmeter of range 0 V volts. The required value of series resistance R is given by :
V = Ig R + Ig G
or V/Ig = R + G
or R = V/Ig G

Fig. 22.1
For maximum accuracy, a multimeter is always provided with a number of voltage ranges. This
is achieved by providing a number of high resistances in the multimeter as shown in Fig. 22.1 (ii).
Each resistance corresponds to one voltage range. With the help of selector switch S, we can put any
www.JobsCare.info
Electronic Instruments 603
resistance (R1, R2 and R3) in series with the galva-
nometer. When d.c. voltages are to be measured,
the multimeter switch is turned on to d.c. position.
This puts the circuit shown in Fig. 22.1 (ii) in ac-
tion. By throwing the range selector switch S to a
suitable position, the given d.c. voltage can be mea-
sured.
The multimeter can also measure a.c. voltages.
To permit it to perform this function, a full-wave
rectifier is used as shown in Fig. 22.2. The recti-
fier converts a.c. into d.c. for application to the
galvanometer. The desired a.c. voltage range can
be selected by the switch S. When a.c. voltage is Fig. 22.2
to be measured, the multimeter switch is thrown to
a.c. position. This puts the circuit shown in Fig. 22.2 in action. By throwing the range selector switch
S to a suitable position, the given a.c. voltage can be measured. It may be mentioned here that a.c.
voltage scale is calibrated in r.m.s. values. Therefore, the meter will give the r.m.s. value of the a.c.
voltage under measurement.
(ii) Multimeter as ammeter. When low resistance is connected in parallel with a galvanom-
eter, it becomes an ammeter. Fig. 22.3 (i) shows a low resistance S (generally called shunt) connected
in parallel with the galvanometer of resistance G. If Ig is the full scale deflection current, then the
galvanometer becomes an ammeter of range 0 I amperes. The required value of shunt resistance S
is given by :
Is S = IgG
Is G +1
or Is/Ig = G/S or +1 =
Ig S
Is + I g G+S I G+S
or = or =
Ig S Ig S

Fig. 22.3
In practice, a number of low resistances are connected in parallel with the galvanometer to pro-
vide a number of current ranges as shown in Fig. 22.3 (ii). With the help of range selector switch S,
any shunt can be put in parallel with the galvanometer. When d.c. current is to be measured, the
multimeter switch is turned on to d.c. position. This puts the circuit shown in Fig. 22.3 (ii) in action.
By throwing the range selector switch S to a suitable position, the desired d.c. current can be mea-
sured.
www.JobsCare.info
604  Principles of Electronics
The multimeter can also be used to measure alternating current. For this purpose, a full - wave
rectifier is used as shown in Fig. 22.4. The rectifier converts a.c. into d.c. for application to the
galvanometer. The desired current range can be selected by switch S. By throwing the range selector
switch S to a suitable position, the given a.c. current can be measured. Again, the a.c. current scale is
calibrated in r.m.s. values so that the instrument will give r.m.s. value of alternating current under
measurement.
(iii) Multimeter as ohmmeter. Fig. 22.5 (i) shows the circuit of ohmmeter. The multimeter
employs the internal battery. A fixed resistance R
and a variable resistance r are connected in series
with the battery and galvanometer. The fixed resis-
tance R limits the current within the range desired
and variable resistance r is for zero-adjustment
reading. The resistance to be measured is connected
between terminals A and B. The current flowing
through the circuit will depend upon the value of re-
sistor connected across the terminals. The ohmme-
ter scale is calibrated in terms of ohms. The ohm-
meter is generally made multirange instrument by
using different values of R as shown in Fig. 22.5 (ii).
To use ohmmeter, terminals A and B are shorted
and resistance r is adjusted to give full scale deflec-
tion of the galvanometer. Under this condition, the
resistance under measurement is zero. Because the
needle deflects to full scale, the ohmmeter scale must
then indicate full scale deflection as zero ohm. Then
probes A and B are connected across the resistance
to be measured. If the resistance to be measured is Fig. 22.4
high, lower current flows through the circuit and the
meter will indicate lower reading. It may be mentioned here that each time the ohmmeter is used, it
is first shorted across AB and r is adjusted to zero the meter. This calibrates the meter and accommo-
dates any decrease in the terminal voltage of the battery with age.

Fig. 22.5
Typical multimeter circuit. Fig 22.6 shows a typical multimeter circuit incorporating three
voltage and current ranges.
www.JobsCare.info
Electronic Instruments 605

Fig. 22.6
Here the full-scale deflection (f.s.d.) current of the meter is 100 A and meter resistance is 50 .
The design of this multimeter means finding the values of various resistances.
22.3 Applications of Multimeter
A multimeter is an extremely important elec-
tronic instrument and is extensively used for
carrying out various tests and measurements in
electronic circuits. It is used :
(i) For checking the circuit continuity.
When the multimeter is employed as continu-
ity-checking device, the ohmmeter scale is
utilised and the equipment to be checked is shut
off or disconnected from the power mains.
(ii) For measuring d.c. current flowing
through the cathode, plate, screen and other
vacuum tube circuits.
(iii) For measuring d.c. voltages across Checking the circuit continuity by multimeter
various resistors in electronic circuits.
(iv) For measuring a.c. voltages across power supply transformers.
(v) For ascertaining whether or not open or short circuit exists in the circuit under study.
22.4 Sensitivity of Multimeter
The resistance offered per volt of full scale deflection by the multimeter is known as multimeter
sensitivity.
Multimeter sensitivity indicates the internal resistance of the multimeter. For example, if the
total resistance of the meter is 5000 ohms and the meter is to read 5 volts full scale, then internal
resistance of the meter is 1000 per volt i.e. meter sensitivity is 1000 per volt. Conversely, if the
meter sensitivity is 400 per volt which reads from 0 to 100 V, then meter resistance is 40,000 ohms.
If the meter is to read V volts and Ig is the full scale deflection current, then,
V
Meter resistance =
Ig
Meter sensitivity = Resistance per volt full scale deflection
1
= V V =
Ig Ig
Sensitivity is the most important characteristic of a multimeter. If the sensitivity of a multimeter
is high, it means that it has high internal resistance. When such a meter is connected in the circuit to
606  Principles of Electronics
read voltage, it will draw a very small current. Consequently, there will be no change in the circuit
current due to the introduction of the meter. Hence, it will measure the voltage correctly. On the other
hand, if the sensitivity of multimeter is low, it would cause serious error in voltage measurement. The
sensitivity of multimeters available in the market range from 5 k per volt to 20 k per volt.
22.5 Merits and Demerits of Multimeter
Although multimeter is widely used for manufacturing and servicing of electronics equipment, it has
its own merits and demerits.
Merits
(i) It is a single meter that performs several measuring functions.
(ii) It has a small size and is easily portable.
(iii) It can make measurements with reasonable accuracy.
Demerits
(i) It is a costly instrument. The cost of a multimeter having sensitivity of 20 k per volt is
about Rs. 1000.
(ii) It cannot make precise and accurate measurements due to the loading effect.
(iii) Technical skill is required to handle it.
22.6 Meter Protection
It is important to provide protection for the meter in the event of an accidental overload. This is
achieved by connecting a diode in parallel with the voltmeter as shown in Fig. 22.7.

Fig. 22.7
Let us see how diode across the meter enables it to withstand overload without destroying the
expensive movement. If I is the normal f.s.d. current, a potential difference of IRm is developed
across the diode. The circuit is so designed that IRm does not turn on the diode. In the event of an
accidental overload (say 5 I), the voltage across diode becomes 5 times greater and it is immediately
turned on. Consequently, diode diverts most of the overload current in the same manner as a shunt.
Thus protection of the meter against overload is ensured. Silicon diodes are perhaps the best to use in
such circuits.
Example 22.1. A multimeter has full scale deflection current of 1 mA. Determine its sensitivity.
3
Solution. Full scale deflection current, Ig = 1 mA = 10 A
Multimeter sensitivity = 1/Ig = 1/10 3 = 1000 per volt
Example 22.2. A multimeter has a sensitivity of 1000 per volt and reads 50 V full scale. If the
meter is to be used to measure the voltage across 50000 resistor, will it read correctly ?
Solution. Meter sensitivity = 1000 per volt
Full scale volts = 50 V
Meter resistance = 50 1000 = 50,000
When the meter is used to measure the voltage across the resistance as shown in Fig. 22.8, the
total resistance of the circuit is a parallel combination of two 50,000 resistors. Therefore, the
www.JobsCare.info
Electronic Instruments 607
circuit resistance would be reduced to 25000 and double
the amount of current would be drawn than would other-
wise be the case.
Meter will give highly incorrect reading.
Comments. This example shows the limitation of mul-
timeter. The multimeter will read correctly only if its re-
sistance is very high as compared to the resistance across
which voltage is to be measured.
As a rule, the resistance of the multimeter should be
atleast 100 times the resistance across which voltage is to Fig. 22.8
be measured.
Example 22.3. In the circuit shown in Fig. 22.9 (i), it is desired to measure the voltage across
10 k resistance. If a multimeter of sensitivity 4 k/volt and range 0-10 V is used for the purpose,
what will be the reading ?
Solution. In the circuit shown in Fig. 22.9 (i), the circuit current by Ohms law is 1 mA. There-
fore, voltage across 10 k resistance is 10 V. Let us see whether the given multimeter reads this
value. Fig. 22.9 (ii) shows the multimeter connected across 10 k resistance. The introduction of
multimeter will change the circuit resistance and hence circuit current.
Resistance of meter = 4 k 10 = 40 k
Total circuit resistance = 40 k || 10 k + 10 k
40 10
= + 10 = 8 + 10 = 18 k
40 + 10
20 V
Circuit current = = 1.11 mA
18 k

Fig. 22.9
Voltage read by multimeter = 8 k 1.11 mA = 8.88 V
Example 22.4. If in the above example, a multimeter of sensitivity 20 k per volt is used, what
will be the reading ?
Solution. Meter resistance = 20 k 10 = 200 k
Total circuit resistance = 200 k || 10 k + 10 k
200 10
= + 10 = 9.5 + 10 = 19.5 k
200 + 10
20 V
Circuit current = = 1.04 mA
19.5 k
Voltage read by multimeter = 9.5 k 1.04 mA = 9.88 V
608  Principles of Electronics
A comparison of examples 22.3 and 22.4 shows that a multimeter with higher sensitivity gives
more correct reading.
Example 22.5. In the circuit shown in Fig. 22.10, find the voltage at points A, B, C and D (i)
before the meter is connected and (ii) after the meter is connected. Explain why the meter readings
differ from those without the meter connected.
Solution. (i) When meter is not connected. When meter is not connected in the circuit, the
circuit is a simple series circuit consisting of resistances 20 k, 20 k, 30 k and 30 k.
Total circuit resistance = 20 + 20 + 30 + 30 = 100 k
100 V
Circuit current = = 1 mA
100 k
Voltage at point A = 100 V

Fig. 22.10
Voltage at point B = 100 1 mA 20 k = 80 V
Voltage at point C = 100 1 mA 40 k = 60V
Voltage at point D = 100 1 mA 70 k = 30V
(ii) When meter is connected. When meter is connected in the circuit, the circuit becomes a
series-parallel circuit. The total circuit resistance would depend upon the position of switch S.
(a) When switch is at position A
The voltage at point A is 100 V because point A is directly connected to the voltage source.
Voltage at point A = 100 V
(b) When switch is at position B
80 60
Total circuit resistance = 20 + = 20 + 34.28 = 54.28 k
80 + 60
100 V
Circuit current =
54.28 k
100 V
Voltage at point B = 34.28 k = 63 V
54.28 k
(c) When switch is at point C
60 60
Total circuit resistance = 40 + = 40 + 30 = 70 k
60 + 60
100 V
Circuit current =
70 k
www.JobsCare.info
Electronic Instruments 609
100 V
Voltage at point C = 30 k = 42.8 V
70 k
(d) When switch is at point D
30 60
Total circuit resistance = 70 + = 70 + 20 = 90 k
30 + 60
100 V
Circuit current =
90 k
100 V
Voltage at point D = 20 k = 22.2 V
90 k
Comments. Note that potential measurements are being made in a high-impedance circuit; the
circuit resistance is comparable to meter resistance. As a rule, the resistance of the voltmeter should be
100 times the resistance across which voltage is to be measured. Since such a condition is not realised in
this problem, the meter readings differ appreciably from those without the meter connected.
22.7 Electronic Voltmeters
The electromagnetic and electrostatic voltmeters have two main drawbacks. First, the input resis-
tance/impedance of these instruments is not very high so that there is a considerable *loading effect of
the instrument. Secondly, considerable power is drawn from the circuit under measurement. Both
these drawbacks are overcome in electronic voltmeters. The electronic devices (e.g. vacuum tubes,
transistors etc.) have very high input resistance/impedance and possess the property of amplification.
The latter property permits the input signal to be amplified so that the power to operate the indicating
mechanism comes from a source other than the measured circuit. There are a large number of elec-
tronic voltmeters. However, we shall discuss the following three types of electronic voltmeters :
(i) Vacuum Tube Voltmeter (VTVM)
(ii) Transistor Voltmeter
(iii) Bridge Rectifier Voltmeter

22.8 Vacuum Tube Voltmeter ( VTVM )


A vacuum tube voltmeter consists of any ordinary voltmeter and electron tubes. It is extensively
used for measuring both a.c. and d.c. voltages. The vacuum tube voltmeter has high internal resis-
tance ( > 10 M ) and draws extremely small current from the circuit across which it is connected. In
other words, the loading effect of this instrument is very small. Therefore, a VTVM measures the
exact voltage even across a high resistance. In fact, the ability of VTVM to measure the voltages
accurately has made this instrument the most popular with technicians for trouble shooting radio and
television receivers as well as for laboratory work involving research and design.
(i) Simple VTVM circuit. Fig. 22.11 shows the simple circuit of a vacuum tube voltmeter. It
consists of a triode having meter M connected in the plate circuit. The meter is calibrated in volts. R1
is the grid leak resistor. The voltage to be measured is applied at the grid of triode in such a way that
grid is always negative w.r.t. cathode. This voltage at the grid is transformed by the triode into
corresponding plate current. The meter M connected in the plate circuit directly gives the value of the
voltage under measurement. It may be seen that as grid draws extremely small current (< 1 A),
therefore, internal resistance of VTVM is very large. This circuit has the disadvantage that if the
applied voltages change (especially filament voltage), the plate current will also change. Conse-
quently, the meter will give wrong reading.

* When a voltmeter is connected across a resistance R to measure voltage, the measured voltage will be
less than the actual value. It is because the resistance R is shunted by the voltmeter. This is called
loading effect of the meter. The greater the input resistance of voltmeter, the smaller will be the loading
effect and more accurate is the reading.
www.JobsCare.info
610  Principles of Electronics

Fig. 22.11
(ii) Balanced bridge Type VTVM. The disadvantage of above circuit is overcome in the
balanced bridge type VTVM shown in Fig. 22.12. Here, two similar triodes V1 and V2 are used. The
meter M is connected between the plates of triodes and indicates the voltage to be measured. The
variable resistance r in the plate circuit of V2 is for zero adjustment of the meter. The voltage to be
measured is applied at the grid of triode V1 in such a way that grid is always negative with respect to
cathode.
Operation. When no voltage is applied at the input terminals AB, the plate currents flowing in
both valves are equal as the triodes are similar. Therefore, plates of both valves are at the same
potential. Consequently, the current through the meter M is zero and the meter reads zero volt.
However, in actual practice, there are always some constructional differences in plates, grids and
cathodes of the two valves. The result is that two plate currents differ slightly and the meter may give
some reading. In such a case, the meter needle is brought to zero by changing resistance r.

Fig. 22.12
The voltage to be measured is applied at the grid of triode V1, making the grid negative w.r.t.
cathode. This changes the plate current of triode V1 and the plates of two valves no longer remain at
the same potential. Therefore, a small current flows through the meter M which directly gives the
value of the voltage being measured. It may be noted that actually triode V1 is used for voltage
measurement, the purpose of V2 is simply to prevent zero drift. By using two similar tubes, any
Electronic Instruments 611
change in plate current due to supply fluctuations will equally affect the two plate currents. There-
fore, net change in potential drop across voltmeter is zero.

Fig. 22.13
Range selection. In practice, a VTVM is made
a multirange instrument by employing a potentiometer
at the input circuit as shown in Fig. 22.13. By throwing
the range selector switch S to a suitable position, the
desired voltage range can be obtained. Thus when the
range selector switch S is thrown to position 1, the volt-
age applied to the grid is three times as compared to
position 3. Although only three voltage ranges have
been considered, a commercial VTVM may have more
ranges.
22.9 Applications of VTVM
A VTVM is far superior to a multimeter and performs a
number of measuring functions. A few important appli-
VTVM cations of VTVM are discussed below :

Fig. 22.14
www.JobsCare.info
612  Principles of Electronics
(i) d.c. voltage measurements. A VTVM can accurately measure the d.c. voltages in an elec-
tronic circuit. The d.c. voltage to be measured is applied at the input (i.e. grid of V1) terminals in such
a way that grid of the input valve V1 is always negative. Fig. 22.14 shows the circuit of an amplifier
stage and measurement of d.c. voltage across cathode resistor RK.
(ii) d.c. current measurements. A conventional VTVM does not incorporate a current scale.
However, current values can be found indirectly. For instance, in Fig. 22.14, the d.c. current through
RK can be found by noting the voltage across RK and dividing it by the resistance RK.
(iii) a.c. voltage measurements. For measuring a.c. voltage, a rectifier is used in conjunction
with a VTVM. The rectifier converts a.c. into d.c. for application to the grid of valve V1. In fact,
rectifier circuit is a part of VTVM. Fig. 22.15 shows the transistor power amplifier stage and mea-
surement of a.c. voltage across the speaker.

Fig. 22.15
(iv) Resistance measurements. A VTVM can be used to measure resistances and has the ability
to measure resistances upto 1000 megaohms whereas the ordinary ohmmeter will measure only upto
about 10 megaohms. Fig. 22.16 shows the circuit of VTVM ohmmeter. By throwing the selector
switch S to any suitable position, the desired resistance range can be obtained. The unknown resistor
whose value is to be measured is connected between points A and B. If the unknown resistance has
high value, a higher negative bias will be applied to triode V1. Reverse will happen if the unknown

Fig. 22.16
www.JobsCare.info
Electronic Instruments 613
resistance has low value. The imbalance in the plate currents of the two valves will cause a current
through the meter M which will directly give the value over the resistance scale of the meter.

22.10 Merits and Demerits of VTVM


A VTVM is an extremely important electronic equipment and is widely used for making differ-
ent measurements in electronic circuits.
Merits
(i) A VTVM draws extremely small current from the measuring circuit. Therefore, it gives
accurate readings.
(ii) There is little effect of temperature variations.
(iii) Because a VTVM uses triodes, the voltage to be measured is amplified. This permits the use
of less sensitive meter.
(iv) It has a wide frequency response.
Demerits
(i) It cannot make current measurements directly.
(ii) Accurate readings can be obtained only for sine waves.

22.11 Transistor Voltmeter Circuit


Since vacuum tubes have become obsolete, these have been replaced by transistors and other semi-
conductor devices. Fig. 22.17 shows the circuit of an emitter-follower voltmeter. The voltage E to be
measured is applied between base and emitter. A permanent-magnet moving coil (PMMC) instrument
and a multiplier resistor RS are connected in series with the transistor emitter. The circuit measures the
voltage quite accurately because the emitter follower offers high input resistance to the voltage being
measured and provides a low output resistance to drive current through the coil of PMMC meter.

Fig. 22.17
Operation. The voltage E to be measured is applied between base and emitter of the transistor
and causes a base current IB to flow through the base circuit. Therefore, collector current IC = IB
where is the current amplification factor of the transistor. Since IE M IC and the meter is connected
www.JobsCare.info
614  Principles of Electronics
in the emitter, the meter current Im = IE = IB. Now the meter current Im depends upon the input
voltage to be measured. Therefore, the PMMC meter can be calibrated to read the input voltage
directly.
Emitter voltage, VE = E VBE
VE
Meter current, Im = R + R
S m
Here RS = multiplier resistor ; Rm = meter resistance
E
Input resistance of voltmeter, Ri =
IB
Example 22.6. The emitter follower circuit shown in Fig. 22.17 has VCC = 12 V ; Rm = 1kW
and a 2 mA meter. If transistor = 80, calculate (i) the suitable resistance for RS to give full -scale
deflection when E = 5V (ii) the voltmeter input resistance.
Solution.
Meter resistance, Rm = 1 k
3
F.S.D. current of meter,Im (f.s.d.) = 2 mA = 2 10 A
(i) Emitter voltage, VE = E VBE = 5 V 0.7 V = 4.3 V
VE
Im (f.s.d.) =
RS + Rm
3 4.3V
or 2 10 = RS = 1150
1000 + RS
I m ( f .s.d .) 2 mA
(ii) Base current, IB = = = 0.025 mA
80
E = 5V
Input resistance of voltmeter, Ri = = 200 k
I B 0.025 mA
Example 22.7. The emitter-follower voltmeter circuit in Fig. 22.17 has VCC = 20 V, RS + Rm =
9.3 k, Im = 1 mA and transistor = 100.
(i) Calculate the meter current when E = 10V.
(ii) Determine the voltmeter input resistance with and without the transistor.
Solution.
(i) Emitter voltage, VE = E VBE = 10V 0.7 V = 9.3 V
VE 9.3 V
Meter current, Im = = = 1 mA
RS + Rm 9.3 k
I m 1 mA
(ii) Base current, IB = = = 0.01 mA
100
E 10 V
With transistor, Ri = I = 0.01 mA = 1000 k = 1 M
B
Without transistor, Ri = RS + Rm = 9.3 k
Note that without transistor, the voltmeter input resistance = RS + Rm = 9.3 k. However, with
transistor, the voltmeter input resistance = 1 M = 1000 k. The obvious advantage of the electronic
voltmeter is that its loading effect in voltage measurement will be very small.
Example 22.8. In the above example, if E = 5V, all other values remaining the same, what will
be the value of meter current? Comment on the result.
Solution.
E VBE 5V 0.7V
Meter current, Im = = = 4.3V = 0.46 mA
RS + Rm 9.3 k 9.3 k
www.JobsCare.info
Electronic Instruments 615
With E = 5V, the meter should read half of full-scale reading i.e. 0.5 mA. However, the meter current
is actually 0.46 mA. This error is due to VBE and can be eliminated by the modification of the circuit.
22.12 Bridge Rectifier Voltmeter
A permanent-magnet moving coil (PMMC) instrument responds to average or d.c. value of current
through the moving coil. If alternating current is passed through the moving coil, the driving torque
would be *zero. It is because the average value of a sine wave over one cycle is zero. Therefore, a
PMMC instrument connected directly to measure a.c. indicates zero reading. In order to measure a.c.
with a PMMC instrument, the given a.c. is converted into d.c. by using a bridge rectifier. The instru-
ment is then called rectifier type instrument.
Circuit details. Fig. 22.18 shows bridge rectifier voltmeter for the measurement of a.c. volt-
ages. A multiplier resistor RS is connected in **series with the PMMC instrument having resistance
Rm. When a.c. voltage to be measured is applied to the circuit, full-wave rectification will be obtained
as shown in Fig. 22.19. The meter deflection will be proportional to the average current. Since there
is a definite relationship between the average value and r.m.s. value of a sine-wave (r.m.s. value =
1.11 average value), the meter scale can be calibrated to read the r.m.s. value directly.

Fig. 22.18 Fig. 22.19


Operation. When a.c. voltage to be measured is applied to the circuit, it passes the positive half-
cycles of the input and inverts the negative half-cycles.
(i) During the positive half-cycle of the a.c. input voltage, point A is positive w.r.t. point B.
Therefore, diodes D1 and D4 are forward biased while diodes D2 and D3 are reverse biased. As a
result, diodes D1 and D4 conduct and the current follows the path RS D1 PMMC meter D4 back
to point B. Note that multiplier resistor RS and the meter are in series.
(ii) During the negative half-cycle of the a.c. input voltage, diodes D2 and D3 are forward biased
while diodes D1 and D4 are reverse biased. As a result, diodes D2 and D3 conduct and the current
follows path D3 PMMC meter D2 RS back to point A. Note that current through the meter is in
the same direction as for the positive half-cycle. Consequently, full-wave rectification results.

* The driving torque would be in one direction for the positive half-cycle and in the other direction for the
negative half-cycle. The inertia of the coil is so great that at supply frequency (50 Hz),the pointer cannot
follow the rapid reversals of the driving torque. Therefore, pointer of the meter remians stationary at zero
mark.
** If you see carefully, the four diodes (D1, D2, D3 and D4) form the bridge. Note that the same current
flows through the RS and Rm.
Note that a voltmeter is a current-operated device.
www.JobsCare.info
616  Principles of Electronics
The scale of the PMMC meter is calibrated to read directly the r.m.s. value of a.c. voltage being
measured. It may be noted that rectifier voltmeter can be used only to measure pure sine-wave
voltages. When other than pure sine-waves are applied, the meter will not indicate the r.m.s. voltage.
Example 22.9. A PMMC instrument with a full-scale deflection (f.s.d.) current of 100 A and
Rm = 1 k is to be used as a voltmeter of range 0 100 V (r.m.s.). The diodes used in the bridge
rectifier circuit are of silicon. Calculate the value of multiplier resistor RS required.
Solution. Note that 100 A is the average current.
6 4
F.S.D. current of meter, Im (f.s.d.) = 100 A = 100 10 A = 10 A
Total circuit resistance, RT = RS + Rm = (RS + 1000)
Peak value of applied voltage, Vm = 2 Vr.m.s. =
2 100 V = 141. 4V
* Total rectifier drop = 2 VF = 2 0.7 = 1.4V
Peak applied voltage Rectifier Drop
Peak f.s.d. current of meter =
Total circuit resistance
4 141.4 1.4 I av.
10
or
0.637
=
RS + 1000 I peak = 0.637


RS = 890.7 k
Example 22.10. An a.c. voltmeter uses a bridge rectifier with silicon diodes and a PMMC
instrument with f.s.d. current of 75 A. If meter coil resistance is 900 and the multiplier resistor is
708 k, calculate the applied r.m.s. voltage when the meter reads f.s.d.
Solution. The PMMC meter reads average value.
6
75 10 I av.
Peak f.s.d. meter current =
0.637 I peak =
A
0.637
Peak applied voltage Rectifier drop
Now Peak f.s.d. meter current =
Total circuit resistance
6 2 Vr.m.s. 2 0.7
75 10
or =
0.637 RS + Rm
6 1.414 Vr.m.s 1.4
or 117.74 10 = 3
708 10 + 900
6 3
(117.74 10 ) (708 10 + 900) + 1.4
Vr.m.s. = = 60V
1.414
22.13 Cathode Ray Oscilloscope
The cathode ray oscilloscope (commonly abbreviated as CRO) is an electronic device which is capable
of giving a visual indication of a signal waveform. No other instrument used in the electronic industry
is as versatile as the cathode ray oscilloscope. It is widely used for trouble shooting radio and television
receivers as well as for laboratory work involving research and design. With an oscilloscope, the wave-
shape of a signal can be studied with respect to amplitude distortion and deviation from the normal. In
addition, the oscilloscope can also be used for measuring voltage, frequency and phase shift.
In an oscilloscope, the electrons are emitted from a cathode accelerated to a high velocity and
brought to focus on a fluorescent screen. The screen produces a visible spot where the electron beam
strikes. By deflecting the electron beam over the screen in response to the electrical signal, the
electrons can be made to act as an electrical pencil of light which produces a spot of light wherever it
* During positive or negative half-cycle of input a.c. voltage, two diodes (D1 and D4 or D2 and D3) are in
series.
Electronic Instruments 617
strikes. An oscilloscope obtains its remarkable properties as a measuring instrument from the fact
that it uses as an indicating needle a beam of electrons. As electrons have negligible mass, therefore,
they respond almost instantaneously when acted upon by an electrical signal and can trace almost any
electrical variation no matter how rapid. A cathode ray oscilloscope contains a cathode ray tube and
necessary power equipment to make it operate.
22.14 Cathode Ray Tube
A cathode ray tube (commonly abbreviated as CRT) is the heart of the oscilloscope. It is a vacuum
tube of special geometrical shape and converts an electrical signal into visual one. A cathode ray tube
makes available plenty of electrons. These electrons are accelerated to high velocity and are brought
to focus on a fluorescent screen. The electron beam produces a spot of light wherever it strikes. The
electron beam is deflected on its journey in response to the electrical signal under study. The result is that
electrical signal waveform is displayed visually. Fig. 22.20 shows the various parts of cathode ray tube.

Fig. 22.20
(i) Glass envelope. It is conical highly evacuated glass housing and maintains vacuum inside
and supports the various electrodes. The inner walls of CRT between neck and screen are usually
coated with a conducting material, called aquadag. This coating is electrically connected to the
accelerating anode so that electrons which accidentally strike the walls are returned to the anode.
This prevents the walls of the tube from charging to a high negative potential.
(ii) Electron gun assembly. The arrangement of electrodes which produce a focussed beam of
electrons is called the electron gun. It essentially consists of an indirectly heated cathode, a control
grid, a focussing anode and an accelerating anode. The control grid is held at negative potential
w.r.t. cathode whereas the two anodes are maintained at high positive potential w.r.t. cathode.
The cathode consists of a nickel cylinder coated with oxide coating and provides plenty of
electrons. The control grid encloses the cathode and consists of a metal cylinder with a tiny circular
opening to keep the electron beam small in size. The focussing anode focuses the electron beam into
a sharp pin-point by controlling the positive potential on it. The positive potential (about 10,000 V)
on the accelerating anode is much higher than on the focusing anode. For this reason, this anode
accelerates the narrow beam to a high velocity. Therefore, the electron gun assembly forms a narrow,
accelerated beam of electrons which produces a spot of light when it strikes the screen.
(iii) Deflection plate assembly. The deflection of the beam is accomplished by two sets of
deflecting plates placed within the tube beyond the accelerating anode as shown in Fig. 22.20. One
set is the vertical deflection plates and the other set is the horizontal deflection plates.
The vertical deflection plates are mounted horizontally in the tube. By applying proper poten-
tial to these plates, the electron beam can be made to move up and down vertically on the fluorescent
www.JobsCare.info
618  Principles of Electronics
screen. The horizontal deflection plates are mounted in the vertical plane. An appropriate potential
on these plates can cause the electron beam to move right and left horizontally on the screen.
(iv) Screen. The screen is the inside face of the tube and is coated with some fluorescent
material such as zinc orthosilicate, zinc oxide etc. When high velocity electron beam strikes the
screen, a spot of light is produced at the point of impact. The colour of the spot depends upon the
nature of fluorescent material. If zinc orthosilicate is used as the fluorescent material, green light spot
is produced.
Action of CRT. When the cathode is heated, it emits plenty of electrons. These electrons pass
through control grid on their way to screen. The control grid influences the amount of current flow as
in standard vacuum tubes. If negative potential on the control grid is high, fewer electrons will pass
through it and the electron beam on striking the screen will produce a dim spot of light. Reverse will
happen if the negative potential on the control grid is reduced. Thus, the intensity of light spot on the
screen can be changed by changing the negative potential on the control grid. As the electron beam
leaves the control grid, it comes under the influence of focussing and accelerating anodes. As the two
anodes are maintained at high positive potential, therefore, they
produce a field which acts as an electrostatic lens to converge
the electron beam at a point on the screen.
As the electron beam leaves the accelerating anode, it
comes under the influence of vertical and horizontal deflection
plates. If no voltage is applied to the deflection plates, the
electron beam will produce spot of light at the centre (point O
in Fig. 22.21) of the screen. If the voltage is applied to vertical Fig. 22.21
plates only as shown in Fig. 22.21, the electron beam and hence
the spot of light will be deflected upwards (point O1). The spot of light will be deflected downwards
(point O2) if the potential on the plates is reversed. Similarly, the spot of light can be moved horizontally
by applying voltage across the horizontal plates.
22.15 Deflection Sensitivity of CRT
The shift of the spot of light on the screen per unit change in voltage across the deflection plates is
known as deflection sensitivity of CRT. For instance, if a voltage of 100 V applied to the vertical
plates produces a vertical shift of 3 mm in the spot, then deflection sensitivity is 0.03 mm/V. In general,
Spot deflection = Deflection sensitivity Applied voltage
The deflection sensitivity depends not only on the design of the tube but also on the voltage
applied to the accelerating anode. The deflection sensitivity is low at high accelerating voltages and
vice-versa.
Example 22.11. The deflection sensitivity of a CRT is 0.01 mm/V. Find the shift produced in the
spot when 400 V are applied to the vertical plates.
Solution. As voltage is applied to the vertical plates only, therefore, the spot will be shifted
vertically.
Spot shift = deflection sensitivity applied voltage
= 0.01 400 = 4 mm
Example 22.12. The deflection sensitivity of a CRT is 0.03 mm/V. If an unknown voltage is ap-
plied to the horizontal plates, the spot shifts 3 mm horizontally. Find the value of unknown voltage.
Solution. Deflection sensitivity = 0.03 mm/V
Spot shift = 3 mm
Now, spot shift = deflection sensitivity applied voltage
www.JobsCare.info
Electronic Instruments 619
spot shift 3 mm
Applied voltage = = = 100 V
deflection sensitivity 0.03 mm/V

22.16 Applying Signal Across Vertical Plates


If a sinusoidal voltage is applied to the vertical deflection plates, it will make the plates alternately
positive and negative. Thus, in the positive half of the signal, upper plate will be positive and lower
plate negative while in the negative half-cycle, the plate polarities will be reversed.

Fig. 22.22

The result is that the spot moves up and down at the same rate as the frequency of the applied
voltage. As the frequency of applied voltage is 50 Hz, therefore, due to persistence of vision, we will
see a continuous vertical line 2 - 1 - 4 on the screen as shown in Fig. 22.22 (iii). The line gives no
indication of the manner in which the voltage is alternating since it does not reveal the waveform.
22.17 Display of Signal Waveform on CRO
One interesting application of CRO is to present the wave shape of the signal on the screen. As
discussed before, if sinusoidal signal is applied to the vertical deflection plates, we get a vertical line.
However, it is desired to see the signal voltage variations with time on the screen. This is possible
only if we could also move the beam horizontally from left to right at a uniform speed while it is
moving up and down. Further, as soon as a full cycle of the signal is traced, the beam should return
quickly to the left hand side of the screen so that it can start tracing the second cycle.

Fig. 22.23
In order that the beam moves from left to right at a uniform rate, a voltage that varies linearly
with time should be applied to the horizontal plates. This condition is exactly met in the saw tooth
wave shown in Fig. 22.23 (i).
When time t = 0, the negative voltage on the horizontal plates keep the beam to the extreme left
on the screen as shown in Fig. 22.23 (ii). As the time progresses, the negative voltage decreases
linearly with time and the beam moves towards right forming a horizontal line. In this way, the saw-
tooth wave applied to horizontal plates moves the beam from left to right at a uniform rate.
www.JobsCare.info
620  Principles of Electronics
22.18 Signal Pattern on Screen
If the signal voltage is applied to the vertical plates and saw-tooth wave to the horizontal plates, we
get the exact pattern of the signal as shown in Fig. 22.24. When the signal is at the instant 1, its
amplitude is zero. But at this instant, maximum negative voltage is applied to horizontal plates. The
result is that the beam is at the extreme left on the screen as shown. When the signal is at the instant
2, its amplitude is maximum. However, the negative voltage on the horizontal plates is decreased.
Therefore, the beam is deflected upwards by the signal and towards the right by the saw tooth wave.
The result is that the beam now strikes the screen at point 2. On similar reasoning, the beam strikes
the screen at points 3, 4 and 5. In this way, we have the exact signal pattern on the screen.

Fig. 22.24

22.19 Various Controls of CRO


In order to facilitate the proper functioning of CRO, various controls are provided on the face of
CRO. A few of them are given below:
(i) Intensity control. The knob of intensity control regulates the bias on the control grid and
affects the electron beam intensity. If the negative bias on the grid is increased, the intensity of elec-
tron beam is decreased, thus reducing the brightness of the spot.
(ii) Focus control. The knob of focus control regulates the positive potential on the focussing
anode. If the positive potential on this anode is increased, the electron beam becomes quite narrow
and the spot on the screen is a pin-point.
(iii) Horizontal position control. The knob of horizontal position control regulates the ampli-
tude of d.c. potential which is applied to the horizontal deflection plates, in addition to the usual
saw-tooth wave. By adjusting this control, the spot can be moved to right or left as required.
(iv) Vertical position control. The knob of vertical position control regulates the amplitude of
d.c. potential which is applied to the vertical deflection plates in addition to the signal. By adjusting
this control, the image can be moved up or down as required.
22.20 Applications of CRO
The modern cathode ray oscilloscope provides a powerful tool for solving problems in electrical
measurements. Some important applications of CRO are :
1. Examination of waveforms
2. Voltage measurement
3. Frequency measurement
www.JobsCare.info
Electronic Instruments 621

Fig. 22.25
1. Examination of waveform. One of the important uses of CRO is to observe the wave
shapes of voltages in various types of electronic circuits. For this purpose, the signal under study is
applied to vertical input (i.e., vertical deflection plates) terminals of the oscilloscope. The sweep
circuit is set to internal so that sawtooth wave is applied to the horizontal input i.e. horizontal deflec-
tion plates. Then various controls are adjusted to obtain sharp and well defined signal waveform on
the screen.
Fig. 22.25 shows the circuit for studying the performance of an audio amplifier. With the help of
switch S, the output and input of amplifier is applied in turn to the vertical input terminals. If the
waveforms are identical in shape, the fidelity of the amplifier is excellent.
2. Voltage measurement. As discussed before, if the signal is applied to the vertical deflec-
tion plates only, a vertical line appears on the screen. The height of the line is proportional to peak-
to-peak voltage of the applied signal. The following procedure is adopted for measuring voltages
with CRO.
(i) Shut off the internal horizontal sweep generator.
(ii) Attach a transparent plastic screen to the face of oscilloscope. Mark off the screen with
vertical and horizontal lines in the form of graph.
(iii) Now, calibrate the oscilloscope against a known voltage. Apply the known voltage, say
10 V, to the vertical input terminals of the oscilloscope. Since the sweep circuit is shut off, you will get
a vertical line. Adjust the vertical gain till a good deflection is obtained. Let the deflection sensitivity
be V volts/mm.
(iv) Keeping the vertical gain unchanged, apply the unknown voltage to be measured to the
vertical input terminals of CRO.
(v) Measure the length of the vertical line obtained. Let it be l mm.
Then, Unknown voltage = l V volts
3. Frequency measurement. The unknown frequency can be accurately determined with the
help of a CRO. The steps of the procedure are as under :
(i) A known frequency is applied to horizontal input and unknown frequency to the vertical
input.
(ii) The various controls are adjusted.
(iii) A pattern with loops is obtained.
www.JobsCare.info
622  Principles of Electronics
(iv) The number of loops cut by the horizontal line gives the frequency on the vertical plates (fv)
and the number of loops cut by the vertical line gives the frequency on the horizontal plates (fH).
fv No. of loops cut by horizontal line
=
fH No. of loops cut by vertical line
For instance, suppose during the frequency measurement test, a pattern shown in Fig. 22.26 is
obtained. Let us further assume that frequency applied to horizontal plates is 2000 Hz. If we draw
horizontal and vertical lines, we find that one loop is cut by the horizontal line and two loops by the
vertical line. Therefore,
fv No. of loops cut by horizontal line
=
fH No. of loops cut by vertical line
fv 1
or =
2000 2
or fv = 2000 1/2 = 1000 Hz
i.e. Unknown frequency is 1000 Hz.
Example 22.13. In an oscilloscope, 200 V, 50 Hz signal produces a
deflection of 2 cm corresponding to a certain setting of vertical gain
control. If another voltage produces 3 cm deflection, what is the value
of this voltage ? Fig. 22.26
Solution. Deflection sensitivity = 200 V/2 cm = 100 V/cm
Unknown voltage = D. S. deflection = 100 3 = 300 V
Example 22.14. When signals of different frequencies were applied to the vertical input termi-
nals of oscilloscope, the patterns shown in Fig. 22.27 were obtained. If the frequency applied to
horizontal plates in each case is 1000Hz, determine the unknown frequency.

Fig. 22.27
Solution.
(i) The number of loops cut by horizontal and vertical line is one.
fv 1
= or fv = fH = 1000 Hz
fH 1
(ii) The number of loops cut by horizontal line is 2 and the number of loops cut by vertical line
is 1.
fv 2
= or fv = 2 fH = 2 1000 = 2000 Hz
fH 1
(iii) The number of loops cut by the horizontal line is 6 and that by vertical line is 1.
fv 6
=
fH 1
or fv = 6 fH = 6 1000 = 6000 Hz
www.JobsCare.info
Electronic Instruments 623
MULTIPLE-CHOICE QUESTIONS
1. An ammeter is connected in ............... with 10. The input resistance of a VTVM is about
the circuit element whose current we wish ...............
to measure. (i) 1000 (ii) 10 k
(i) series (iii) 20 k (iv) 10 M
(ii) parallel 11. If the negative potential on the control grid
(iii) series or parallel of CRT is increased, the intensity of spot
(iv) none of the above ...............
2. A galvanometer in series with a high resis- (i) is increased
tance is called .............................. (ii) is decreased
(i) an ammeter (ii) a voltmeter (iii) remains the same
(iii) a wattmeter (iv) none of the above (iv) none of the above
3. An ammeter should have ............... resis- 12. For display of signal pattern ............... volt-
tance. age is applied to the horizontal plates of a
(i) infinite (ii) very large CRO.
(iii) very low (iv) none of the above (i) sinusoidal (ii) rectangular
4. A voltmeter is connected in ............... with (iii) sawtooth (iv) none of the above
the circuit component across which poten- 13. Two multimeters A and B have sensitivities
tial difference is to be measured. of 10 k/V and 30 k/V respectively. Then
(i) parallel ...............
(ii) series (i) multimeter A is more sensitive
(iii) series or parallel (ii) multimeter B is more sensitive
(iv) none of the above (iii) both are equally sensitive
5. A voltmeter should have .............. resistance. (iv) none of the above
(i) zero (ii) very high 14. A galvanometer of resistance G is shunted
(iii) very low (iv) none of the above by a very small resistance S. The resistance
of the resulting ammeter is ...............
6. The sensitivity of a multimeter is given in
............... (i) GS (ii) G + S
G+S
(i) (ii) amperes
(iii) G S (iv) none of the above
(iii) k/V (iv) none of the above
15. A VTVM is never used to measure ...............
7. If the full-scale deflection current of a mul-
timeter is 50 A, its sensitivity is ............... (i) voltage (ii) current
(i) 10 k/V (ii) 100 k/V (iii) resistance (iv) none of the above
(iii) 50 k/V (iv) 20 k/V 16. The sensitivity of a voltmeter which uses a
100 A meter movement is ...............
8. If a multimeter has a sensitivity of 1000
per volt and reads 50 V full scale, its inter- (i) 1 k/V (ii) 10 k/V
nal resistance is............... (iii) 5 k/V (iv) data insufficient
(i) 20 k (ii) 50 k 17. What is the total resistance of a voltmeter
(iii) 10 k (iv) none of the above on the 10 V range when the meter move-
ment is rated for 50 A of full-scale current ?
9. A VTVM has ............... input resistance than
that of a multimeter. (i) 10 k (ii) 20 k
(i) more (ii) less (iii) 200 k (iv) none of the above
(iii) same (iv) none of the above 18. The material used to coat inside face of CRT
www.JobsCare.info
624  Principles of Electronics
is ............... 25. Which of the following is likely to have the
(i) carbon (ii) sulphur largest resistance ?
(iii) silicon (iv) phosphorus (i) voltmeter of range 10 V
19. When an ammeter is inserted in the circuit, (ii) moving coil galvanometer
the circuit current will ............... (iii) ammeter of range 1 A
(i) increase (iv) a copper wire of length 1 m and diam-
(ii) decrease eter 3 mm
(iii) remain the same 26. An ideal ammeter has ............... resistance.
(iv) none of the above (i) low (ii) infinite
20. A series ohmmeter circuit uses a 3 V battery (iii) zero (iv) high
and a 1 mA meter movement. What is the 27. The resistance of an ideal voltmeter is .........
half-scale resistance for this movement ? (i) low (ii) infinite
(i) 3 k (ii) 1.5 k (iii) zero (iv) high
(iii) 4.5 k (iv) 6 k 28. To send 10% of the main current through a
21. The most accurate device for measuring volt- moving coil galvanometer of resistance
age is ............... 99 , the shunt required is ...............
(i) voltmeter (ii) multimeter (i) 11 (ii) 9.9
(iii) CRO (iv) VTVM (iii) 100 (iv) 9
22. The horizontal plates of a CRO are supplied 29. A voltmeter has a resistance of G ohms and
with ............... to observe the waveform of a range V volts. The value of resistance re-
signal. quired in series to convert it into voltmeter
(i) sinusoidal wave of range nV is .....................
(ii) cosine wave G
(i) nG (ii)
(iii) sawtooth wave n
G
(iv) none of the above (iii) n 1 (iv) (n 1) G
23. A CRO is used to measure ...............
30. An ammeter has a resistance of G ohms and
(i) voltage (ii) frequency range of I amperes. The value of resistance
(iii) phase (iv) all of above required in parallel to convert it into an am-
24. If 2 % of the main current is to be passed meter of range nI is ....................
through a galvanometer of resistance G, then (i) nG (ii) (n 1) G
resistance of the shunt required is ............. G G
(i) G/50 (ii) G/49 (iii) n 1 (iv)
n
(iii) 49 G (iv) 50 G

Answers to Multiple-Choice Questions


1. (i) 2. (ii) 3. (iii) 4. (i) 5. (ii)
6. (iii) 7. (iv) 8. (ii) 9. (i) 10. (iv)
11. (ii) 12. (iii) 13. (ii) 14. (i) 15. (ii)
16. (ii) 17. (iii) 18. (iv) 19. (ii) 20. (i)
21. (iii) 22. (iii) 23. (iv) 24. (ii) 25. (i)
26. (iii) 27. (ii) 28. (i) 29. (iv) 30. (iii)
www.JobsCare.info
Electronic Instruments 625
Chapter Review Topics
1. What is a multimeter ? How does it work ?
2. What type of measurements can be made with a multimeter ? Explain with suitable diagrams.
3. Briefly explain the advantages of 20 k/volt multimeter as compared to a 10 k/volt multimeter.
4. What are the applications of a multimeter ?
5. Discuss the advantages and disadvantages of a multimeter.
6. What is a VTVM ? Explain balanced bridge Type VTVM with a neat circuit diagram.
7. What are the applications of VTVM ?
8. Discuss the advantages and disadvantages of VTVM.
9. Briefly explain the differences between a VTVM and a multimeter.
10. Explain the construction and working of a cathode ray tube.
11. How will you make the following measurements with a CRO :
(i) voltage (ii) frequency ?
12. Write short notes on the following :
(i) Limitations of multimeter
(ii) Advantages of oscilloscope
(iii) Vacuum tube voltmeter
(iv) Oscilloscope controls

Problems
1. A voltmeter is used to measure voltage across 20 k resistor as shown in Fig. 22.28. What will be the
voltage value if (i) voltmeter has infinite resistance (ii) voltmeter has a sensitivity of 1000 per volt
and reads 100 V full scale ? [(i) 50 V (ii) 45 V]

Fig. 22.28 Fig. 22.29


2. The three range voltmeter is arranged as shown in Fig. 22.29. The ranges are 0 to 3 V, 0 to 15 V and
0 to 50 V as marked. If the full scale deflection current is 10 mA, what should be the values of R1, R2
and R3 ? The resistance of the meter is 5 . [305 , 1505 , 5005 ]
3. If the sensitivity of voltmeter in Fig. 22.28 is 500 /volt (Full-scale reading being 100 V), what will
be the reading of the voltmeter ? [41.7 V]
4. What is the lowest full-scale voltage that could be displayed with a 100 A meter movement with an
internal resistance of 150 ? What would be the sensitivity of this meter in ohms per volt ?
[15 mV, 10,000 /V]
5. If a 20,000 /V meter with 5 k internal resistance is used in an ohmmeter with a 3-V-battery, what
internal resistance is required in the meter to produce proper zeroing? [60 k ]
6. A PMMC instrument with f.s.d. = 100 A and Rm = 1 k is to be used as an a.c. voltmeter with f.s.d.
= 100 V (r.m.s.) as shown in Fig. 22.18. Silicon diodes are used in the bridge rectifier circuit. Calcu-
late the pointer indications for the voltmeter when the r.m.s. input voltage is (i) 75 V (ii) 50V.
[0.75 f.s.d. ; 0.5 f.s.d.]
7. In the above example, calculate the voltmeter sensitivity. /V]
[9 k
www.JobsCare.info
626  Principles of Electronics

Discussion Questions
1. Why is sensitivity of best multimeter not more than 20 k per volt ?
2. Why do we generally prefer VTVM to multimeter for measurements in electronic circuits ?
3. Why does oscilloscope give more accurate measurements than a VTVM ?
4. What is the basic difference between vacuum tubes and cathode ray tube ?
5. How can a multimeter be used for continuity checking ?
6. Which would usually have more linear scales, dc or ac meters ?
7. Which is more sensitive, a 0 59 A or a 0 1 mA meter ?
8. On a multirange ohmmeter, where is 0 mark ?
9. What component prevents meter damage in a VTVM?
10. Could a 0 1 mA-movement 100 V voltmeter and a 0 50 A movement 100 V voltmeter be
used in series across 125 V ?
CONTENTS
CONTENTS

Learning Objectives
+ 0 ) 2 6 - 4
%!
Introduction

Analog and Digital Instruments


Functions of Instruments
ELECTRONIC
Electronic versus Electrical

Instruments
Essentials of an Electronic
INSTRUMENTS
Instrument
The Basic Meter Movement
Characteristics of Moving Coil
Meter Movement
Variation of Basic Meter
Movement
Converting Basic Meter to DC
Ammeter
Multirange Meter
Measurement of Current
Loading Effect of a Voltmeter
Ohmmeter
The Multimeter
Rectifier Type AC Meter
Electronic Voltmeters
The Digital Voltmeter (DVM)
Cathode Ray Tube (CRT)
Normal Operation of a CRO
Dual Trace CRODual Beam
CRO
Lissajous Figures
Applications of a CRO
The Q Meter
Logic Analysers

Signal Generators
Audio Generators
The Digital Voltmeter
Pulse Generators
RF Generators
Frequency Synthesizer
IEEE-488 General Purpose
Interface Bus (GPIB) Instruments

CONTENTS
CONTENTS
2696 Electrical Technology

73.1. Introduction
Electronic instrumentation is such an interesting field that it combines elements of technologies
ranging from the nineteenth to the twenty first centuries. Modern computer-based instrumentation is
now evident in every reasonably equipped laboratory and workshop and in catalogs and advertise-
ments of all of the manufacturers. Yet at the root of many space-age instruments is circuitry, such as
the wheatstone bridge that is found in nineteenth-century textbooks. Although newer techniques are
still in widespread use in new as well as old instruments. In this chapter on electronic instruments you
will find both types discussed.
The scientific and technological progress of any nation depends on its ability to measure, calcu-
late and finally, estimate the unknown. Also, the success of an engineer or technician is judged by his
ability to measure precisely and to correctly interpret the circuit performance. There are three ways
of making such measurements :
(a) by mechanical meanslike measuring gas pressure by Bourdon pressure gauge.
(b) by electrical meanslike measuring potential difference with an electrical voltmeter.
(c) by electronic meanswhich is a very sensitive way of detecting the measured quantity
because of amplification provided by the active electron device.
The electronic instruments generally have higher sensitivity, faster response and greater flexibil-
ity than mechanical or electrical instruments in indicating, recording and, where required, in control-
ling the measured quantity.

73.2. Analog and Digital Instruments


The deflection type instruments with a scale and movable pointer are called analog instruments.
The deflectiion of the pointer is a function of (and, hence, analogous to) the value of the electrical
quantity being measured.
Digital instruments are those which use logic circuits and techniques to obtain a measurement
and then display it in numerical-reading (digital) form. The digital readouts employ either LED
displays or liquid crystal displays (LCD).
Some of the advantages of digital instruments over analog instruments are as under :
1. easy readability
2. greater accuracy
3. better resolution
4. automatic polarity and zeroing

73.3. Functions of Instruments


Functionally, different instruments may be divided into the following three categories :
1. Indicating instruments
These are the instruments which indicate the instantaneous value of quantity being measured, at
the time it is being measured. The indication is in the form of pointer deflection (analog instruments)
or digital readout (digital instruments). Ammeters and voltmeters are examples of such instruments.
2. Recording instruments
Such instruments provide a graphic record of the variations in the quantity being measured over
a selected period of time. Many of these instruments are electromechanical devices which use paper
charts and mechanical writing instruments such as an inked pen or stylus.
Electronic recording instruments are of two types :
(a) null typewhich operate on a comparison basis.
(b) galvanometer typewhich operate on deflection type.
Electronic Instruments 2697
3. Controlling instruments
These are widely used in industrial processes. Their function is to control the quantity being mea-
sured with the help of information feed back to them by monitoring devices. This class forms the basis of
automatic control systems (automation) which are extensively employed in science and industry.

73.4. Electronic Versus Electrical Instruments


Both electrical and electronic instruments measure electrical quantities like voltage and current
etc. Purely electrical instruments do not have any built-in amplifying device to increase the ampli-
tude of the quantity being measured. The common dc voltmeter based on moving-coil meter move-
ment is clearly an electrical instrument.
The electronic instruments always include in their make-up some active electron device such as
vacuum tube, semiconductor diode or an integrated circuit etc.
As seen, the main distinguishing factor between the two types of instruments is the presence of an
electron device in the electronic instruments. Of course, movement of electrons is common to both
types, their main difference being that control of electron movement is more effective in electronic
instruments than in electrical instruments.
Although electronic instruments are usually more expensive than their electrical counterparts,
they offer following advantages for measurements purposes :
1. since electronic instruments can amplify the input signal, they possess very high sinsitivity
i.e. they are capable of measuring extremely small (low-amplitude) signals,
2. because of high sensitivity, their input impedance is increased which means less loading
effect when making measurements,
3. they have greater speed i.e. faster response and flexibility,
4. they can monitor remote signals.

73.5. Essentials of an Electronic Instrument


As shown Fig. 73.1, an electronic instrument is made up of the following three elements :
1. Transducer
It is the first sensing element and is required only when measuring a non-electrical quantity, say,
temperature or pressure. Its function is to convert the non-electrical physical quantity into an electri-
cal signal.
Of course, a transducer is not required if the quantity being measured is already in the electrical
form.
2. Signal Modifier
It is the second element and its function is to make the incoming signal suitable for application to
the indicating device.
For example, the signal may need ampli-
fication before it can be properly displayed.
Other types of signal modifiers are : voltage
Transducer Indicating
dividers for reducing the amount of signal ap- (Thermocouple) Meter
plied to the indicating device or wave shap-
ing circuits such as filters, rectifiers or chop- Signal
per etc. Modifier
3. Indicating Device
For general purpose instruments like volt-
meters, ammeters or ohm meters, the indicat-
ing device is usually a deflection type meter Fig. 73.1
as shown in Fig. 73.1. In digital readout in-
struments, the indicating device is of digitial design.
2698 Electrical Technology

73.6. Measurement Standards


All instruments, whether electrical or electronic, are calibrated at the time of manufacture against
a measurement standard.
1. International Standards
These are defined by international agreement and are maintained at the international Bureau of
Weights and Measurements in Paris.
2. Primary Standards
These are maintained at national standards laboratories in each country. They are not available
for use outside these laboratories. Their principal function is to calibrate and verify the secondary
standards used in industry.
3. Secondary Standards
These are the basic reference standards used by industrial laboratories and are maintained by the
particular industry to which they belong. They are periodically sent to national laboratory for calibra-
tion and verification against primary standards.
4. Working Standards
These are the main tools of a measurement laboratory and are used to check and calibrate the
instrument used in the laboratory.

73.7. The Basic Meter Movement


It is also called D Arsonval meter movement or a permanent-magnet moving-coil (PMMC)
meter movement. Since it is widely used in electronic instruments, it is worthwhile to discuss its
construction and principle of operation.
1. Construction
As shown in Fig. 73.2, it consists of a permanent horse-shoe magnet with soft iron pole pieces
attached to it. Between the two pole-pieces is situated a cylinder-shaped soft iron core around which
moves a coil of fine wire wound on a light metal frame. The metal frame is mounted in jewel bearings

Permanent
Magnet

Core

N S Pole
Piece S

I N

(a) (b)

Fig. 73.2

so that it can rotate freely. A light pointer attached to the moving coil moves up-scale as the coil
rotates when current is passed through it. The rotating coil is prevented from continuous rotation by
a spring which provides restoring torque.
The moving coil movement described above is being increasingly replaced by tautband move-
Electronic Instruments 2699
ment in which the moving coil and the pointer are suspended between bands of spring metal so that
the restoring force is torsional. The bands perform two functions (i) they support the coil
and (ii) they provide restoring torque thereby eliminating the pivots and jewels used with coil spring
movement.
As compared to pivoted movement, the taut-band has the advantages of
1. greater sensitivity i.e. small full-scale deflection current
2. ruggedness,
3. minimal friction,
4. easy to manufacture.
2. Principle of Operation
This meter movement works on the motor principle and is a current-responding device. The
deflection of the pointer is directly proportional to the amount of current passing through the coil.
When direct current flows through the coil, the magnetic field so produced reacts with the field
of the permanent magnet. The resultant force turns the coil alongwith its pointer. The amount of
deflection is directly proportional to the amount of current in the coil. Hence, their scale is linear.
With correct polarity, the pointer reads up-scale to the right whereas incorrect polarity forces the
pointer off-scale to the left.

73.8. Characteristics of Moving Coil Meter Movement


We will discuss the following three characteristics :
(i) full-scale deflection current (Im ),
(ii) internal resistance of the coil (R m ),
(iii) sensitivity (S).
1. Full-scale Deflection Current (Im)
It is the current needed to deflect the pointer all the way to the right to the last mark on the
calibrated scale. Typical values of Im for D Arsonval movement vary from 2 A to 30 mA.
It should be noted that for smaller currents, the number of turns
in the moving coil has to be more so that the magnetic field pro- Rm
duced by the coil is strong enough to react with the field of the per-
manent magnet for producing reasonable deflectin of the pointer. +
Fine wire has to be used for reducing the weight of the moving coil
but it increases its resistance. Heavy currents need thick wire but
lesser number of turns so that resistance of the moving coil is com-
paratively less. The schematic symbol is shown in Fig. 73.3. Fig. 73.3

2. Internal Resistance (Rm )


It is the dc ohmic resistance of the wire of the moving coil. A movement with smaller Im has
higher R m and vice versa. Typical values of R m range from 1.2 for a 30 mA movement to 2 k for
a 50 A movement.
3. Sensitivity (S)
It is also known as current sensitivity or sensitivity factor. It is given by the reciprocal of full-
scale deflection current Im .
1
S = ohm/volt.
Im
For example, the sensitivity of a 50-A meter movement is
1 = 1
S = /V = 20,000 /V = 20 k/V
50 A 50 10 6
2700 Electrical Technology

The above figure shows that a full-scale deflection of 50 A is produced whenever 20,000 of
resistance is present in the meter circuit for each volt of applied voltage. It also represents the ohms-
per-volt rating of the meter. The sensitivity of a meter movement depends on the strength of the
permanent magnet and number of turns in the coil. Larger the number of turns, smaller the amount of
current required to produce full-scale deflection and, hence, higher the sensitivity. A high current
sensitivity means a high quality meter movement. It also determines the lowest range that can be
covered when the meter movement is modified as an ammeter (Art 73.10) or voltmeter (Art 73.12)

73.9. Variations of Basic Meter Movement


The basic moving-coil system discussed in Art 73.7 can be converted into an instrument to
measure dc as well as ac quantities like current, voltage and resistance etc. Without any modification,
it can carry a maximum current of Im can withstand a maximum dc voltage v = Im Rm .
1. DC instruments
(a) it can be made into a dc ammeter, milliammeter or micrommeter by adding a suitable
shunt resistor R sh in parallel with it as shown in Fig. 73.4 (a),
(b) it can be changed into a dc voltmeter by connecting a multiplier resistor R mult in series
with it as shown in Fig. 73.4 (b),

Rm Rm Rm R

Shunt
Rmult.
Rsh

DC DC Ohm-Meter
Ammeter Volt-Meter

(a) (b) (c)

Fig. 73.4
(c) it can be converted into an ohmmeter with the help of a battery and series resistor R as
shown in Fig. 73.4 (c).
2. AC Instruments
(a) it can be changed into an ac audio-frequency ammeter or voltmeter by simply adding an
extra rectifier as shown in Fig. 73.5 (a).

Thermo-
Insulating couple
Bead
Source

Source
a.c.

a.c.

Heater

(a) (b)

Fig. 73.5
Electronic Instruments 2701
(b) it can be converted into a radio frequency ammeter or voltmeter by adding a thermocouple
as shown in Fig. 73.5 (b).
The above modifications of the basic meter movement have been tabulated below :
Basic Meter Movement

DC Instruments AC Instruments

dc ammeter dc voltmeter ohmmeter audio-frequency radio-frequency


by using a by using by using ac ammeter ammeter
shunt series battery or voltmeter or voltmeter
resistor multiplier and series by using a by using a
resistor resistor rectifier thermocouple

73.10. Converting Basic Meter to DC Ammeter


As stated earlier and again shown in Rm
Fig. 73.6 (a), the basic meter movement Im Rm I A Im B I
can carry a maximum current of Im i.e. its
Ish
full-scale deflection current. However, its Rsh
current range can be increased (i.e. multi-
plied) to any value by connecting a low Ish = (I Im)
resistance (called shunt resistance R sh) in
parallel with it as shown in Fig. 73.6 (b). + +
The shunted meter works as an ammeter (a) (b)
with an extended range. Fig. 73.6
Suppose, we want to measure a line current of I with the help of this meter movement. Obvi-
ously, the value of R sh should be such as to shunt or bypass a current of (I Im). As seen, range extension
is from Im to I. The ratio I/Im = n is known as the multiplying power or multiplying factor of the
shunt. It means that a shunt allows the meter to measure current I which is n times larger than Im .
Value of Rsh
In Fig. 73.6 (b), voltage across the meter and the shunt is the same because they are joined in
parallel.
Im Rm = Ish . Rsh = (I Im ) R sh
Im 1 Rm
Rsh = .R = . R Rsh =
( I I m ) m ( I / I m 1) m (n 1)
Hence, n is the multiplying factor of the shunt. It is seen that larger the value of n i.e. greater the
range extension required, smaller the shunt resistance needed. Incidentally, it may be noted that the
resistance of the shunted meter is
R R
= R m || R sh = m sh
Rm + Rsh
It is much less than either R m or R sh
Example 73.1. It is required to convert a 5-mA meter with 20 internal resistance into a 5-A
ammeter. Calculate
(a) the value of shunt resistance required
(b) multiplying factor of the shunt.
2702 Electrical Technology

Solution. Here, I = 5A, Ish = 5 mA = 0.005 A , R m = 20


Im
(a) R sh = . R = 0.005 20 = 0.02 (approx)
(I I sh ) m (5 0.005)
I 5
(b) n = I = 0.005 = 1000
sh
Rm 20 20
Note. R sh = (n 1) = (1000 1) = 999 = 0.02 as found above
Fig. 73.7 shows such an ammeter connected in a load circuit.

73.11. Multirange Meter


The shunt resistance discussed above gives only a single range ammeter. By using universal
shunt (also called Ayrton shunt), we can obtain a multirange ammeter as shown in Fig. 73.8.
It is seen that by changing the switch position from A to B to C and finally to D, the current range
can be extended as desired.
1. Switch at A
Here, the meter is unshunted and so can read up to its full-scale deflection current of 1 mA only.
2. Switch at B
In this case, R 1 shunts the meter and extends its range to 10 mA i.e. increases it ten times.
Rm 100
Since n = 10 R1 = = 11.11
(n 1) (10 1)
3. Switch at C
Here, R 2 shunts R m and extends meter range from 1 mA to 0.1 A i.e. to 100 mA. Obviously,
Im = 1mA, Rm = 100 

AO 1mA
Rm
R1 10 mA
5A A B I
I I B
sh
S
Rsh R2 C
Supply RL
0.1A

R3 1A
+

Fig. 73.7 Fig. 73.8

100 100
n = = 100 R2 = = 1.01
1 (100 1)
4. Switch at D
In this case, R 3 shunts R m and extends the current range of the meter from 1 mA to 1.0 A i.e. 1.0
A to 1000 mA. Hence n = 1000/1 = 1000.
100
R3 = = 0.1001
(1000 1)
Incidentally, it may be noted that greater the range extension, smaller the shunt resistance.
Electronic Instruments 2703
Alternative Method
Rm = 100  0 1 mA
An alternative circuit for range extension is shown in Fig. 73.9.
It is called add on method of shunting the meter because resis-
tances can be added one after another for changing the range. Un- R1 R2 R3
like in Fig. 73.8, there is no possibility of the meter being in the
circuit without any shunt. 1.0A
C
As seen, the universal, shunt consists of three resistances R 1, R 2 0.1 A
and R 3. How they are connected as a shunt is determined by the B
switch position. When S is at position A , the combination (R 1 + R 2
A
+ R 3) becomes connected across R m. When S is at position B, (R 2 +
R 3) become connected in parallel across (R 1 + R m ) and so on.
1. Switch at A
Fig. 73.9
In this case, multiplying factor n = 10 mA/1 mA = 10

Rm
(R 1 + R 2 + R 3) = = 100 ...(i)
(n 1) 9
2. Switch at B
Here, (R 2 + R 3) become in parallel with (R 1 + R m) or (R 1 + 100). Also, n = 100/1 = 100

R1 + 100
R 2 + R3 = ...(ii)
99
3. Switch at C
In this position, R 3 is in parallel with (R 1 + R 2 + 100) and n = 1000/1 = 1000

R1 + R2 + 100
R3 = ...(iii)
999
Solving for R 1, R 2 and R 3 from Eq. (i), (ii) and (iii) we have
R1 = 10 , R 2 = 1 and R 3 1/9

73.12. Measurement of Current


While measuring current flowing in a circuit, following two points must be kept in mind :
1. The current meter must be connected in series with the circuit where current is to be mea-
sured (Fig. 73.7). The full circuit current cannot flow through the meter unless it is made a
series component.
2. The dc meter must be connected with the correct polarity for the pointer to read up-scale to
the right. Reversed polarity deflects the pointer down-scale to the left forcing it against the
stop which can sometime bend the pointer.

73.13. Converting Basic Meter to DC Voltmeter


The basic meter movement can measure a maximum voltage of Im R m which is very small [Fig.
73.10 (a)]. However, its voltage range can be extended to any value by connecting a large resistance
in series with it as shown in Fig. 73.10 (b). The series resistance is also called multiplier resistance
because it multiplies the voltage reading capability of the meter many times. It is usually connected
inside the voltmeter case.
2704 Electrical Technology

Rm Im
Rse Rm
(V V)
Rse

Load
Im (V-n) V V
V = Im Rm Im
V (V = ImRm)
+
(a) (b)
B

Fig. 73.10 Fig. 73.11

But it should be noted that the voltmeter is connected in parallel with the load across which the
voltage is to be measured (Fig. 73.11).
Value of Rsc
Suppose, it is desired to extend the voltage range of the meter from v to V. The ratio V /v is known
as the voltage multiplication. As seen from Fig. 73.11, drop across R se is (V v) and current through
it is the same as meter current i.e. Im
Im R se = (V ) ...(i)
V V I m Rm V
R se = = = Rm
Im Im Im
The voltage multiplication (m) can be found from Eq. (i) above,
Dividing both sides by , we get
I m Rse
= V 1

V = 1 + I m Rse = 1 + I m Rse
I m Rm
Rse Rse
V = 1 + R m = 1 + R
m m
It is seen that for a given meter, higher the series resistance, greater the voltage range extension.
Example 73.2. A 50-A meter movement with an internal resistance of k is to be used as dc
voltmeter of range 50 V. Calculate the
(a) multiplier resistance required and (b) voltage multiplication.

V
Solution. (a) R se = Rm
Im
50 1000
R3 R2 R1
=
50 10 6
6

= 10 1000 = 999,000 = 999 k C 50 V B 25 V A 5V

73.14. Multirange DC Voltmeter


A multirange voltmeter with ranges of 05 V. 025 V and 050 V 0-50 A
is shown in Fig. 73.12. Different values of resistors R 1, R 2 and R 3 can 1k
be found in the same way as in Art. 73.11. It would be found that for
the meter movement shown in figure Fig. 73.12
Electronic Instruments 2705
R1 = 99 K, R 2 = 499 K, R 3 = 999 K
It is seen that higher the voltage range greater the multiplier resistance required (in almost the
same proportion as the ranges).

73.15. Loading Effect of a Voltmeter


When the voltmeter resistance is not high as compared to the resistance of the circuit across
which it is connected, the measured voltage becomes less. The decrease in voltage may be negligible
or it may be appreciable depending on the sensitivity (ohms-per-volt rating) and input resistance of
the voltmeter. It is called voltmeter loading effect because the voltmeter loads down the circuit
across which it is connected. Since input resistance of electronic voltmeter is very high (10 M or
more), loading is not a problem in their case.
Consider the circuit shown in Fig. 73.13 in which two 15-K resistors are connected in series
across a 100-V dc source. The drop across each is 50V. Now, suppose, that a 30-K voltmeter is
connected across R 2 to measure voltage drop across it. Due to loading effect of the voltmeter, the
reading is reduced from 50V to 40V as explained below. As seen from Fig. 73.13 (b), combined
resistance of R 2 and voltmeter is 15 K || 30 K = 10 K.

R1 = 15 K 15 K A 15 K A
50 V 60 V
R2 = 30 K
100 V 100 V 15 K 100 V 10 V 40 V
15 K 50 V V

B B
(a) (b) (c)

Fig. 73.13

10 100 = 40V
drop across 10 K =
10 + 15
Loading effect can be minimized by using a voltmeter whose resistance is as high as possible as
compared to that of the circuit across which it is connected.
Correction Formula
The loading effect can be neutralized by using the following formula :
R1 R2
V corr = V means + .V
Rv R1 + R2 ) means
(
where V corr = corrected voltage reading
V meas = measured voltage reading
Rv = voltmeter resistance
R 1, R 2 = voltage dividing resistances in the circuit
In the above case,
15 15
V corr = 40 + 40 = 40 + 10 = 50 V
30 (15 + 15)

73.16. Ohmmeter
The basic meter movement can be used to measure resistance it is combined with a battery and a
current-limiting resistance as shown in Fig. 73.14 (a). In that case, it is known as an ohmmeter.
2706 Electrical Technology

Ohmmeter OHMS

5k 150
0
1400 
100  0.5
mA

k
200
1.5 V

X Y
Leads
0 1 0
Leads Leads
Open shorted

(a) (b)

Fig. 73.14
For measuring resistance, the ohm-meter leads
X-Y are connected across the unknown resistance af-
ter switching off the power in the circuit under test.
Only in that case, the ohmmeter battery can provide
current for the meter movement. Since the amount
of current depends on the amount of external resis-
tance, the meter scale can be calibrated in ohms (in-
stead of mA).
When the leads X-Y are shorted, meter current is
1.5V/(100 + 1400) = 1 mA. The meter shows full-
scale deflection to the right. The ohmmeter reading
corresponds to 0 because external resistance is zero.
When leads X-Y are open i.e. do not touch each other,
meter current is zero. Hence, it corresponds to infi-
nite resistance on the ohmmeter scale.
Following points about the ohmmeter are worth
noting :
1. the resistance scale is non-linear i.e. it is ex-
panded at the right near zero ohm and
crowded at the left near infinite ohm. This Fig. 73.15. Digital micro-ohmmeter
nonlinearity is due to the reciprocal func-
tion I = V/R.;
2. the ohmmeter reads up-scale regardless of
the polarity of the leads because direction
of current is determined by the internal bat-
tery;
3. at half-scale deflection, external resistance
equals the internal resistance of the ohm-
meter.
4. the test leads should be shorted and ZERO
OHMS control adjusted to bring the pointer
to zero on each range. Fig. 73.16. Digital milli-ohmmeter
Electronic Instruments 2707
Fig. 73.15 shows a digital micro-ohmmeter having a range of 1 2 k with 3 digit,
7-segment LED display. It has a basic accuracy of 0.2% 1 digit and is based on a design using
MOS LSI ICs and glass epoxy PCB.
Fig. 73.16 shows a battery-operated portable digital milli-ohmmeter having a measurement
range of 200 m 2 k with an accuracy of 0.5% 1 digit. It has a 3 digit 7-segment LED
display.

73.17. The Multimeter


It is extensively used in cable industry, motor industry, transformer
and switchgear industry. It is also called volt-ohm-milliammeter
(VOM). It is a general purpose instrument having the necessary cir-
cuitry and switching arrangement for measuring ac/dc voltage or ac/
dc current or resistance. It is simple, compact and portable because
the only power it uses is the battery for the ohm-meter.
Multimeters may be of analog type (Fig. 73.17) or digital type
(Fig. 73.18). The analog type is of the pointer and scale type i.e. it
uses the basic D Arsoval meter movement. However digital
multimeters (DMMs) are becoming increasingly popular because of
their easy readability, numerical display and imporved accuracy.
Fig. 73.17. A digital multimeter
Fig. 73.17 shows the photograph of an
analog multimeter designed primarily for electrical, electronic, radio
and TV engineers and technicians. It sells under the brand name of
Motwane Multimeter 8 X Mark-III.
It is a 5-function, 30-range meter which measures high ac/dc voltages
from 0 to 2.5 kV and ac/dc currents from 0 to 10 A. Its three resistance
ranges cover from 0 to 20 M. It is reputed for its excellent reliability,
operational simplicity and easy portability.
Fig. 73.18 depicts a digital multimeter which can measure dc voltage
upto 1000 V, ac voltages upto 750 V, ac/dc currents from 15 A to 10 A
and resistances from 0 to 100 M. It
has a 5 digit multifunction vacuum fluo-
rescent display allowing the user to mea-
sure two different parameters of the same
signal from one test connection. The user
can also view both measurements at the
same time.
In Fig. 73.19 is shown a hand-held
Fig. 73.18
autoranging, digital multimeter (DMM)
Courtesy : Fluke Corporation
having high contrast, 4 digit LCD read-
out. It has been designed for speed, accuracy and reliability.

73.18. Rectifier Type AC Meter


Rectifier
The D Arsonval meter movement can be used for measuring al-
ternating quantities provided a rectifier is added to the measuring cir-
cuit. A similar rectifier arrangement is found as part of AC VOLTS
function in multimeters (Art. 73.17). Such as meters are more widely
used than either (costly but more accurate) dynamometer type or more
Fig. 73.19. Courtesy : Fluke
delicate thermal and hotwire type. Corporation
2708 Electrical Technology

(a) With Half-Wave Rectifier


The circuit of an ac Voltmeter using half-wave diode recifier is shown in Fig. 73.20. Here, a half-
wave rectifier has been combined in series with a dc meter movement.
When used as a dc voltmeter (i.e. without rectifier) it would have (say, for example) a range of
10V . However, if an ac voltages of rms value 10V is applied across input terminals A B, it would read
4.5 V.
It is so because an ac voltage of rms value 10 V has a peak value of 10 2 = 14 V and an
average value of 0.636 14 = 9V . Since in the half-wave rectified output, one half-cycle is absent, the
average for the full cycle is 9/2 = 4.5 V . The meter movement will, therefore read 4.5 V i.e. 45% of the
dc value. It may also be noted that ac sensitivity of a half-wave ac meter is only 45 per cent of the dc
sensitivity.
(b) With Full-Wave Rectifier
The circuit is shown in Fig. 73.21. In this case, the meter reading would be 90% of rms input
voltage i.e. 90% of the dc value.
73.19. Electronic Voltmeters
A VOM can be used to measure voltages but it lacks both sensitivity and high input resistances.
Moreover, its input resistance is different for each range. The electronic voltmeter (EVM), on the
A RSC D

14 V
14 V Im = 1mA 9V
O Rm = 100 

0
1 CYCLE
B
(a) (b )
Fig. 73.20
other hand, has input resistance ranging from 10 M to 100 M, thus producing less loading of the
circuit under test than the VOM. Another advantage of EVM is that its input resitance remains con-
stant over all ranges.
Output
Two types of voltme-
ters are in use today (i) D2
D1
analog and (ii) digital.
AC Input
However, a distinction R
must be made between a
digital instrument and an
instrument with digital
readout. A digital instru- D3 D4
M
ment is one which uses
internal circuitry of digi-
tal design. A digital read-
out instrument is one
whose measuring cir- Fig. 73.21
cuitry is of analog design but the indicating device is of digital design.
The electronic voltmeters go by a variety of nemes reflecting the technology used.
(i) vacuum-tube voltmeter (V T V M)it uses vacuum tubes with deflection meter movement,
(ii) digital voltmeters like transistor voltmeter (T V M) and FET voltmeter (FETVM).
Electronic Instruments 2709
73.20. Direct Current FET VM
The schematic diagram of a FET VM using difference amplifier is shown in Fig. 73.22. The two
FETs are identical so that increase in the current of one FET is offset by corresponding decrease in the
source current of the other. The two FETs form the
+ VDD
lower arms of the balanced bridge circuit whereas the
two drain resistors R D form the upper arms. The meter
movement is connected across the drain terminals of
RD RD
the FETs.
The circuit is balanced under zero-input-voltage
condition provided the two FETs are identical. In that R
M
case, there would be no current through M. Zero-Ad- A
F 1F 2
just potentiometer is used to get null deflection in case
there is a small current through M under zero-signal R 1

condition. B
RS
Full-scale calibration is adjusted with the help of
variable resistor R. VDD

When positive voltage is applied to the gate of F1, Fig. 73.22


some current flows through M. The magnitude of this
current is found to be proportional to the voltage being measured. Hence, meter is calibrated in volts
to indicate input voltage.

73.21. Electronic Voltmeter for Alternating Currents


The block diagram of such an EVM for ac Voltage
AC DC
measurements is shown in Fig. 73.23 where Input
Divider
Amplifier
Rectifier
& Filter Meter
voltage divider allows range selection. The am-
plifier provides the necessary gain to establish
voltmeter sensitivity as well as high input im-
pedance. The negative feed-back ensures sta- Negative
bility and accurate overall gain. Feedback

73.22. The Digital Voltmeter (DVM) Fig. 73.23

Such a voltmeter displays measurements


of dc or ac voltages as discrete numerals instead of pointer deflections on a continuous scale as in
analog instruments. As compared to other voltmeters, a DVM offers the advantages of :

Integrator

VX Digital
R Control
Logic Counters Readout
VREF + +

Comparator

Clock

Fig. 73.24
1. greater, speed, 2. higher accuracy and resolution, 3. no parallax, 4. reduced human error,
5. compatibility with other digital equipment for further processing and recording.
2710 Electrical Technology

With the development and perfection of IC


modules, the size and power requirement of DVMs have
reduced to a level where they can compete with Slope  VX Slope  Vref
conventional analog instrument both in price and

Volts
portability.
The block diagram of a DVM based on dual-slope
technique is shown in Fig. 73.24. The dual-slope ana-
log-digital (A - D) converter consists of five basic blocks 0
: an Op-Amp used as an integrator, a level comparator, T t
a basic clock (for generating timing pulses), a set of Time
decimal counters and a block of logic circuitry.
Fig. 73.25
The unknown voltage V x is applied through switch
S to the integrator for a known period of time T as shown in Fig. 73.25. This period is determined by
counting the clock frequency in decimal counters. During time period T, C is charged at a rate
proportional to V x.
At the end of time interval T, S is shifted to the reference voltage V ref of opposite polarity. The
capacitor charge begins to decrease with time and results in a down-ward linear ramp voltage. During
the second period a known voltage (i.e. V ref is observed for an unkown time (t). This unknown time
t is determined by counting timing pulses from the clock until the voltage across the capacitor reaches
its basic reference value (reference may be ground or any other basic reference level). From similar
triangles of Fig. 73.25.
Vx Vref Vref
= Vx = V ref
T t t
The count after t which is proportional to the input volt-
age V x is displayed as the measured voltage.
By using appropriate signal conditioners, currents,
resistances and ac voltages can be measured by the same in-
strument.
DVMs are often used in data processing systems or data
logging systems. In such systems, a number of analog input
signals are scanned sequentially by an electronic system and
then each signal is converted into an equivalent digital value
by the A/D converter in the DVM. The digital value is then
transmitted to a printer alongwith the information about the
input line from which the signal has been derived. The whole
data is then printed out. In this way, a large number of intput
signals can be automatically scanned or processed and their
values either printed or logged.
Fig. 73.26 shows a portable digital dc micro-voltmeter
(Agronic-112). It has a measurement range of 1 V - 1000 V
with an accuracy of 0.2% 1 digit. It uses latest MOS LSI
ICs and glass epoxy PCB. It has 3 digit, 7-segment LED
display and is widely-used by the testing and servicing Fig. 73.26. Digital Voltmeter
departments of industries, research laboratories, educational institutions and service centres.

73.23. Cathode-Ray Oscilloscope (CRO)


It is generally referred to as oscilloscope or scope and is the basic tool of an electronic engineer
and technician as voltmeter, ammeter and wattmeter are those of an electrical engineer or electrician.
Electronic Instruments 2711
The CRO provides a
two-dimensional visual
Input Vertical
display of the signal Signal Amplifier
waveshape on a screen To Crt
thereby allowing an
electronic engineer to HT Supply
see the signal in vari- Crt
LT Supply
ous parts of the circuit.
To All
It, in effect, gives the Circuits
electronic engineer an
eye to see what is Trigger Sweep Horizontal
Circuit Generator Amplifier
happening inside the
circuit itself. It is only
by seeing the signal Fig. 73.27
waveforms that he/she
can correct errors, understand mistakes in the circuit design and thus make suitable adjustments.
An oscilloscope can display and also measure many electrical quantities like ac/dc voltage, time,
phase relationships, frequency and a wide range of waveform characteristics like rise-time, fall-time
and overshoot etc. Non-electrical quantities like pressure, strain, temperature and acceleration etc.
can also be measured by using different transducers to first convert them into an equivalent voltage.
As seen from the block diagram of an oscilloscope (Fig. 73.27), it consists of the following major
sub-systems :
1. Cathode Ray Tube (CRT)it displays the quantity being measured.
2. Vertical amplifierit amplifies the signal waveform to be viewed.
3. Horizontal amplifierit is fed with a sawtooth voltage which is then applied to the X -plates.
4. Sweep generatorproduces sawtooth voltage waveform used for horizontal deflection of
the electron beam.
5. Trigger circuitproduces trigger pulses to start horizontal sweep.
6. High and low-voltage power supply.
The operating controls of a basic oscilloscope are shown in Fig. 73.28.
The different terminals provide.
1. horizontal amplifier input,
2. vertical amplifier input,
3. sync. input,
4. Z-axis input,
5. external sweep input.
As seen, different controls permit adjustment of
1. Intensityfor correct brightness of the trace on the screen,
2. Focusfor sharp focus of the trace.
3. Horizontal centeringfor moving the pattern right and left on the screen.
4. Vertical centeringfor moving the pattern up and down on the screen.
5. Horizontal gain (also Time/div or Time/cm)for adjusting pattern width.
6. Vertical gain (also volt/div or volt/cm)for adjusting pattern height.
7. Sweep frequencyfor selecting number of cycles in the pattern.
8. Sync. voltage amplitudefor locking the pattern.
The different switches permit selection of :
1. sweep type,
2. sweep range,
3. sync. type
2712 Electrical Technology

A CRO can operate upto 500 MHz, can allow viewing of signals within a time span of a few
nanoseconds and can provide a number of waveform displays simultaneously on the screen. It also
has the ability to hold the displays for a short or long time (of many hours) so that the original signal
may be compared with one coming on later.

73.24. Cathode Ray Tube (CRT)


It is the heart of an oscilloscope and
is very similar to the picture tube in a tele-
vision set.
Construction
The cross-sectional view of a general-
purpose electrostatic deflection CRT is
shwon in Fig. 73.29. Its four major com-
ponents are :
1. an electron gun for producing a
stream of electrons,
2. focussing and accelerating an-
odes-for producing a narrow and
sharply-focussed beam of elec- A photography of Cathode ray tube
trons,
3. horizontal and vertical deflecting plates-for controlling the path of the beam,
4. an evacuated glass envelope with a phosphorescent screen which produces bright spot when
struck by a high-velocity electron beam.

Astigmatism
Focus
Horizontal
Vertical Centering
Centering

x 100
x 10 x 1000
x1 CAL
Frequency Intensity
Selector

Vertical Sync Fine Horizontal


Gain Amplitude Frequency Gain

Vert. I/P Into Ext. Hor Horz.


Pilot Amp. Input

GND Sync. Input Power Saw-Tooth GND

Fig. 73.28
Electronic Instruments 2713
As shown, a CRT is a self-contained unit like any electron tube with a base through which leads
are brought out for different pins.
1. Electron Gun Assembly
The electron gun assembly consists of an indirectly-heated cathode K, a control grid G, a pre-
accelerator anode A 1, focussing anode A 2 and an accelerating anode A 3. The sole function of the

A3 Aquadag
G A1 Coating
K A2 Y X
F

Flourescent
Electron Focussing & Screen
Deflection
Emission Acceleration
Electron Gun

Fig. 73.29
electrons gun assembly is to provide a focussed beam of electrons which is accelerated towards the
flourescent screen. The electrons are given off by thermionic emission from the cathode. The control
grid is a metallic cylinder with a small aperture in line with the cathode and kept at a negative poten-
tial with respect to K. The number of electrons allowed to pass through the grid aperture (and, hence,
the beam current) depends on the amount of the control grid bias. Since the intensity (or brightness)
of the spot S on the screen depends on the strength of beam current, the knob controlling the grid bias
is called the intensity control.
The anodes A 1 and A 3, which are both at positive potential with respect to K, operate to acceler-
ate the electron beam (Fig. 73.30). The cylindrical focussing anode A 2, being at negative potential,
repels electrons from all sides and compresses them into a fine beam. The knob controlling the
potential of A 2 provides the focus control.

H
V
G A1 A2 A3

H S

 1350V
 350V
Intensity Focus
Control Control

Fig. 73.30
2. Deflecting Plates
Two sets of deflecting plates are used for deflecting the thin pencil-like electronic beam both in
the vertical and horizontal directions. The first set marked Y (nearer to the gun) is for vertical deflec-
tion and X -set is for horizontal deflection. When no potential is applied across the plates, beam passes
between both sets of plates undeflected and produces a bright spot at the centre of the screen.
2714 Electrical Technology

If upper Y -plate is given a positive potential, the beam



is deflected upwards depending on the value of the ap-
plied potential. Similarly, the beam (and hence the spot) V
deflects downwards when lower Y -plate is made positive.
However, if an alternating voltage is applied across the
Y -plates, the spot keeps moving up and down thereby pro- 0 t
TSW
ducing a vertical luminous trace on the screen due to
persistance of vision. The maximum displacement of the
Fig. 73.31
spot from its central position is equal to the amplitude of
the applied voltage.
The screen spot is deflected horizontally if similar voltages are applied to the X -plates. The dc
potentials on the Y -and X -plates are adjustable by means of centring controls.
It must be remembered that the signal to be displayed on the screen is always applied across the
Y -plates. The voltage applied across X -plates is a ramp voltage i.e. a voltage which increases linearly
with time. It has a sawtooth wave-form as shown in Fig. 73.31. It is also called horizontal time-base
or sweep voltage. It has a sweep time of T sw.
3. Glass Envelope
It is funnel-shaped having a phosphor-coated screen at its flared end. It is highly-evacuated in
order to permit the electron beam to traverse the tube easily. The inside of the flared part of the tube
is coated with a conducting graphite layer called Aquadag which is maintained at the same potential
as A 3. This layer performs two functions (i) it accelerates the electron beam after it passes between
the deflecting plates and (ii) collects the electrons produces by secondary emission when electron
beam strikes the screen. Hence, it prevents the formation of negative charge on the screen.
The screen itself is coated with a thin layer of a flourescent material called phosphor. When
struck by high-energy electrons, it glows. In other words, it absorbs the kinetic energy of the elec-
trons and converts it into light-the process being known as flourescence. That is why the screen is
called flourescent screen. The colour of the emitted light depends on the type of phosphor used.

73.25. Deflection Sensitivity of a CRT


Fig. 73.32 shows the upward deflection of an electron beam when it passes between the vertical
or Y -plates of a CRT. The beam deflects
upwards because the upper Y -plate has + Vd
been made positive with respect to the Deflected
Electron
lower plate. Reversing the polarity of the Beam
applied voltage would, obviously, cause Y y
the beam to deflect downwards.
The vertical deflection of the beam is
1 V d
y = .D. l . d
2 D VA
where V A is the accelerating voltage ap-
plied to the electrons which make up the 1 D
electron beam.
The deflection sensitivity of a CRT
is definition as the vertical deflection of
the beam on the screen per unit deflect- Fig. 73.32
ing voltage.
Electronic Instruments 2715
y
S =
Vd
lD
Using the above equation, we get S = 2
dV A
The deflection factor which is defined as the reciprocal of deflection sensitivity is given by
G = 1/S.
Substituting the value of S from above
d V
G = 2 . . A volt/metre
l D
73.26. Normal Operation of a CRO
The signal to be viewed or displayed on the screen is applied across the Y -plates of a CRT. But to
see its waveform or pattern, it is essential to spread it
out horizontally from left to right. It is achieved by ap- Crt
plying a sawtooth voltage wave (produced by a time Screen
base generator) to X -plates. Under these conditions, the
electron beam would move uniformly from left to right
thereby graphic vertical variations of the input signal
versus time. Due to repetitive tracing of the viewed wave-
form, we get a continuous display because of persis-
tence of vision. However, for getting a stable stationary
display on the screen, it is essential to synchronize the TSW = TS TSW = 2TS
horizontal sweeping of the beam (sync) with the input
(a) (b)
signal across Y -plates. The signal will be properly synced
only when its frequency equals the sweep-generator fre- Fig. 73.33
quency.
In general, for proper synchronization of time-base with the signal, the condition is
Tsw = n Ts
where T s the time-period of the signal and n is an Input Vertical
integer. Signal Amplifier

If n = 1, then T sw = T s i.e. time-periods of the Crt


sweep voltage and input signal voltage are equal, then
one cycle of the signal would be displayed as shown
Horizontal
in Fig. 73.33 (a). Amplifier
On the other hand, if T sw is twice T s, then two
cycles of the signal voltage would be displayed as
Sawtooth
shown in Fig. 73.33 (b). Obviously, three full cycles Sweep Generator
of the input voltage would be spread out on the
screen when T sw = 3 T s.
Internal Synchronization
External Internal
The periodic sawtooth voltage which is applied Syne Syne
to X -plates for horizontal sweep (or scan) of the
beam across the screen is usually provided by the Fig. 73.34
unijunction relaxation oscillator. When the sawtooth voltage falls abruptly to zero, the beam experi-
ence no horizontal deflection and hence flies back almost instantly to the original (central) position.
The usual method of synchronizing the input signal is to use a portion of the input signal to
trigger the sweep generator so that the frequency of the sweep signal is locked or synchronized to the
input signal. It is called internal sync. because the synchronization is obtained by internal wiring
connection as shown in the block the diagram of Fig. 73.34.
2716 Electrical Technology

73.27. Triggered and Non-Triggered Scopes


Oscilloscopes may be classified into two basic types :
1. triggered sweep type.
2. recurrent sweep (free-running) type.
Triggered oscilloscopes, being more sophisticated, are generally used in industrial laboratories
and plants, in engineering and technical school laboratories and in all those applications which require
study of low- and high-frequency
waveforms, for accurate measurement
of time and timing relationships etc.
A non-triggered oscilloscope is
generally used in servicing work where
a certain amount of waveform error can
be tolerated and bandwidth require-
ments are limited to a few MHz.
The sweep (or ramp) generator
which produces sawtooth voltage for X -
deflection plates is prresent in both types
of scopes. In non-triggered oscillo-
scopes, this generator runs continuously
(recurrent sweep) and the control and
calibration of the sweep is based on the
repetition freqeuncy of the sweep. For
producing a stable stationary display, the
sweep frequency has to be forced into
synchronization with the input signal
on the Y -plates. This is done by manu-
ally adjusting the free-running sweep
freqeuncy to a value very close to sig-
nal frequency (or some submultiple of
it) and then depending on the internal
sync signal (derived from the input) for Fig. 73.35. A non-triggered oscilloscope
locking the sweep generator into exact
step. Unfortunately, this method is limited to the display of signals which have contant frequency and
amplitude. Hence voice or music signals from a microphone cannot be displayed on this scope
because it has to be readjusted for each for new change in frequency. Moreover, a free-running or
recurrent time base cannot display less than one complete cycle of the input signal on the scope
screen. On the other hand, triggered time base can be adjusted to pick out a small part of a waveform
which can then be expended horizontally for evaluation of waveform details.
The triggered oscilloscope is provided with a triggered (or driven) sweep. Here the input signal
is caused to generate pulses that trigger the sweep thereby ensuring that the sweep is necessarily in
step with the trigger that drives it. Hence, screen display remains stable in spite of variations in the
frequency or amplitude of the input signal. It means that there is automatic mode of triggering in such
scopes. Consequently, input signals of very short duration can be displayed for the simple reason that
sweep is initiated by a trigger pulse derived from the waveform under observation.
Fig. 73.35 illustrates a triggered oscilloscope having a bandwidth 0-6 MHz, vertical
sensitivity of 10 mV/div and horizontal sweep rate varying from 0.2 s/div to 0.1 s/div.
Electronic Instruments 2717
73.28. Dual Trace CRO
Such oscilloscopes are used extensively by industrial firms and research laboratories. They produce
a dual-trace display by means of electronic switching of two separate input signals. As shown in the
block diagram of Fig. 73.36, there are two vertical
Channel
input circuits marked channel A and B with A Pre Amp
identical pre-amplifiers. The outputs of the these
preamplifiers are fed to an electronic switch which
alternately connects them to the main vertical Electronic Vertical
Switch Amplifier To
amplifier of the oscilloscope. In this type of scope, CRT
ther is only one electron beam. The electron switch Channel
is also capable of selecting a variety of display B Pre Amp
modes.
In Fig. 73.37 is shown a dual trace Fig. 73.36
oscilloscope (type VOS-26) which has a band-width of 5-15 MHz, vertical sensitivity of 10 mV/cm to
30 V/cm and calibrated sweep speed from 0.3 s/cm to 10 ms/cm. It is a very sensitive yet simple
oscilloscope which assures long life and easy maintenance.

73.29. Dual Beam CRO


Such a CRO has two sets of vertical deflection plates and has two electronic beams which pro-
duce two separate traces on the
scope screen by using the same
set of horizontal deflection
plates. This scope makes it
possible to observe two time-
related wave forms at different
points i.e. the electronic circuit.
Such a scope does not
have the same number of dis-
play modes as the dual-trace
scope yet it is ideally suited for
different input signals.
Fig. 73.37. Dual Beam CRO
73.30. Storage Oscilloscope
It can retain a CRT display for 10 to 150 hours after the pattern is first produced on the screen. It
uses the phenomenon of secondary electron emission to build up and store electrostatic charges on
the surface of an insulated target. Such oscillo-
scopes are especially useful.
1. For real-time observation of events
that occur only once.
2. For displaying the waveform of a very
low-frequency signal.
Fig. 73.38 shows a 4 channel storage os-
cilloscope with 400 MHz bandwidth. It has a Fig. 73.38. 4 channel storage oscilloscope
standard floppy disk drive which makes the
saving of screen images or data to a disk, simple. The disk can then be inserted into your personal
computer (PC) for importing to desk top publishing or spreadsheet programs. The storage oscillo-
scopes finds their application in biophysics/biomedical research, audio system measurement and
analysis, power supply and power-related design, electrophysical and electromechanical system de-
sign etc.
2718 Electrical Technology

73.31. Sampling CRO


It is specifically meant to observe very high frequency repetitive electric signals by using the
sampling technique. Such high-frequency signals cannot be viewed by conventional oscilloscopes
because its frequency range is limited by the gain-bandwidth product of its vertical amplifier. The
sampling technique slows down the signal frequency many thousands of times thereby making it
easier to view it on the screen.
The oscilloscope shown in Fig. 73.38 is a sampling cathode ray oscilloscope. Its sample rate is
100 M samples per second.

73.32. Digital Readout CRO


It provides digital readout of the signal information such as voltage or time etc. in addition to the
conventional CRT display. It consists of a high-speed laboratory CRO and an electronic counter, both
contained in one cabinet.

73.33. Handheld Battery Operated Oscilloscope


Fig. 73.39 shows a handheld battery operated oscillo-
scope Model THS 720 P manufactured by Tektronix Cor-
poration. It has built in 3-3/4 digit digital multimeter
(DMM) with *data logger and power analyser. It has a
bandwidth of over 100 MHz and the sampling rate is as
high as 500 M samples per second. The oscilloscope and
power analyser can operate simultaneously and indepen-
dently on the same or separate signals. This type of an
oscilloscope is extremely useful for electric/power elec-
tronics measurements. Examples of such measurements are
: (1) testing and verifying correct operation of motors (2)
checking transformer efficiency, (3) verifying power
supply performance, (4) measuring the effect of neutral
current etc.
Fig. 73.39. Courtesy : Tektronix
Corporation
73.34. Lissajous Figures
Lissajous figures (or patterns) are Scope
named in honour of the French scientist who
first obtained them geometrically and opti-
Audio Audio
cally. They illustrate one of the earliest uses Oscillator-1 Oscillator-2
to whch the CRO was put.
Lissajous patterns are formed when two
sine waves are applied simultaneously to the
vertical and horizontal deflecting plates of
a CRO. The two sine waves may be ob-
tained from two audio oscillators as shown
in Fig. 73.40. Obviously, in this case, a sine Vert. Horiz.
Input Input
wave sweeps a sine-wave input signal. The
shape of the Lissajous pattern depends on Fig. 73.40

* Data logger is a system which can acquire data and store it in a memory.
Electronic Instruments 2719
the frequency and phase relationship of the Vertical
two sine waves. Lissajous Deflection Voltage
Figures ev = Em sin ( +
)
Two sine waves of the same frequency
and amplitude may produce a straight line,
an ellipse or a circle depending on their phase = 0
difference (73.41).
In general, the shape of Lissajous fig-
ures depends on (i) amplitude, (ii) phase dif-
ference and (iii) ratio of frequency of the
two waves. = 30 OR 330
Lissajous figures are used for (i) deter-
mining an unknown frequency by compar-
ing it with a known frequency (ii) checking
audio oscillator with a known-frequency sig-
nal and (iii)checking audio amplifiers and = 90 OR 270
feedback networks for phase shift.

73.35. Frequency Determination


with Lissajous Figures
= 150 OR 210
The unknown signal is applied across
one set of deflecting plates and a known sig-
nal across the other. By studying the result-
ant Lissajous pattern, unknown frequency
can be found.
Depend on the frequency ratio, the vari- = 180
ous patterns obtained are shown in Fig.
73.42. The ratio of the two frequencies is
given by
Horizontal
f H No. of points of horizontal tangency Deflection Voltage
= eh = Em sin
f No. of points of vertical tangency
t
In Fig. 73.41 (a), there is one point of
tangency along the horizontal as well as ver-
tical axis. Hence, f H = f v i.e. the signals have
the same frequency. In Fig. 73.42 (e) f H/f = Fig. 73.41
3/2. In other words f H = 1.5 f .
It should be noted in passing that this method of frequency determination has limitations and is
being discarded gradually because low-cost digital frequency counters are becoming increasingly
available in the market Fig. 73.43. The two main limitatioins of this method are as under :
(i) the numerator and denominator of the frequency ratio must be whole numbers,
(ii) the maximum ratio of frequencies that can be used is 10 : 1. Beyond that, the Lissajous
patterns become too complex to analyse.
Fig. 73.43 shows a 10-digit digital frequency counter Model No. PM 6685 manufactured by
Fluke Corporation. This can measure frequencies from 10 Hz to 300 MHz. This is an ideal instru-
ment for R and D laboratories, testing, servicing and even outside the lab environment such as in base
station transmitters of large telecommunication networks like GSM.
2720 Electrical Technology

(a) (b) (c)

1:1 2:1 1:2

(d) (e) (f)

3:1 3:2 4:3

Fig. 73.42

73.36. Applications of a CRO


As stated earlier, no other instrument in electronic
industry is as versatile as a CRO. In fact, a modern
oscilloscope is the most useful single piece of electronic
equipment that not only removes guess work from tech-
nical troubleshooting but makes it possible to deter-
mine the trouble quickly. Some of its uses are as Fig. 73.43
under : (Courtesy : Fluke Corporation)
(a) In Radio Work
1. to trace and measure a signal throughout the RF, IF and AF channels of radio and television
receivers.
2. it provides the only effective way of adjusting FM receivers, broadband high-frequency RF
amplifiers and automatic frequency control circuits;
3. to test AF circuits for different types of distortions and other spurious oscillations;
4. to give visual display of waveshapes such as sine waves, square waves and their many differ-
ent combinations;
5. to trace transistor curves
6. to visually show the composite synchronized TV signal
7. to display the response of tuned circuits etc.
(b) Scientific and Engineering Applications
1. measurement of ac/dc voltages,
2. finding B/H curves for hysteresis loop,
3. for engine pressure analysis,
4. for study of stress, strain, torque, acceleration etc.,
5. frequency and phase determination by using Lissajous figures,
6. radiation patterns of antenna,
7. amplifier gain,
Electronic Instruments 2721
8. modulation percentage,
9. complex waveform as a short-cut for Fourier analysis,
10. standing waves in transmission lines etc.

73.37. The Q Meter


This instrument is designed to measure some of the electrical properties of coils and capacitors
by measuring the Q-value of an R-L-C circuit.
1. Construction
As shown in Fig. 73.44, it essentially consists of
(i) a frequency-calibrated continuously-variable RF oscillator,
(ii) a calibrated variable capacitor C,
(iii) V T V M which is calibrated to read Q directly.

L R

Oscillator
(50 kHz- T T
(30-460 pF)

50 Mhz) VTVM
Rsh
E C EC
00 2

Fig. 73.44
2. Principle of Operation
The basic principle used in Q-meter is the resonant rise of the voltage across the capacitor in an
R.L.C circuit. The condition for series resonance (Art. 14.10) is
XL = X C and E = IR
The value of circuit Q is
X L X C EC
Q = = =
R R E
If the applied voltage E is constant, then Q EC. Hence, by measuring voltage drop across C
under resonant conditions, Q can be found. Alternatively, V T V M can be calibrated directly in terms of
Q (rather than voltage).
As seen from Fig. 73.44, the oscillator
delivers current to a very small (0.02 ) Coil
C EL
shunt resistance R sh thereby developing a
voltage E across it. It becomes the applied L R
E = IR
voltage for the RLC circuit and corresponds EL I
to the source voltage E of Fig. 73.45.
It is measured by a thermocouple meter
marked Multiply Q by. The voltage EC E
Ee
across variable C is measured by the VTVM.
The value of Q-factor is given by Q = EC/E. Fig. 73.45

For making measurement, the unknown coil is connected to the test terminals T T of the instru-
ment and the circuit is tuned to resonance (Fig. 73.44)
(i) either by setting the oscillator to a given frequency and varying C,
2722 Electrical Technology

(ii) or by keeping value of C constant and adjusting the oscillator frequency.


The reading on the V T V M must be multiplied by the index setting of the Multiply Q by meter in
order to obtain the actual value of Q.
3. Applications
Some of the specialized uses of this instrument are to measure
1. Q of a coil,
2. inductance and capacitance,
3. distributed capacitance of a coil,
4. Q and p.f. of a dielectric material,
5. mutual inductance of coupled circuits,
6. coefficient of coupling,
7. critical coupling,
8. reactance and effective resistance of an inductor at operating frequency,
9. bandwidth of a tuned circuit etc.
The above list does not exhaust the number of its possible applications. It has been very truly
said that if ever an RF problem exists, a Q meter can always provide the answer.

73.38. Logic Analysers


The oscilloscope is probably the best tool for the development of analog or digital system de-
sign. It can be used to examine the waveforms and determine the voltage and rise time of the analog
or digital signals. But the oscilloscope has two limitations especially when used in digital system
design : (1) high speed random pulses can not be observed easily (2) oscilloscope cannot monitor a
few signal lines simultaneously. For example, in a commonly available oscilloscope, the maximum
number of inputs are four.
For these reasons, the logic analysers has been developed. It operates on a slightly different
principle than that of an oscilloscope. Because there are many signal lines in a digital system (such as
a microprocessor based system), the data is changing rapidly on each line, a logic analyser must take
a snap shot of the activities on the lines and store the logic state of each signal in memory for each
cycle of the system clock. The conditions under which the snapshot is taken are determined by
triggering circuits, which can respond to various combinations of events.
The logic analyser enables the activity of many digital signal points to be recorded simulta-
neously and then examined in detail. The information is recorded with respect to a clock signal to
determine whether they are HIGH or LOW with respect to a defined threshold voltage. This informa-
tion is stored in memory and is then available for detailed analysis via the logic analysers display.
The clock signal can be internally or externally generated.

Digital System Data Information


Under Test Gathering Unit Processing Display
& Storage Unit Unit
CLK

Logic Analyser

Fig. 73.46
Electronic Instruments 2723
Fig. 73.46 shows a block diagram of a typical logic analyser. It has a data gathering unit, infor-
mation processing and storage unit and a display unit. The data gathering unit has (1) a pod slots for
carrying data from the digital system under test to the logic analyser and (2) a key pad. The key pad
is used to enter commands and set up the parameters that the logic anayser will use. The display unit
is a cathode ray tube (CRT) that displays the command menu for the operator and also displays the
output data.
Applications
M hardware/software debugging
M parametric/mixed signal testing
M hardware simulation and stimulus-response testing
M complex debugging with deep memory.
Fig. 73.47 shows a family of logic analysers, available
from Tektronix corporation. Each logic analyser has at least
34 channels, 4-channel digitizing oscilloscope, off-line Fig. 73.47. Logic analysers
analysis capability for viewing data and creating setups on a separate PC.
Applications.
Logic analyser is a very powerful tool in the field of microprocessor based system develop-
ment. Some of its major applications in this area are :
1. Hardware debug and verification.
2. Processor/bus debug and verification.
3. Embedded software integration, debug and verification.
73.39. Spectrum Analysers
The spectrum analyser is an instrument that brings together a superhetrodyne radio receiver with a
swept frequency local oscillator and an oscilloscope to present a display of amplitude versus frequency.
Input

cy
q uen
Input Fre
g
nin
Tu
e pt
Sw

xer
Mi IF Filter
Horiz
CRT
Display
Vert
Detector
Sweep

Fig. 73.48
(Courtesy Hewlett Packard)
Fig. 73.48 shows a simple block diagram of a spectrum analyser. As seen, the spectrum analyser
is actually a superhetrodyne receiver in which local oscillator is a sweep generator. A low frequency
saw-tooth wave is applied to both the sweep oscillator and the horizontal deflection plates of the
CRT, producing a horizontal deflection that is a function of frequency. The lowest frequency is
represented by left side of the trace while the highest frequency is represented by the right side of the
trace. The sweep is from left to right.
2724 Electrical Technology

The input signal is mixed with local oscillator to produce the


IF (i.e. intermediate-frequency or difference) signal. The band-
width of the IF amplifier is relatively narrow. so the output signal
at the detector will have a strength that is proportional to the fre-
quency that the local oscillator is converting to the IF at that in-
stant. The display will then contain poles that represent the am-
plitudes of the various input frequency components.
There are several spectrum analysers available in the market (a)
manufactured by the companies like Rhode & Schwarz (Tektronix),
Hewlett Packard (now called Agilent Technologies), and so on.
Fig. 73.49 shows two commercially available spectrum analyses.
The spectrum analyser shown in Fig. 73.49 (a) is Model FSE
30 manufactured by Rhode & Schwarz but marked by Tektronix
Corporation.
It has a frequency range from 20 Hz to 76.5 GHz, a band-
width from 1 Hz to 10 MHz. Another spectrum analyser Model (b)
No. 3066 shown in Fig. 73.49 (b) is a real-time instrument. It has Fig. 73.49
a frequency range from DC to 5 MHz, a bandwidth from DC to 3 (Courtesy : Rhode & Schwarz and
Tektronics Corporation)
GHz. The real-time spectrum analyser take a very different ap-
proach compared to traditional sweeping spectrum analysers. Rather than acquiring one frequency
step at a time, the real time spectrum analyser captures a block of frequencies all at once.
It is possible to use computers to do spectrum analysis of the signals. There is a variety of
software available over the internet from several companies. Some softwares can be downloaded
free of cost from the companies web-sites.
Applications
The spectrum analyser is used to :
1. check the spectral purity of signal sources.
2. evaluate local electromagnetic interference (EMI) problems.
3. do site surveys prior to installing radio receiving or transmitting equipment.
4. test transmitters.
5. analyse signatures.

73.40. Signal Generators


A signal generator is an instrument that provides a controlled output waveform or signal for use
in testing, aligning or in measurements on other circuits or equipment. The signal generators can
be classified into the following categories :
1. Audio generators 2. Function generators
3. Pulse generators 4. RF generators
5. Frequency synthesizers 6. Other signal generators.

73.41. Audio Generators


The audio generators covers the frequency range 20 Hz to 20 kHz, although few models produce
signals up to 100 kHz. Audio generators always produce pure sine waves and most also produce
square waves. They uses a 600 output impedance and produce output levels from 40 dB mW to
+ 4 dB mW.
Two methods of frequency selection are typically used in audio signal generators. continuous
and step. On the continuous type of a dial, we turn a knob to the desired frequency. Many such audio
Electronic Instruments 2725
generators have a scale that reads 20 to 200 (or
alternatively 2 to 20) and a range selector switch
determines whether the output frequencies will be
20 to 200 Hz, 200 to 2000 Hz or 2000 to 20,000
Hz. In a step-tuned generator, these controls are
replaced by a rotary or pushbutton switch bank.
As many as four decode switches might be used,
although three is a more common number. These
will be marked 0 through 100, 0 through 10 and
0.1 through 1.0 in decade steps. A multiplier
switch determines whether the actual frequency
will be X1, X10, X100 or X1000 the frequency
indicated on the selector switches.
Fig. 73.50 shows a block diagram of an au-
dio signal generator. The audio oscillator section
is usually an RC phase-shift oscillator (or a Wien
Bridge oscillator) circuit. A power amplifier stage
provides buffering between the load and the os-
cillator and it develops the output signal ampli- Audio signal generator
tude. The ac voltmeter at the output is strictly optional, but in some models it is used with a level
control to set precisely the input signal to the attenuator. Not all quality audio signal generators use
this feature. So the lack of an ac output meter is not, in itself, indication of quality. In some models,
an audio digital frequency counter is used ahead of the attenuator to provide digital display of the
output frequency.

Frequency Output Output Level


Select Meter Set

Sinewave
Power Sine Output
Audio Attenuator
Ampl. Square
Oscillator

Squarer

Power
Ampl.

Fig. 73.50
Applications
The audio generators are basically used to test the amplitude and frequency response of audio
amplifiers.

73.42. Function Generators


These generators typically, cover at least the same frequency range as audio signal generators
(i.e. 20 Hz to 20 kHz) but most modern designs have extended frequency ranges. A very common
frequency range for function generators is 0.01 Hz to 3 MHz.
2726 Electrical Technology

The major difference between a func-


tion generator and an audio generator is in
the number of output waveforms. The au-
dio signal generator produces only sine
waves and square waves. While almost all
function generators produce these basic
waveforms plus triangular waves. Besides
this, some function generators also produce
sawtooth, pulse and non-symmetrical square
waves. Fig. 73.51 shows the controls of a
typical function generator.
Fig. 73.52 shows a simple block dia- Fig. 73.51
gram of a function generator. The major
parts of a function generator are schmitt trigger, integrator, sine-wave converter and an attenuator.
The schmitt trigger converts a slowly varying input signal to a square wave signal. This square wave
signal is available at the output as well as it is also connected to the integrator as an input through a
potentiometer (R). The potentiometer is used to adjust the frequency of the output signal. The
frequency range is adjusted by selecting the appropriate capacitor connected in the integrator circuit.
The sine-wave converter is a six-level (or more) diode-resistor loading circuit.

R Frequency
control
Schmitt
trigger

Output
Integrator Attenuator

Sine wave
converter
Frequency
range selection

Fig. 73.52
Let us see how a simple diode-resistor circuit shown in Fig. 73.53 (a) is used to convert a trian-
gular wave into a square wave.
Note that if diodes D1 and D2 and resistors R 3 and R 4 were not present in the circuit, of Fig. 73.53
(a), R 1 and R 2 would simply behave as a voltage divider. In this case the output from the circuit would
be an attenuated version of the triangular wave :
R2
V0 = Vi
R1 + R2
With diodes D1 and R 3 in the circuit, R 1 and R 2 still behave as a simple voltage divider until the
voltage drop across R 2, V R2 exceeds + V 1. At this point D1 becomes forward biased, and R 3 is
effectively in parallel with R 2. Now,
R2 || R3
V0 = Vi
R1 + R2 || R3
Electronic Instruments 2727

Triangular
input wave
Input R1 Output
+ V1
R3 D1 D2 R4
+V1 V1

R2

V1

(a) (b)

Fig. 73.53
Output voltage levels above + V 1 are attenuated to a greater extent than levels below + V 1.
Consequently, the output voltage rises less steeply than without D1 and R 3 in the circuit (refer to Fig.
73.53 (b). When the output falls below + V 1, the diode D1 is reverse biased. As a result of this, R 3 is
no longer in parallel with R 2, and the attenuation is once again R 2/(R 1 + R 2). Similarly during the
negative half cycle of the input, the output voltage,
R2
V0 = Vi
R1 + R2
until V 0 goes below V 1. Then D2 becomes forward biased, putting R 4 in parallel with R 2 and making,
R2 || R4
V0 = Vi
R1 + R2 || R4
With R 3 = R 4, the negative half-cycle of the output is similar in shape to the positive half-cycle. If
we employ six or more diodes, all connected via resistors to different bias voltage levels (refer to Fig.
73.54 (a), a good sine-wave approximation can be achieved. With six diodes, the three positive bias
voltage levels and three negative bias voltage levels, the slope of the output wave changes three times
during each quarter cycle. Assuming correctly selected bias voltages and resistor values, the output
wave shape is as shown in Fig. 73.54 (b).

Input R1 Output

R7 D5 D6 R8
+V3  V3 V1
R5 D3 D4 R6 V2
+V2  V2 V3
R3 D1 D2 R4
+V1  V1
 V1
 V2
R2  V3

(a) (b)
Fig. 73.54
2728 Electrical Technology

Applications
The function generator is an essential equipment for an electronic
laboratory to generate signals to test a variety of analog and digital
system during the design phase as well as to trouble shoot such sys-
tems.
Fig. 73.55 shows the picture of a Tektronix function gen-
erators Model No. CFG 253. This model has a frequency range
Fig. 73.55
form 0.03 Hz to 3 MHz. In addition to sine, square and triangu-
(Courtesy : Tektronix Corporation)
lar waves it can also produce TTL signals. Another function
generator from Tektronix (Model No. CFG280 has a wide frequency range form 0.01 Hz to 11 MHz.
It has a built in frequency counter with a range from 1 Hz to 100 MHz.
Applications
The function generators can be employed in a variety of applications in the area of product
design, training, manufacturing production test, field repair, bench calibration and repair, labora-
tory and research, and education. Mainly they are used for testing amplifiers, filter and digital
circuits.

73.43. Pulse Generators


Fig. 73.56 shows the block diagram of a pulse generator. As seen, an astable multivibrator
generates square waves. This is used to trigger monostable multivibrator (i.e. one-shot). The pulse
repetition rate is set by the square-wave frequency. The one-shot triggers on the leading edge of the
square-wave and produces one output pulse for each input cycle. The duration of each output pulse
is set by the on-time of the one-shot. It may be very short or may approach the period of the square
wave. The attenuator facilities output amplitude control and dc level shifting.

Astable Monostable
Multivibrator or Attenuator and Output
multivibrator DC level shifter
Square wave or one-shot
source

Frequency Pulse Duraticn

Fig. 73.56

A typical pulse generator will allow the user to select


the repetition rate, duration, amplitude and number of out-
put pulses to be output in a given burst. The most common
frequency range is from 1 Hz to 50 MHz. The pulse width
is adjustable from 10 ns to over 10 ms and the output is
variable from 3 mV to 30 V.
Fig. 73.57 shows a pulse generator from Fluke Corpo-
ration Model No. PM5786. This instrument has a frequency
Fig. 73.57. (Courtesy Fluke Corporation)
range from 1 Hz to 125 MHz. The output pulse width can
Electronic Instruments 2729
be varied from 8 ns to 100 ms. The instrument can also be used to generate pulse bursts. The output
voltage level can be adjusted up to 10 V.
Applications
The pulse generators are used extensively to test :
1. Memory circuits
2. Shift registers
3. Counters
4. Other digital components, subsystems and systems.

73.44. RF Generators
A radio frequency (RF) signal generator has a sinusoidal output with a frequency somewhere in
the range of 100 kHz to 40 GHz region. Fig. 73.58 shows the block diagram of an RF generator. As
soon, the instrument consists of an RF oscillator, an amplifier, a calibrated attenuator and an output
level meter.
Output level
meter

Calibrated
RF oscillator Amplifier attenuator

Frequency

Frequency Output Output


range amplitude
Set level

Fig. 73.58
The RF oscillator has a continuous frequency control and a frequency range switch, to set the
output to any desired frequency. The amplifier includes an output amplitude control. This allows the
voltage applied to the attenuator to be set to a calibration point on the output level meter. The output
level must always be reset to this calibration point everytime the frequency is changed. This is
necessary to ensure that the output voltage levels are correct, as indicated on the calibrated attenuator.
The oscillator circuit used in an RF generator is usually either a Hartley Oscillator or Colpitts
oscillator. Most RF signal generators include facilities for
amplitude modulation and frequency modulation of the out-
put. Switches are provided on the front panel to allow the
user to select no modulation as well as internal or external
A M or FM modulation. It may be noted that each section
of the RF generator is shielded by enclosing it in a metal
box. The whole system is then completely shielded. The
purpose of shielding is (1) to prevent RF interference be-
tween the components and (2) to prevent the emission of
RF energy from any point except the output terminals. As
a matter of fact, even the power line is decoupled by means
of RF chokes and capacitors to prevent RF emission from
it.
Fig. 73.59 (a) shows an analog RF generator Model Fig. 73.59
No. SML01 manufactured by Tektronics Corporation. This (Courtesy Fluke Corporation)
2730 Electrical Technology

instrument is a general purpose signal generator and is available at low cost. It has a wide frequency
range from 9 kHz to 3.3 GHz. Another RF generator Model No. SMP04 shown in Fig. 73.59 (b) from
Tektronix Corporation is a high precision signal source. It has a wide frequency range from 0.01 GHz
to 40 GHz. This instrument can produce, AM, FM, phase and pulse modulated signals as well.
Applications
The RF signal generators are widely used in the area of radar and communication, research and
development laboratories, education and training, electromagnetic interference (EMI) testing and
material testing. Their main applications are :
1. To perform variety of tests on radio transmitters and receivers.
2. To test the amplitude and frequency response of RF amplifiers during the design phase.

73.45. Frequency Synthesizer


It is another type of RF generator that uses phase-locked loop (PLL) to generate output frequen-
cies over a wide range. The most common range is from 1 MHz to 160 MHz. Fig. 73.60 shows a
simple block diagram of a frequency synthesizer. As seen, the major components of the frequency
synthesizer are : voltage controlled oscillator (VCO), divide-by-N counter, phase detector, crystal
oscillator, low-pass filter and a square-wave circuit.
pw

fr E f0 = Nfr

fo/N 0

Voltage-controlled Output
Crystal Phase Low-pass
Oscillator
Oscillator detector Filter
VCO

Divide-by-N Square-wave
Counter Circuit

Frequency
selection 1 2 5 0 MHz
Switches
Fig. 73.60

The output of the crystal oscillator (a reference frequency, fr), is fed into one input of a phase
detector. The other input of a phase detector has another square-wave applied to it as shown in the
figure. The frequencies of these two square waves is identical but there is a phase difference ()
between them. The output of the phase detector is a pulse waveform with pulse width controlled by
the phase difference. The output of the phase detector is applied to the low-pass filter which con-
verts it into a dc voltage, E. The dc voltage, E is used as the control voltage for the VCO and it
determines the output frequency of VCO. The output of VCO is fed to a circuit that converts it into
a square wave for triggering a digital divide-by-N counter. The divide-by-N counter divides the
VCO frequency by a number set by a bank of switches. These switches may be push buttons with
digital readouts or they may be thumb-wheel type which indicate their position numerically. The
switches are connected in such a way that the displayed number is the factor N by which the output
frequency is divided before being applied to the phase detector. The switches allow the user to
obtain frequency which is any integer multiple of the crystal oscillator frequency.
Electronic Instruments 2731
Applications
The frequency synthesizer is used in almost same areas as the RF signal generators.

73.46. Other Signal Generators


There are some signal generators that do not fit well in various pres-etabilished categories.
Some of these signal generators are as discussed below :
1. RF Markers : These devices are usually crystal controlled and have a fixed output fre-
quency for use as a reference. These are used to calibrate TV signals.
2. Digitally Programmable Test Oscillators : These instruments can have extremely wide
frequency range although some versions have much narrow range also. The set frequency can be
programmed through the front panel keypad or via a computer interface input such as IEEE-488
general purpose interface bus (commonly known as GPIB).
3. Arbitrary Waveform Gen-
erators : These instruments allow the
user to design and generate virtually any
desired waveform. The arbitrary wave-
form generator is quite useful to perform
a variety of tests on communication
equipment. For example, a modulated
signal that varies over the entire band-
width and amplitude range of the equip-
ment shown in Fig. 73.61 could be cre-
ated for testing purpose. Noise could
also be superimposed upon the signal and
gaps might be introduced between wave-
form bursts, to investigate the response
of the system. Once such a waveform Fig. 73.61
has been designed, it could be stored in
the instrument memory and can be recalled repeatedly for production testing.
Fig. 73.62 shows an arbitrary waveform generator from Agilent Technologies Model No. 33250A.
This instrument can generate real world signals up to 80 MHz. It has a capability to display the
waveforms in colour. It can be used as a function generator/pulse generator as well. The instrument
has a GPIB/LAN interfaces.
Applications
The arbitary waveform generators are used extensively in the following areas :
1. Communications design and test for producing (a)
arbitrary IF based signals and (b) standard wave-
forms for communication.
2. Mixed signal design and test.
3. Disk drive read/write design and test.
4. Real word simulations.
5. High-speed low filter data and clock pulse
generation. Fig. 73.62 (Courtesy : Agilent Technologies)

73.47. IEEE-488 General Purpose Interface Bus (GPIB) Instruments


These days automatic test equipment (ATE) is one of the leading methods for testing electronic
equipment in factory production and troubleshooting situations. The basic method is to use a pro-
grammable digital computer to control a bank of test instruments. The bank of instruments can be
configured for a special purpose or for general use. For example, we could select a particular line-
up of equipment needed to test a broadcast audio console and provide a computer program to do a
2732 Electrical Technology

variety of measurements such as gain, frequency response, total harmonic distortion etc. The other
possibility could be to do a generalized test set. This method is adopted by number of industries
who have many electronic devices or systems to test. There is a main bank of electronic test equip-
ment adapters to make the devices (or systems) under test interconnect with the system and a com-
puter program to do a variety of measurements such as gain, frequency response, total harmonic
distortion etc. The other possibility could be to do a generalized test set. This method is adopted by
number of industries who have many electronic devices or systems to test. There is a main bank of
electronic test equipment adapters to make the devices (or systems) under test interconnect with the
system and a computer program for each type
of equipment. Such an approach reduces the IEEE-488/GPIB
test equipment cost drastically.
The Institution of Electrical and Elec- (a) Instrument
1 Instrument Instrument Instrument
tronic Engineers (IEEE) has laid out a speci- (Controller) 2 3 4
fication titled IEEE standard Digital Interface Instrument
for programmable instrumentation or IEEE- 2
IEEE-488/GPIB
488. This specification provides details for a (b)
standard interface between a computer and in- Instrument Instrument Instrument
struments. The IEEE-488 bus or General pur- 3 1 4
(Controller)
pose interface bus (GPIB) is a tool that is
based on the IEEE specifications. The
Hewlett-Packard interface bus (HPIB) is a Fig. 73.63
proprietary version of the IEEE-488 bus.
The digital signals on the IEEE-488 bus are generally similar to TTL (transistor-transistor logic),
i.e. a logic LOW is less than 0.8 V and a
logic HIGH is greater than 2.0 V. The digi-
tal signals can be connected to the instru-
ments through a multiconductor cable up Controller
to 20 metres in length provided that an (Talker &
Listener)
instrument load is placed every 2 metres. (Computer)
GENERAL INTERFACE MANAGEMENT BUS

Most IEEE-488/GPIB systems operate un-


restricted to 250 kilobytes per second or
DATA BYTE TRANSFER CONTROL BUS

faster with some restrictions.


Talker
There are two basic configurations for
DATA INPUT OUTPUT BUS

(Frequency
the IEEE-488/GPIB system : (1) linear Counter)
DIO 1
and (2) star. In the linear configuration
DIO 2
shown in Fig. 73.63 (a), a tap-off to the
DIO 3
next instrument is taken from the previ- Listener
ous one in series. On the other hand, in DIO 4
star configuration shown in Fig. 73.63 (b), (Signal DIO 5
Generator) DIO 6
the instruments are connected from a cen-
tral point. DIO 7
Talker DIO 8
Fig. 73.64 shows the basic structure
Listener
of IEEE-488/GPIB system. The figure in- DAV
dicates four different devices (i.e. com- (Digital NRFD
puter, frequency counter, signal generator Multimeter) NDAC
and digital multimeter) connected to the IFC
bus. The IEEE-488/GPIB system itself ATN
consists of three major buses : (1) gen- SRQ
eral interface management (GIM) bus. (2) REN
data I/O (DIO) bus and (3) data byte trans- EOI
fer (DBT) bus. Fig. 73.64
Electronic Instruments 2733
1. General Interface Management (GIM) bus : This bus coordinates the whole system and
ensures an ordely flow of data over the data input/output (DIO) bus. This bus has a number of
signals explained below.
IFC (Interface Clear Signal) : This signal Driver
VCC
is used by the controller to place all devices in a

Bus
predefined quiescent or standby condition.
ATN (Attention Signal) : This signal is used
by the computer/controller to let the system know
how data on the DIO bus lines are to be interpreted
and which device is to respond to the data. Receiver
SRQ (Service Request Signal) : This signal
is used by a device on the system to ask the con-
troller for attention. It is basically an interupt re-
quest signal.
REN (Remote Enable Signal) : This signal
is used by the controller to select between two al- Fig. 73.65
ternate sources of device programming data.
EOI (End or Identify Signal) : This
signal is used by talkers for the following
two purposes : (1) it will follow the end of
a multiple byte sequence of data in order to
indicate that the data are now finished. (2)
It is also used in conjunction with the AT N
signal for polling the system.
2. Data I/O (DIO) bus : This bus is
a bidirectional 8-bit data bus that carries
data, interface messages and device-depen-
dent messages between the controller, talk-
ers and listeners. This bus sends asynchro-
nously in byte-serial format.
3. Data Byte Transfer (DBT) bus:
This bus controls the sending of data along
the DIO bus. There are three signals on this
bus as explained below :
DAV (Data Valid Signal) : This sig-
nal indicates the availability and validity of
data on the line. If the measurement is not
finished, for example, the D AV signal will
be false.
NRFD (Not Ready For Data
Signal) : This signal lets the controller
know whether or not the specific device ad-
dressed is in a condition to receive data.
NDAC (Not Data Accepted Signal) :
This signal is used to indicate to the control-
ler whether or not the device accepted the
GPIB Code
data sent to it over the DIO bus. ASCII character Ref : ANSI STD X3. 4-1977
Each signal line in the bus has a circuit decimal IEEE STD 488-1978, ISO STD 646-1973
similar to the one shown in Fig. 73.65. As
Fig. 73.66
seen from this circuit, each signal line has a
pull-up and pull-down resistors, receiver, and driver-circuits. Besides this each signal line has also a
shunt protection diode and a stray capacitance.
Fig. 73.66 shows the 7-bit binary signals used in the IEEE-488/GPIB system for ASCII and
2734 Electrical Technology

GPIB message codes.


The signals defined for the three buses in the IEEE-488/GPIB systems are implemented as con-
ductors in a system interface cable. Each IEEE-488/GPIB compatible instrument will have a female 36-
pin Amphenol-style connector on the rear panel. The pin-out definitions are given in Table 73.1.
The devices connected to IEEE-488/GPIB system (i.e. computer, frequency counter, signal
generator and digital multimeter) are categorised as controller, listener and/or talker.
Table 73.1
Pin No Signal Lin Pin No Signal Line
1 DIO 1 13 DIO 5
2 DIO 2 14 DIO 6
3 DIO 3 15 DIO 7
4 DIO 4 16 DIO 8
5 EOI 17 REN
6 DAV 18 Ground (6)
7 NRFD 19 Ground (7)
8 NDAC 20 Ground (8)
9 IFC 21 (Ground 9)
10 SRQ 22 (Ground 10)
11 ATN 23 (Ground 11)
12 Shield 24 Digital Ground

1. Controller : Its function is to communicate device addresses and other inface buses to
instruments in the system.
2. Listener : Its function is to receive commands from other instruments (usually the control-
ler) when the correct address is placed on the bus.
The listener acts on the message received but does
not send back any data to the controller. The sig-
nal generator shown in Fig. 73.22 is an example
of a listener.
3. Talker : Its function is to respond to the
message sent to it by the controller. The frequency
counter shown in Fig. 73.62 is an example of a talker.
There is also a combination device that ac-
cepts commands from the controller to set up Fig. 73.67
ranges, tasks etc. and then returns data back over
the DIO bus to the controller. The digital multimeter shown in Fig. 73.62 is an example of this
category.
The IEEE-488 was introduced to the electronic industry in 1977. Since then it has evolved to
IEEE-488.1 in 1987 and further to IEEE-488.2 in 1990. At present the system allows the control
upto 14 instruments and it has data transfer rate greater than 1 M bytes/s.

73.48. VXI bus


The VXI bus is another fast growing platform for instrumentation systems. It was introduced in
1987 and since then it has experienced tremendous growth and acceptance around the world. V X I
uses a mainframe chassis with a maximum of 13 slots to hold modular instruments on plug-in
boards. Fig. 73.67 shows an example of system using V X I instruments. The V X I backplane includes
Electronic Instruments 2735
the 32-bit VME data bus, high performance instrumentation buses for precision timing and synchro-
nization between instrument components, standarized initialization and resource management to ease
configuration.

OBJECTIVE TESTS 73
1. Digital instruments are those which ohm resistance. If the coil resistance of the meter
(a) have numerical readout is 1000 , a potential difference of 500 mV is re-
(b) use LED or LCD displays quired across it for full-scale deflection. Under
this condition, the current in the shunt would be
(c) have a circuitry of digital design
(a) 2.5 A (b) 25 A
(d) use deflectioin type meter movement.
(c) 0.25 A (d) 0.025 A
2. The main difference between the electronic and
electrical instruments is that an electronic in- 11. It is desired to convert a 0-1000 A meter move-
strument contains ment, with an internal resistance of 100 into
a 0-100 mA meter. The required value of shunt
(a) an electronic device (b) a transducer
resistance is about
(c) a digital readout (d) electrons.
(a) 1 (b) 10
3. The essential elements of an electronic instru-
(c) 99 (d) 100
ment are
12. Loading effect is principally caused by... instruments
(a) transducer (b) signal conditioner
(a) high resistance (b) low-sensitivity
(c) indicating device (d) all of the above.
(c) high-sensitivity (d) high-range
4. The current sensitivity of a meter is expressed in
13. A multimeter is used to measure
(a) ampere (b) ohm/ampere
(a) resistance (b) current
(c) ohm/volt (d) ampere/division.
(c) voltage (d) all of the above
5. The basic meter movement can be converted
into an ohmmeter by connecting a .....with it. 14. A sinusoidal voltage of rms value 10 V is applied
to a D Arsonval movement connected in series
(a) high resistance in series
with a half-wave rectifier. It will show a reading
(b) low resistance in parallel of... volt
(c) battery in series (a) 9 (b) 4.5
(d) battery and a variable resistance in series (c) 10 (d) 7.7
6. The D Arsonval meter movement can be con- 15. A V T V M produces negligible loading effect on
verted into an audio-frequency ac ammeter by a circuit under test primarily because
adding a ...... to it.
(a) it virtually drawn no current from the circuit
(a) thermocouple (b) rectifier
(b) of its very high internal resistance
(c) chopper (d) transducer.
(c) it uses high vacuum tubes
7. In a linear meter, half-scale deflection occurs
(d) it is a null deflection instrument.
when there is ... per cent of the rated current
through its coil 16. In a 3 digit voltmeter, the largest number that
can be read is
(a) 100 (b) 25
(a) 0999 (b) 1999
(c) 50 (d) 75
(c) 4999 (d) 9999
8. A 0-1 mA meter has a sensitivity of
17. A 3 digit voltmeter having a resolution of 100
(a) 1 k /V (b) 1 mA
mV can be used to measure maximum voltage of
(c) 1 k (d) 1000 A.
(a) 100 V (b) 200 V
9. A moving coil instrument has a resistance of 10
(c) 1000 V (d) 5000 V
and takes 40 mA to produce full-scale deflection.
The shunt resistance required to convert this in- 18. The signal to be observed on the screen of an
strument for use as an ammeter of range 0 to 2 A oscilloscope is applied
is (a) across its X -plates (b) across its Y -plates
(a) 0.1021 (b) 0.2041 (c) to the horizontal amplifier
(c) 0.2561 (d) 0.4210 (d) to the trigger circuit.
10. A moving coil ammeter has a fixed shunt of 0.02 19. When a 30 V dc is applied to the vertical de-
flection plates of a cathode ray tube, the bright
2736 Electrical Technology

spot moves 1 cm away from the centre. If 30 V 24. Two complete signal cycles would be displayed
(rms) ac is applied, then the movement of the on the screen of a scope when time-period of
spot will be nearly the sweep generator is ...... the signal time period.
(a) 1 cm (b) 1.5 cm (a) half (b) twice
(c) 2 cm (d) 3 cm (c) equal (d) thrice
20. Production of a steady stationary display of a 25. A non-triggered oscilloscope is one which
signal waveform on the scope screen is due to (a) has no sweep generator
(a) persistence of vision (b) cannot produce a stable stationary screen
(b) flourescent material of the screen display
(c) proper sync. between the signal and the (c) has a continuously running time-base genera
sweep generator tor
(d) electrostatic focussing of the electron beam. (d) can display a portion of the input signal
21. Two sinusoidal signals of frequency f and 3f are wave form.
applied at x and y inputs respectively to an os- 26. A dual-trace CRO has
cilloscope. Which one of the following patterns (a) one electron gun (b) two electron guns
can be observed on the screen ? (c) one electron gun and one two-pole switch
(d) two electron guns and one two-pole switch
(a) (b) 27. The operation a Q-meter is based on
(a) self-induction (b) series resonance
(c) (d) (c) mutual induction (d) eddy currents.
28. The resolution of a logic analyser is
22. The X - and Y -inputs of a CRO are respectively (a) maximum number of input channels
V sin wt and V sin wt. The resulting Lissajous (b) the minimum duration of the glitch it can
pattern will be capture
(a) a straight line (b) a circle (c) its internal clock period
(c) an elipse (d) a figure of eight (d) the minimum amplitude of the input signal
23. The deflection sensitivity of a CRT depends in- it can display
versely on the 29. A spectrum analyser can be described as
(a) length of the vertical deflecting plates (a) voltage selective frequency meter
(b) distance between screen and deflecting plates (b) current selective frequency meter
(c) deflecting voltage (c) frequency selective voltmeter
(d) None of these
(d) separation between Y -plates.

ANSWERS
1. (c) 2. (a) 3. (d) 4. (c) 5. (d) 6. (b) 7. (c) 8. (a) 9. (b) 10. (b)
11. (a) 12. (b) 13. (d) 14. (b) 15. (b) 16. (a) 17. (c) 18. (a) 19. (a) 20. (c)
21. (b) 22. (a) 23. (d) 24. (b) 25. (c) 26. (c) 27. (b) 28. (a) 29. (d)

GO To FIRST
MEASUREMENTS
NOTES

GATE 2009

JANARDHANARAO
GATE 2009 JENNAS MEASUREMENTS NOTES 1
GATE 2009 JENNAS MEASUREMENTS NOTES 2
GATE 2009 JENNAS MEASUREMENTS NOTES 3
GATE 2009 JENNAS MEASUREMENTS NOTES 4
GATE 2009 JENNAS MEASUREMENTS NOTES 5
GATE 2009 JENNAS MEASUREMENTS NOTES 6
GATE 2009 JENNAS MEASUREMENTS NOTES 7
GATE 2009 JENNAS MEASUREMENTS NOTES 8
GATE 2009 JENNAS MEASUREMENTS NOTES 9
GATE 2009 JENNAS MEASUREMENTS NOTES 10
GATE 2009 JENNAS MEASUREMENTS NOTES 11
GATE 2009 JENNAS MEASUREMENTS NOTES 12
GATE 2009 JENNAS MEASUREMENTS NOTES 13
GATE 2009 JENNAS MEASUREMENTS NOTES 14
GATE 2009 JENNAS MEASUREMENTS NOTES 15
GATE 2009 JENNAS MEASUREMENTS NOTES 16
GATE 2009 JENNAS MEASUREMENTS NOTES 17
GATE 2009 JENNAS MEASUREMENTS NOTES 18
GATE 2009 JENNAS MEASUREMENTS NOTES 19
GATE 2009 JENNAS MEASUREMENTS NOTES 20
GATE 2009 JENNAS MEASUREMENTS NOTES 21
GATE 2009 JENNAS MEASUREMENTS NOTES 22
GATE 2009 JENNAS MEASUREMENTS NOTES 23
GATE 2009 JENNAS MEASUREMENTS NOTES 24
GATE 2009 JENNAS MEASUREMENTS NOTES 25
GATE 2009 JENNAS MEASUREMENTS NOTES 26
GATE 2009 JENNAS MEASUREMENTS NOTES 27
GATE 2009 JENNAS MEASUREMENTS NOTES 28
GATE 2009 JENNAS MEASUREMENTS NOTES 29
GATE 2009 JENNAS MEASUREMENTS NOTES 30
GATE 2009 JENNAS MEASUREMENTS NOTES 31
GATE 2009 JENNAS MEASUREMENTS NOTES 32
GATE 2009 JENNAS MEASUREMENTS NOTES 33
GATE 2009 JENNAS MEASUREMENTS NOTES 34
GATE 2009 JENNAS MEASUREMENTS NOTES 35
GATE 2009 JENNAS MEASUREMENTS NOTES 36
GATE 2009 JENNAS MEASUREMENTS NOTES 37
GATE 2009 JENNAS MEASUREMENTS NOTES 38
GATE 2009 JENNAS MEASUREMENTS NOTES 39
GATE 2009 JENNAS MEASUREMENTS NOTES 40
GATE 2009 JENNAS MEASUREMENTS NOTES 41
GATE 2009 JENNAS MEASUREMENTS NOTES 42
GATE 2009 JENNAS MEASUREMENTS NOTES 43
GATE 2009 JENNAS MEASUREMENTS NOTES 44
GATE 2009 JENNAS MEASUREMENTS NOTES 45
GATE 2009 JENNAS MEASUREMENTS NOTES 46
GATE 2009 JENNAS MEASUREMENTS NOTES 47
GATE 2009 JENNAS MEASUREMENTS NOTES 48
GATE 2009 JENNAS MEASUREMENTS NOTES 49
GATE 2009 JENNAS MEASUREMENTS NOTES 50
GATE 2009 JENNAS MEASUREMENTS NOTES 51
GATE 2009 JENNAS MEASUREMENTS NOTES 52
GATE 2009 JENNAS MEASUREMENTS NOTES 53
GATE 2009 JENNAS MEASUREMENTS NOTES 54
GATE 2009 JENNAS MEASUREMENTS NOTES 55
GATE 2009 JENNAS MEASUREMENTS NOTES 56
GATE 2009 JENNAS MEASUREMENTS NOTES 57
GATE 2009 JENNAS MEASUREMENTS NOTES 58
GATE 2009 JENNAS MEASUREMENTS NOTES 59
GATE 2009 JENNAS MEASUREMENTS NOTES 60
GATE 2009 JENNAS MEASUREMENTS NOTES 61
GATE 2009 JENNAS MEASUREMENTS NOTES 62
GATE 2009 JENNAS MEASUREMENTS NOTES 63
GATE 2009 JENNAS MEASUREMENTS NOTES 64
GATE 2009 JENNAS MEASUREMENTS NOTES 65
GATE 2009 JENNAS MEASUREMENTS NOTES 66
GATE 2009 JENNAS MEASUREMENTS NOTES 67
GATE 2009 JENNAS MEASUREMENTS NOTES 68
GATE 2009 JENNAS MEASUREMENTS NOTES 69
GATE 2009 JENNAS MEASUREMENTS NOTES 70
GATE 2009 JENNAS MEASUREMENTS NOTES 71
GATE 2009 JENNAS MEASUREMENTS NOTES 72
GATE 2009 JENNAS MEASUREMENTS NOTES 73
GATE 2009 JENNAS MEASUREMENTS NOTES 74
GATE 2009 JENNAS MEASUREMENTS NOTES 75
GATE 2009 JENNAS MEASUREMENTS NOTES 76
GATE 2009 JENNAS MEASUREMENTS NOTES 77
GATE 2009 JENNAS MEASUREMENTS NOTES 78
GATE 2009 JENNAS MEASUREMENTS NOTES 79
GATE 2009 JENNAS MEASUREMENTS NOTES 80
GATE 2009 JENNAS MEASUREMENTS NOTES 81
GATE 2009 JENNAS MEASUREMENTS NOTES 82
GATE 2009 JENNAS MEASUREMENTS NOTES 83
GATE 2009 JENNAS MEASUREMENTS NOTES 84
GATE 2009 JENNAS MEASUREMENTS NOTES 85
GATE 2009 JENNAS MEASUREMENTS NOTES 86
GATE 2009 JENNAS MEASUREMENTS NOTES 87
GATE 2009 JENNAS MEASUREMENTS NOTES 88
GATE 2009 JENNAS MEASUREMENTS NOTES 89
GATE 2009 JENNAS MEASUREMENTS NOTES 90
GATE 2009 JENNAS MEASUREMENTS NOTES 91
GATE 2009 JENNAS MEASUREMENTS NOTES 92
GATE 2009 JENNAS MEASUREMENTS NOTES 93
GATE 2009 JENNAS MEASUREMENTS NOTES 94
GATE 2009 JENNAS MEASUREMENTS NOTES 95
GATE 2009 JENNAS MEASUREMENTS NOTES 96
GATE 2009 JENNAS MEASUREMENTS NOTES 97
GATE 2009 JENNAS MEASUREMENTS NOTES 98
GATE 2009 JENNAS MEASUREMENTS NOTES 99
GATE 2009 JENNAS MEASUREMENTS NOTES 100
GATE 2009 JENNAS MEASUREMENTS NOTES 101
GATE 2009 JENNAS MEASUREMENTS NOTES 102
GATE 2009 JENNAS MEASUREMENTS NOTES 103
GATE 2009 JENNAS MEASUREMENTS NOTES 104
GATE 2009 JENNAS MEASUREMENTS NOTES 105
GATE 2009 JENNAS MEASUREMENTS NOTES 106

jnrao.k@gmail.com

Contact: 09966715471
CLASS NOTES ON ELECTRICAL MEASUREMENTS & INSTRUMENTATION 2015

CLASS NOTES ON

ELECTRICAL MEASUREMENTS & INSTRUMENTATION

FOR

5TH & 6TH SEMESTER OF

ELECTRICAL ENGINEERING & EEE (B.TECH PROGRAMME)

DEPARTMENT OF ELECTRICAL ENGINEERING


VEER SURENDRA SAI UNIVERSITY OF TECHNOLOGY
BURLA -768018, ODISHA, INDIA

1
CLASS NOTES ON ELECTRICAL MEASUREMENTS & INSTRUMENTATION 2015

DISCLAIMER
This document does not claim any originality and cannot be used as a substitute for prescribed
textbooks. The matter presented here is prepared by the author for their respective teaching
assignments by referring the text books and reference books. Further, this document is not
intended to be used for commercial purpose and the committee members are not accountable for
any issues, legal or otherwise, arising out of use of this document.

2
CLASS NOTES ON ELECTRICAL MEASUREMENTS & INSTRUMENTATION 2015

SYLLABUS
ELECTRICAL MEASUREMENTS & INSTRUMENTATION (3-1-0)

MODULE-I (10 HOURS)


Measuring Instruments: Classification, Absolute and secondary instruments, indicating instruments,
control, balancing and damping, constructional details, characteristics, errors in measurement, Ammeters,
voltmeters: (DC/AC) PMMC, MI, Electrodynamometer type
Wattmeters: Electrodynamometer type, induction type, single phase and three phase wattmeter,
compensation, Energymeters: AC. Induction type siqgle phase and three phase energy meter,
compensation, creep, error, testing, Frequency Meters: Vibrating reed type, electrical resonance type
MODULE-II (10 HOURS)
Instrument Transformers: Potential and current transformers, ratio and phase angle errors, phasor
diagram, methods of minimizing errors; testing and applications.
Galvanometers: General principle and performance equations of D' Arsonval Galvanometers, Vibration
Galvanometer and Ballistic Galvanometer.
Potentiometers: DC Potentiometer, Crompton potentiometer, construction, standardization, application.
AC Potentiometer, Drysdale polar potentiometer; standardization, application.
MODULE-III (10 HOURS)
DC/AC Bridges :General equations for bridge balance, measurement of self inductance by Maxwells
bridge (with variable inductance & variable capacitance), Hays bridge, Owens bridge, measurement of
capacitance by Schearing bridge, errors, Wagners earthing device, Kelvins double bridge.
Transducer: Strain Gauges, Thermistors, Thermocouples, Linear Variable Differential Transformer
(LVDT), Capacitive Transducers, Peizo-Electric transducers, Optical Transducer, Torque meters,
inductive torque transducers, electric tachometers, photo-electric tachometers, Hall Effect Transducer
MODULE-IV (10 HOURS)
CRO: Block diagram, Sweep generation, vertical amplifiers, use of CRG in measurement of frequency,
phase, Amplitude and rise time of a pulse.
Digital Multi-meter: Block diagram, principle of operation, Accuracy of measurement, Electronic
Voltmeter: Transistor Voltmeter, Block diagram, principle of operation, various types of electronic
voltmeter, Digital Frequency meter: Block diagram, principle of operation
TEXT BOOKS
[1]. A Course in Elec. & Electronics Measurements & Instrumentation: A K. Sawhney
[2]. Modern Electronic Instrumentation and Measurement Techniques: Helfrick & Cooper
[3]. Electrical Measurement and Measuring Instruments - Golding & Waddis

3
CLASS NOTES ON ELECTRICAL MEASUREMENTS & INSTRUMENTATION 2015

Model Question Paper: Set-1


Full Marks: 70 Time: 3 hours
Answer any six questions including question No. 1 which is compulsory.
The figures in the right-hand margin indicate marks.
(Answer any six questions including Q.No. 1)
[2X10]
1. Answer the following questions:
(a) Differentiate between the spring control and gravity control.
(b) Why an ammeter should have a low resistance value.
(c) What are the precautions taken while using a DC voltmeter and DC ammeter.
(d) What is the major cause of creeping error in an energy meter
(e) What are the errors occurs in instrument transformers.
(f) Differentiate the principle of dc potentiometer and ac potentiometer.
(g) What are the sources of errors in AC bridge measurement.
(h) Differentiate between dual trace and dual beam CRO.
(i) What are active and passive transducers? Give examples.
(j) What is piezoelectric effect.

2. (a) With a neat diagram explain in detail the construction of PMMC instrument.
(b)What are the shunts and multiplier? Derive the expression for both, with reference to
meters used in electrical circuits. [5+5]
3. (a) Discuss with block diagram, the principle of operation of single phase energy meter.
(b) An energy meter is designed to make 100 revolutions of the disc for one unit of
energy. Calculate the number of revolutions made by it , when connected to a load
carrying 40 A at 230V and 0.4 p.f. for 1 hour. If it actually makes 360 revolutions, find
the percentage error. [5+5]
4. (a) Derive expression for actual transformation ratio, ratio error and phasor angle error of
a P.T.
(b) A current transformer with bar primary has 300 turns in its secondary winding. The
resistance and reactance of the secondary circuit are 1.5 and 1.0 respectively,
including the transformer winding. With 5A flowing in the secondary winding, the
magnetizing mmf is 100AT and the core loss is 1.2 W. Determine the ratio and phase
angle errors. [5+5]
5. (a) Derive the equation of balance for Anderson bridge and also draw the phasor diagram.
(b) An AC bridge is balanced at 2KHz with the following components in each arm:
Arm AB=10K, Arm BC=100F in series with 100K, Arm AD=50K
Find the unknown impendence RjX in the arm DC, if the detector is between BD.
6. (a) What is transducer? Briefly explain the procedure for selecting a transducer.
(b) Derive an expression for gauge factor in terms of Poissions ration. [5+5]
7. (a) With a block diagram, explain the working of CRO
(b) With a block diagram, explain in detail the digital frequency meter [5+5]

8. Write short notes on


(a) Kelvins double bridge
(b) D- Arsonaval galvanometer [5+5]

4
CLASS NOTES ON ELECTRICAL MEASUREMENTS & INSTRUMENTATION 2015

Model Question Paper: Set-2


Full Marks: 70 Time: 3 hours
Answer any six questions including question No. 1 which is compulsory.
The figures in the right-hand margin indicate marks.
(Answer any six questions including Q.No. 1)
[2X10]
Q.1 Answer the following questions:
(a) Why is damping required for an electromechanical measuring instrument?
(b) Why is scale of MI instrument calibrated non- linearly?
(c) What is the difference between analog and digital frequency meter?
(d) Which instrument can be used to measure non-sinusoidal voltage?
(e) What is the major cause of creeping error in an energy meter?
(f) What do you mean by active and passive transducer? Give suitable examples.
(g) Draw a suitable AC bridge used for measurement of frequency.
(h) What are the steps to be taken for minimizing errors in PT?
(i) Discuss briefly the role of ordinary galvanometer?
(j) What is the function of time base generator in CRO?

Q.2 (a) Derive the torque equation of a moving iron instrument? [5]
(b) Sketch and explain the working of moving-coil instrument. [5]

Q.3 (a) Discuss a method for measurement of low resistance. [5]


(b) Explain the operation of a Wagners earthing device. [5]

Q.4 Derive the errors of CT and PT, and discuss its preventives. [10]

Q.5 (a) Discuss about a ac bridge used for measurement of capacitance [5]
(b) Discuss about a galvanometer which is used for measurement of frequency. [5]

Q.6 (a) How is the voltmeter calibrated with DC potentiometer ? [5]


(b) What is the use of LVDT? Discuss its basic principle of operation. [5]

Q.7 (a) How are the frequency and phase measured in CRO. [5]
(b)Draw the block diagram of an electronic voltmeter and explain its operation. [5]

8. Write short notes on any two: [5X2]


(a) Owens bridge
(b)Digital frequency meter
(c) Hall-effect transducer

5
CLASS NOTES ON ELECTRICAL MEASUREMENTS & INSTRUMENTATION 2015

Model Question Paper: Set-3


Full Marks: 70 Time: 3 hours
Answer any six questions including question No. 1 which is compulsory.
The figures in the right-hand margin indicate marks.
(Answer any six questions including Q.No. 1)
[2X10]
1. Answer the following questions:
(k) Why scales of the gravity control instruments are not uniform but are crowded?
(l) Why eddy current damping cannot be used for moving iron instrument?
(m) What is mean by Phantom Load?
(n) Why do we use a multiplier with a voltmeter?
(o) What is the major cause of creeping error in an energy meter?
(p) Why is dynamometer type instrument chiefly used as a wattmeter?
(q) Define gauge factor of a strain gauges how is it related to poisons ratio ?
(r) Why the secondary of a CT is never left open circuited?
(s) Why there are two conditions of balance in ac bridges, where as there is only one for
dc bridges?
(t) How to prevent the loading of a circuit under test when a CRO is used?

2. (a) Draw the possible methods of connecting the pressure coil of a wattmeter and
compare the errors. Explain the meaning of compensating winding in a wattmeter and
show how they help to reduce the error. [5]
(b) Sketch and explain the working of moving-coil instrument. [5]

3. (a) What are the different testing conducted on a single phase energy meter? [5]
(b)The meter constant of 230V, 10A energy meter is 1700. The meter is tested under half
load and rated voltage at unity p.f. The meter is found to make 80 revolutions in 138 sec.
Find % error. [5]

4. Differentiate between a C.T. and P.T. Discuss the theory of a P.T with phasor diagrams.
Derive expression for actual transformation ratio, ratio error and phasor angle error of a
P.T. [10]
5. (a) Describe how an AC potentiometer, can be used for the calibration of wattmeter? [5]
(b)Explain how LVDT can be used for measurement of displacement. [5]
6. (a) Derive the equation of balance of a Schering Bridge. Draw the phasor diagram under
null conditions and explain how loss angle of capacitor can be calculated. [5]
(b) Describe the general working principle of a D Arsonval Galvanometer. [5]

7. (a) Explain the use of CRG in the measurement of frequency. [5]


(b)Draw the block diagram of an electronic voltmeter and explain its operation. [5]

8. Write short notes on: [5X2]


(a) Thermo couple
(b)Peizo-Electric Transducers.

6
CLASS NOTES ON ELECTRICAL MEASUREMENTS & INSTRUMENTATION 2015

Model Question Paper: Set-4


Full Marks: 70 Time: 3 hours
Answer any six questions including question No. 1 which is compulsory.
The figures in the right-hand margin indicate marks.
(Answer any six questions including Q.No. 1)
1. Answer the following questions: [2 x 10]
(a) What are the parameters on which the critical damping of galvanometer depends?
Why critical damping is important?
(b) The current flowing through a resistance of 10.281 k is measured as 1.217 mA.
Calculate the voltage drop across the resistor to the appropriate number of significant
errors.
(c) What are the main advantages and disadvantages of PMMC instruments?
(d) Why an electrodynamometer type instrument is called a Universal Instrument?
(e) Write the working principle of resonance type frequency meter.
(f) What are the advantages of electronic voltmeter over electro mechanical type
voltmeter?
(g) Why Maxwell Bridge is limited to the measurement of medium Q coils?
(h) What will happen in a current transformer, if the secondary circuit is accidentally
opened while the primary winding is energized?
(i) Suggest a transducer for the measurement of displacement in the order of one tenth
of millimeter and write the basic principle of measurement.
(j) What is the function of delay line in oscilloscope?

Q. 2. [5 + 5]

a) The coil of a measuring instrument has a resistance of 1 and the instrument has a full
scale deflection of 250 V when a resistance of 4999 is connected with it. Find the
current range of the instrument when used as an ammeter with the coil connected across a
shunt of (1/499) and the value of the shunt resistance for the instrument to give a full
scale deflection of 50 A.
b) Distinguish between gross error, systematic error and random error with examples. What
are the methods for their elimination/reduction?

Q. 3. [5 + 5]

a) Draw the circuit diagram of Schering Bridge. Derive the conditions for balancing the
bridge and draw the phasor diagram during balanced condition.
b) Describe the theory and method of measurement of low resistance using Kelvins double
Bridge. How the effect of thermo electric emf is taken into account during
measurement?

7
CLASS NOTES ON ELECTRICAL MEASUREMENTS & INSTRUMENTATION 2015

Q. 4. [10]

A single range student type potentiometer has 20 step dial switch where each step
represents 0.1 V. The dial resistors are 20 . The slide wire of the potentiometer is
circular and has 10 turns and a resistance of 10 each. The slide wire has 200 divisions
and interpolation can be done to one fourth of a division. The working battery has a
voltage of 10 V and negligible internal resistance. Draw the circuit diagram and calculate
i) The measuring range of potentiometer
ii) The resolution
iii) Working current
iv) Resistance of series rheostat

Q. 5. [5 + 5]

a) What is the requirement of Screening of bridge components? Draw the circuit diagram
of Wagners earthing device and explain its operation.
b) Define the sensitivity of a strain gauge. Draw the circuit for measurement of strain and
derive the expression of output voltage in terms of strain.

Q. 6. [5 + 5]

a) Derive the torque equation of moving iron instrument and comment on the shape of the
scale.
b) Prove that for electrodynamometer type wattmeter
true power = {cos / [cos cos ( ]} x actual wattmeter reading
Where cos = power factor of the circuit
= tan-1 (L/R) where L and R are the inductance and resistance of the pressure
coil of the circuit.
Q. 7. [5 + 5]

a) Describe the construction and principle of operation of D Arsonval type Galvanometer.


b) Discuss the major sources of errors in current transformer. What are the means to reduce
errors in CT? Explain design and constructional feature to reduce the error.

Q. 8. [6 + 4]

a) Describe the measurement of frequency, phase angle and time delay using oscilloscope
with suitable diagrams and mathematical expressions.
b) With block diagram explain the operation of Ramp type digital voltmeter.

8
CLASS NOTES ON ELECTRICAL MEASUREMENTS & INSTRUMENTATION 2015

MEASURING INSTRUMENTS

1.1 Definition of instruments


An instrument is a device in which we can determine the magnitude or value of the
quantity to be measured. The measuring quantity can be voltage, current, power and energy etc.
Generally instruments are classified in to two categories.

Instrument

Absolute Instrument Secondary Instrument

1.2 Absolute instrument

An absolute instrument determines the magnitude of the quantity to be measured in terms of the
instrument parameter. This instrument is really used, because each time the value of the
measuring quantities varies. So we have to calculate the magnitude of the measuring quantity,
analytically which is time consuming. These types of instruments are suitable for laboratory use.
Example: Tangent galvanometer.

1.3 Secondary instrument

This instrument determines the value of the quantity to be measured directly. Generally these
instruments are calibrated by comparing with another standard secondary instrument.
Examples of such instruments are voltmeter, ammeter and wattmeter etc. Practically
secondary instruments are suitable for measurement.

Secondary instruments

Indicating instruments Recording Integrating Electromechanically


Indicating instruments

9
CLASS NOTES ON ELECTRICAL MEASUREMENTS & INSTRUMENTATION 2015

1.3.1 Indicating instrument

This instrument uses a dial and pointer to determine the value of measuring quantity. The pointer
indication gives the magnitude of measuring quantity.

1.3.2 Recording instrument

This type of instruments records the magnitude of the quantity to be measured continuously over
a specified period of time.

1.3.3 Integrating instrument

This type of instrument gives the total amount of the quantity to be measured over a specified
period of time.
1.3.4 Electromechanical indicating instrument

For satisfactory operation electromechanical indicating instrument, three forces are necessary.
They are
(a) Deflecting force

(b) Controlling force

(c)Damping force

1.4 Deflecting force

When there is no input signal to the instrument, the pointer will be at its zero position. To deflect
the pointer from its zero position, a force is necessary which is known as deflecting force. A
system which produces the deflecting force is known as a deflecting system. Generally a
deflecting system converts an electrical signal to a mechanical force.

Fig. 1.1 Pointer scale

10
CLASS NOTES ON ELECTRICAL MEASUREMENTS & INSTRUMENTATION 2015

1.4.1 Magnitude effect

When a current passes through the coil (Fig.1.2), it produces a imaginary bar magnet. When a
soft-iron piece is brought near this coil it is magnetized. Depending upon the current direction
the poles are produced in such a way that there will be a force of attraction between the coil and
the soft iron piece. This principle is used in moving iron attraction type instrument.

Fig. 1.2

If two soft iron pieces are place near a current carrying coil there will be a force of repulsion
between the two soft iron pieces. This principle is utilized in the moving iron repulsion type
instrument.

1.4.2 Force between a permanent magnet and a current carrying coil

When a current carrying coil is placed under the influence of magnetic field produced by a
permanent magnet and a force is produced between them. This principle is utilized in the moving
coil type instrument.

Fig. 1.3

1.4.3 Force between two current carrying coil

When two current carrying coils are placed closer to each other there will be a force of repulsion
between them. If one coil is movable and other is fixed, the movable coil will move away from
the fixed one. This principle is utilized in electrodynamometer type instrument.

11
CLASS NOTES ON ELECTRICAL MEASUREMENTS & INSTRUMENTATION 2015

Fig. 1.4

1.5 Controlling force

To make the measurement indicated by the pointer definite (constant) a force is necessary which
will be acting in the opposite direction to the deflecting force. This force is known as controlling
force. A system which produces this force is known as a controlled system. When the external
signal to be measured by the instrument is removed, the pointer should return back to the zero
position. This is possibly due to the controlling force and the pointer will be indicating a steady
value when the deflecting torque is equal to controlling torque.

Td = Tc (1.1)

1.5.1 Spring control

Two springs are attached on either end of spindle (Fig. 1.5).The spindle is placed in jewelled
bearing, so that the frictional force between the pivot and spindle will be minimum. Two springs
are provided in opposite direction to compensate the temperature error. The spring is made of
phosphorous bronze.

When a current is supply, the pointer deflects due to rotation of the spindle. While spindle is
rotate, the spring attached with the spindle will oppose the movements of the pointer. The torque
produced by the spring is directly proportional to the pointer deflection .

TC (1.2)

The deflecting torque produced Td proportional to I. When TC = Td , the pointer will come to a
steady position. Therefore
I (1.3)

12
CLASS NOTES ON ELECTRICAL MEASUREMENTS & INSTRUMENTATION 2015

Fig. 1.5

Since, and I are directly proportional to the scale of such instrument which uses spring
controlled is uniform.
1.6 Damping force

The deflection torque and controlling torque produced by systems are electro mechanical.
Due to inertia produced by this system, the pointer oscillates about it final steady position before
coming to rest. The time required to take the measurement is more. To damp out the oscillation
is quickly, a damping force is necessary. This force is produced by different systems.

(a) Air friction damping


(b) Fluid friction damping
(c) Eddy current damping
1.6.1 Air friction damping

The piston is mechanically connected to a spindle through the connecting rod (Fig. 1.6). The
pointer is fixed to the spindle moves over a calibrated dial. When the pointer oscillates in
clockwise direction, the piston goes inside and the cylinder gets compressed. The air pushes the
piston upwards and the pointer tends to move in anticlockwise direction.

13
CLASS NOTES ON ELECTRICAL MEASUREMENTS & INSTRUMENTATION 2015

Fig. 1.6

If the pointer oscillates in anticlockwise direction the piston moves away and the pressure of the
air inside cylinder gets reduced. The external pressure is more than that of the internal pressure.
Therefore the piston moves down wards. The pointer tends to move in clock wise direction.

1.6.2 Eddy current damping

Fig. 1.6 Disc type

An aluminum circular disc is fixed to the spindle (Fig. 1.6). This disc is made to move in the
magnetic field produced by a permanent magnet.

14
CLASS NOTES ON ELECTRICAL MEASUREMENTS & INSTRUMENTATION 2015

When the disc oscillates it cuts the magnetic flux produced by damping magnet. An emf is
induced in the circular disc by faradays law. Eddy currents are established in the disc since it has
several closed paths. By Lenzs law, the current carrying disc produced a force in a direction
opposite to oscillating force. The damping force can be varied by varying the projection of the
magnet over the circular disc.

Fig. 1.6 Rectangular type

1.7 Permanent Magnet Moving Coil (PMMC) instrument


One of the most accurate type of instrument used for D.C. measurements is PMMC instrument.
Construction: A permanent magnet is used in this type instrument. Aluminum former is
provided in the cylindrical in between two poles of the permanent magnet (Fig. 1.7). Coils are
wound on the aluminum former which is connected with the spindle. This spindle is supported
with jeweled bearing. Two springs are attached on either end of the spindle. The terminals of the
moving coils are connected to the spring. Therefore the current flows through spring 1, moving
coil and spring 2.

Damping: Eddy current damping is used. This is produced by aluminum former.


Control: Spring control is used.

15
CLASS NOTES ON ELECTRICAL MEASUREMENTS & INSTRUMENTATION 2015

Fig. 1.7

Principle of operation
When D.C. supply is given to the moving coil, D.C. current flows through it. When the current
carrying coil is kept in the magnetic field, it experiences a force. This force produces a torque
and the former rotates. The pointer is attached with the spindle. When the former rotates, the
pointer moves over the calibrated scale. When the polarity is reversed a torque is produced in the
opposite direction. The mechanical stopper does not allow the deflection in the opposite
direction. Therefore the polarity should be maintained with PMMC instrument.
If A.C. is supplied, a reversing torque is produced. This cannot produce a continuous deflection.
Therefore this instrument cannot be used in A.C.

Torque developed by PMMC

Let Td =deflecting torque

TC = controlling torque
= angle of deflection
K=spring constant
b=width of the coil

16
CLASS NOTES ON ELECTRICAL MEASUREMENTS & INSTRUMENTATION 2015

l=height of the coil or length of coil


N=No. of turns
I=current
B=Flux density
A=area of the coil
The force produced in the coil is given by
F = BIL sin (1.4)

When = 90
For N turns, F = NBIL (1.5)
Torque produced Td = F r distance (1.6)

Td = NBIL b = BINA (1.7)

Td = BANI (1.8)

Td I (1.9)

Advantages
Torque/weight is high
Power consumption is less
Scale is uniform
Damping is very effective
Since operating field is very strong, the effect of stray field is negligible
Range of instrument can be extended
Disadvantages
Use only for D.C.
Cost is high
Error is produced due to ageing effect of PMMC
Friction and temperature error are present

17
CLASS NOTES ON ELECTRICAL MEASUREMENTS & INSTRUMENTATION 2015

1.7.1 Extension of range of PMMC instrument


Case-I: Shunt
A low shunt resistance connected in parrel with the ammeter to extent the range of current. Large
current can be measured using low current rated ammeter by using a shunt.

Fig. 1.8

Let Rm =Resistance of meter

Rsh =Resistance of shunt

I m = Current through meter

I sh =current through shunt


I= current to be measure
Vm = Vsh (1.10)

I m Rm = I sh Rsh

I m Rsh
= (1.11)
I sh Rm

Apply KCL at P I = I m + I sh (1.12)

Eqn (1.12) by I m

I I
= 1 + sh (1.13)
Im Im

18
CLASS NOTES ON ELECTRICAL MEASUREMENTS & INSTRUMENTATION 2015

I R
=1+ m (1.14)
Im Rsh

R
I = I m 1 + m (1.15)
Rsh
R
1 + m is called multiplication factor
Rsh
Shunt resistance is made of manganin. This has least thermoelectric emf. The change is
resistance, due to change in temperature is negligible.

Case (II): Multiplier

A large resistance is connected in series with voltmeter is called multiplier (Fig. 1.9). A large
voltage can be measured using a voltmeter of small rating with a multiplier.

Fig. 1.9
Let Rm =resistance of meter

Rse =resistance of multiplier

Vm =Voltage across meter

Vse = Voltage across series resistance


V= voltage to be measured
I m = I se (1.16)

Vm Vse
= (1.17)
Rm Rse

V R
se = se (1.18)
Vm Rm

19
CLASS NOTES ON ELECTRICAL MEASUREMENTS & INSTRUMENTATION 2015

Apply KVL, V = Vm + Vse (1.19)

Eqn (1.19) Vm

V V R
= 1 + se = 1 + se (1.20)
Vm Vm Rm

R
V = Vm 1 + se (1.21)
Rm

Rse
1 + Multiplication factor
Rm

1.8 Moving Iron (MI) instruments


One of the most accurate instrument used for both AC and DC measurement is moving iron
instrument. There are two types of moving iron instrument.
Attraction type
Repulsion type
1.8.1 Attraction type M.I. instrument
Construction: The moving iron fixed to the spindle is kept near the hollow fixed coil (Fig. 1.10).
The pointer and balance weight are attached to the spindle, which is supported with jeweled
bearing. Here air friction damping is used.

Principle of operation
The current to be measured is passed through the fixed coil. As the current is flow through the
fixed coil, a magnetic field is produced. By magnetic induction the moving iron gets magnetized.
The north pole of moving coil is attracted by the south pole of fixed coil. Thus the deflecting
force is produced due to force of attraction. Since the moving iron is attached with the spindle,
the spindle rotates and the pointer moves over the calibrated scale. But the force of attraction
depends on the current flowing through the coil.

Torque developed by M.I


Let be the deflection corresponding to a current of i amp
Let the current increases by di, the corresponding deflection is + d

20
CLASS NOTES ON ELECTRICAL MEASUREMENTS & INSTRUMENTATION 2015

Fig. 1.10
There is change in inductance since the position of moving iron change w.r.t the fixed
electromagnets.
Let the new inductance value be L+dL. The current change by di is dt seconds.
Let the emf induced in the coil be e volt.
d di dL
e= ( Li ) = L + i (1.22)
dt dt dt
Multiplying by idt in equation (1.22)
di dL
e idt = L idt + i idt (1.23)
dt dt

e idt = Lidi + i 2 dL (1.24)

Eqn (1.24) gives the energy is used in to two forms. Part of energy is stored in the inductance.
Remaining energy is converted in to mechanical energy which produces deflection.

Fig. 1.11

21
CLASS NOTES ON ELECTRICAL MEASUREMENTS & INSTRUMENTATION 2015

Change in energy stored=Final energy-initial energy stored


1 1
= ( L + dL)(i + di) 2 Li 2
2 2
1
= {( L + dL)(i 2 + di 2 + 2idi) Li 2 }
2
1
= {( L + dL)(i 2 + 2idi) Li 2 }
2
1
= {Li 2 + 2 Lidi + i 2 dL + 2ididL Li 2 }
2
1
= {2 Lidi + i 2 dL}
2
1
= Lidi + i 2 dL (1.25)
2
Mechanical work to move the pointer by d
= Td d (1.26)
By law of conservation of energy,
Electrical energy supplied=Increase in stored energy+ mechanical work done.
Input energy= Energy stored + Mechanical energy
1
Lidi + i 2 dL = Lidi + i 2 dL + Td d (1.27)
2
1 2
i dL = Td d (1.28)
2
1 dL
Td = i 2 (1.29)
2 d
At steady state condition Td = TC

1 2 dL
i = K (1.30)
2 d
1 2 dL
= i (1.31)
2 K d
i2 (1.32)
When the instruments measure AC, i 2 rms
Scale of the instrument is non uniform.

22
CLASS NOTES ON ELECTRICAL MEASUREMENTS & INSTRUMENTATION 2015

Advantages
MI can be used in AC and DC
It is cheap
Supply is given to a fixed coil, not in moving coil.
Simple construction
Less friction error.
Disadvantages
It suffers from eddy current and hysteresis error
Scale is not uniform
It consumed more power
Calibration is different for AC and DC operation
1.8.2 Repulsion type moving iron instrument
Construction: The repulsion type instrument has a hollow fixed iron attached to it (Fig. 1.12).
The moving iron is connected to the spindle. The pointer is also attached to the spindle in
supported with jeweled bearing.
Principle of operation: When the current flows through the coil, a magnetic field is produced by
it. So both fixed iron and moving iron are magnetized with the same polarity, since they are kept
in the same magnetic field. Similar poles of fixed and moving iron get repelled. Thus the
deflecting torque is produced due to magnetic repulsion. Since moving iron is attached to
spindle, the spindle will move. So that pointer moves over the calibrated scale.
Damping: Air friction damping is used to reduce the oscillation.
Control: Spring control is used.

23
CLASS NOTES ON ELECTRICAL MEASUREMENTS & INSTRUMENTATION 2015

Fig. 1.12

1.9 Dynamometer (or) Electromagnetic moving coil instrument (EMMC)

Fig. 1.13

24
CLASS NOTES ON ELECTRICAL MEASUREMENTS & INSTRUMENTATION 2015

This instrument can be used for the measurement of voltage, current and power. The difference
between the PMMC and dynamometer type instrument is that the permanent magnet is replaced
by an electromagnet.

Construction: A fixed coil is divided in to two equal half. The moving coil is placed between the
two half of the fixed coil. Both the fixed and moving coils are air cored. So that the hysteresis
effect will be zero. The pointer is attached with the spindle. In a non metallic former the moving
coil is wounded.
Control: Spring control is used.
Damping: Air friction damping is used.
Principle of operation:

When the current flows through the fixed coil, it produced a magnetic field, whose flux density is
proportional to the current through the fixed coil. The moving coil is kept in between the fixed
coil. When the current passes through the moving coil, a magnetic field is produced by this coil.

The magnetic poles are produced in such a way that the torque produced on the moving coil
deflects the pointer over the calibrated scale. This instrument works on AC and DC. When AC
voltage is applied, alternating current flows through the fixed coil and moving coil. When the
current in the fixed coil reverses, the current in the moving coil also reverses. Torque remains in
the same direction. Since the current i1 and i2 reverse simultaneously. This is because the fixed
and moving coils are either connected in series or parallel.

Torque developed by EMMC

Fig. 1.14

25
CLASS NOTES ON ELECTRICAL MEASUREMENTS & INSTRUMENTATION 2015

Let
L1=Self inductance of fixed coil
L2= Self inductance of moving coil
M=mutual inductance between fixed coil and moving coil
i1=current through fixed coil
i2=current through moving coil
Total inductance of system,
Ltotal = L1 + L2 + 2M (1.33)
But we know that in case of M.I
1 2 d ( L)
Td = i (1.34)
2 d
1 d
Td = i 2 ( L1 + L2 + 2 M ) (1.35)
2 d

The value of L1 and L2 are independent of but M varies with


1 dM
Td = i 2 2 (1.36)
2 d
dM
Td = i 2 (1.37)
d
If the coils are not connected in series i1 i2
dM
Td = i1i2 (1.38)
d
TC = Td (1.39)

i1i2 dM (1.40)
=
K d

Hence the deflection of pointer is proportional to the current passing through fixed coil and
moving coil.

26
CLASS NOTES ON ELECTRICAL MEASUREMENTS & INSTRUMENTATION 2015

1.9.1 Extension of EMMC instrument


Case-I Ammeter connection
Fixed coil and moving coil are connected in parallel for ammeter connection. The coils are
designed such that the resistance of each branch is same.
Therefore
I1 = I 2 = I

Fig. 1.15

To extend the range of current a shunt may be connected in parallel with the meter. The value
Rsh is designed such that equal current flows through moving coil and fixed coil.

dM
Td = I1I 2 (1.41)
d
dM
Or Td = I 2 (1.42)
d
TC = K (1.43)

I 2 dM
= (1.44)
K d

I 2 (Scale is not uniform) (1.45)

Case-II Voltmeter connection

Fixed coil and moving coil are connected in series for voltmeter connection. A multiplier may be
connected in series to extent the range of voltmeter.

27
CLASS NOTES ON ELECTRICAL MEASUREMENTS & INSTRUMENTATION 2015

Fig. 1.16
V V
I1 = 1 , I 2 = 2 (1.46)
Z1 Z2

V V dM
Td = 1 2 (1.47)
Z1 Z 2 d

K V K V dM
Td = 1 2 (1.48)
Z1 Z2 d

KV 2 dM
Td = (1.49)
Z1Z 2 d

Td V 2 (1.50)

V 2 (Scale in not uniform) (1.51)

Case-III As wattmeter

When the two coils are connected to parallel, the instrument can be used as a wattmeter. Fixed
coil is connected in series with the load. Moving coil is connected in parallel with the load. The
moving coil is known as voltage coil or pressure coil and fixed coil is known as current coil.

Fig. 1.17

28
CLASS NOTES ON ELECTRICAL MEASUREMENTS & INSTRUMENTATION 2015

Assume that the supply voltage is sinusoidal. If the impedance of the coil is neglected in
comparison with the resistance R. The current,
v sin wt
I2 = m (1.52)
R
Let the phase difference between the currents I1 and I2 is

I1 = I m sin(wt ) (1.53)

dM
Td = I1I 2 (1.54)
d
V sin wt dM
Td = I m sin( wt ) m (1.55)
R d
1 dM
Td = ( I mVm sin wt sin( wt )) (1.56)
R d
1 dM
Td = I mVm sin wt. sin( wt ) (1.57)
R d
The average deflecting torque

1 2
(Td ) avg = Td d (wt ) (1.58)
2 0
2
1 1 dM
(Td ) avg = RI V sin wt.sin( wt ) d (wt ) (1.59)
d
m m
2 0

Vm I m 1 dM
(Td ) avg = {cos cos(2 wt )}dwt (1.60)
2 2 R d

Vm I m dM
2 2

(Td ) avg = cos .dwt cos(2wt ).dwt (1.61)
4R d 0 0

(Td ) avg =
Vm I m dM

4 R d
[
cos [wt ]0
2
] (1.62)

Vm I m dM
(Td ) avg = [cos (2 0)] (1.63)
4 R d
Vm I m 1 dM
(Td ) avg = cos (1.64)
2 R d
1 dM
(Td ) avg = Vrms I rms cos (1.65)
R d

29
CLASS NOTES ON ELECTRICAL MEASUREMENTS & INSTRUMENTATION 2015

(Td ) avg KVI cos (1.66)

TC (1.67)

KVI cos (1.68)


VI cos (1.69)
Advantages
It can be used for voltmeter, ammeter and wattmeter
Hysteresis error is nill
Eddy current error is nill
Damping is effective
It can be measure correctively and accurately the rms value of the voltage

Disadvantages
Scale is not uniform
Power consumption is high(because of high resistance )
Cost is more
Error is produced due to frequency, temperature and stray field.
Torque/weight is low.(Because field strength is very low)

Errors in PMMC
The permanent magnet produced error due to ageing effect. By heat treatment, this error
can be eliminated.
The spring produces error due to ageing effect. By heat treating the spring the error can
be eliminated.
When the temperature changes, the resistance of the coil vary and the spring also
produces error in deflection. This error can be minimized by using a spring whose
temperature co-efficient is very low.
1.10 Difference between attraction and repulsion type instrument
An attraction type instrument will usually have a lower inductance, compare to repulsion type
instrument. But in other hand, repulsion type instruments are more suitable for economical
production in manufacture and nearly uniform scale is more easily obtained. They are therefore
much more common than attraction type.

30
CLASS NOTES ON ELECTRICAL MEASUREMENTS & INSTRUMENTATION 2015

1.11 Characteristics of meter


1.11.1 Full scale deflection current( I FSD )
The current required to bring the pointer to full-scale or extreme right side of the
instrument is called full scale deflection current. It must be as small as possible. Typical value is
between 2 A to 30mA.
1.11.2 Resistance of the coil( Rm )

This is ohmic resistance of the moving coil. It is due to , L and A. For an ammeter this should
be as small as possible.
1.11.3 Sensitivity of the meter(S)
1 Z
S= ( / volt ), S =
I FSD V
It is also called ohms/volt rating of the instrument. Larger the sensitivity of an instrument, more
accurate is the instrument. It is measured in /volt. When the sensitivity is high, the impedance
of meter is high. Hence it draws less current and loading affect is negligible. It is also defend as
one over full scale deflection current.
1.12 Error in M.I instrument
1.12.1 Temperature error
Due to temperature variation, the resistance of the coil varies. This affects the deflection of the
instrument. The coil should be made of manganin, so that the resistance is almost constant.

1.12.2 Hysteresis error


Due to hysteresis affect the reading of the instrument will not be correct. When the current is
decreasing, the flux produced will not decrease suddenly. Due to this the meter reads a higher
value of current. Similarly when the current increases the meter reads a lower value of current.
This produces error in deflection. This error can be eliminated using small iron parts with narrow
hysteresis loop so that the demagnetization takes place very quickly.

1.12.3 Eddy current error


The eddy currents induced in the moving iron affect the deflection. This error can be reduced by
increasing the resistance of the iron.

31
CLASS NOTES ON ELECTRICAL MEASUREMENTS & INSTRUMENTATION 2015

1.12.4 Stray field error


Since the operating field is weak, the effect of stray field is more. Due to this, error is produced
in deflection. This can be eliminated by shielding the parts of the instrument.

1.12.5 Frequency error


When the frequency changes the reactance of the coil changes.

Z = ( Rm + RS ) 2 + X L2 (1.70)

V V
I= = (1.71)
Z ( Rm + RS ) 2 + X L2

Fig. 1.18

Deflection of moving iron voltmeter depends upon the current through the coil. Therefore,
deflection for a given voltage will be less at higher frequency than at low frequency. A capacitor
is connected in parallel with multiplier resistance. The net reactance, ( X L X C ) is very small,
when compared to the series resistance. Thus the circuit impedance is made independent of
frequency. This is because of the circuit is almost resistive.
L
C = 0.41 (1.72)
( RS ) 2
1.13 Electrostatic instrument
In multi cellular construction several vans and quadrants are provided. The voltage is to be
measured is applied between the vanes and quadrant. The force of attraction between the vanes

32
CLASS NOTES ON ELECTRICAL MEASUREMENTS & INSTRUMENTATION 2015

and quadrant produces a deflecting torque. Controlling torque is produced by spring control. Air
friction damping is used.

The instrument is generally used for measuring medium and high voltage. The voltage is reduced
to low value by using capacitor potential divider. The force of attraction is proportional to the
square of the voltage.

Fig. 1.19
Torque develop by electrostatic instrument
V=Voltage applied between vane and quadrant
C=capacitance between vane and quadrant
1
Energy stored= CV 2 (1.73)
2
Let be the deflection corresponding to a voltage V.
Let the voltage increases by dv, the corresponding deflection is + d
When the voltage is being increased, a capacitive current flows
dq d (CV ) dC dV
i= = = V +C (1.74)
dt dt dt dt
V dt multiply on both side of equation (1.74)

33
CLASS NOTES ON ELECTRICAL MEASUREMENTS & INSTRUMENTATION 2015

Fig. 1.20

dC 2 dV
Vidt = V dt + CV dt (1.75)
dt dt

Vidt = V 2 dC + CVdV (1.76)


1 1
Change in stored energy= (C + dC )(V + dV )2 CV 2 (1.77)
2 2

=
1
2
[ ] 1
(C + dC )V 2 + dV 2 + 2VdV CV 2
2
1
2
[ 1
]
= CV 2 + CdV 2 + 2CVdV + V 2 dC + dCdV 2 + 2VdVdC CV 2
2
1
= V 2 dC + CVdV
2
1
V 2 dC + CVdV = V 2 dC + CVdV + F rd (1.78)
2
1
Td d = V 2 dC (1.79)
2

1 dC
Td = V 2 (1.80)
2 d

At steady state condition, Td = TC

1 dC
K = V 2 (1.81)
2 d

1 2 dC
= V (1.82)
2K d

34
CLASS NOTES ON ELECTRICAL MEASUREMENTS & INSTRUMENTATION 2015

Advantages
It is used in both AC and DC.
There is no frequency error.
There is no hysteresis error.
There is no stray magnetic field error. Because the instrument works on electrostatic
principle.
It is used for high voltage
Power consumption is negligible.
Disadvantages
Scale is not uniform
Large in size
Cost is more
1.14 Multi range Ammeter
When the switch is connected to position (1), the supplied current I1

Fig. 1.21

I sh1Rsh1 = I m Rm (1.83)
I R I R
Rsh1 = m m = m m (1.84)
I sh1 I1 I m

35
CLASS NOTES ON ELECTRICAL MEASUREMENTS & INSTRUMENTATION 2015

Rm Rm I
Rsh1 = , Rsh1 = , m1 = 1 = Multiplying power of shunt
I1
1 m1 1 Im
Im
Rm I
Rsh 2 = , m2 = 2 (1.85)
m2 1 Im

Rm I
Rsh3 = , m3 = 3 (1.86)
m3 1 Im

Rm I
Rsh 4 = , m4 = 4 (1.87)
m4 1 Im

1.15 Ayrton shunt


R1 = Rsh1 Rsh2 (1.88)

R2 = Rsh2 Rsh3 (1.89)

R3 = Rsh3 Rsh4 (1.90)

R4 = Rsh 4 (1.91)

Fig. 1.22
Ayrton shunt is also called universal shunt. Ayrton shunt has more sections of resistance. Taps
are brought out from various points of the resistor. The variable points in the o/p can be
connected to any position. Various meters require different types of shunts. The Aryton shunt is
used in the lab, so that any value of resistance between minimum and maximum specified can be
used. It eliminates the possibility of having the meter in the circuit without a shunt.

36
CLASS NOTES ON ELECTRICAL MEASUREMENTS & INSTRUMENTATION 2015

1.16 Multi range D.C. voltmeter

Fig. 1.23

Rs1 = Rm ( m1 1)
Rs 2 = Rm (m2 1) (1.92)
Rs3 = Rm (m3 1)
V V V
m1 = 1 , m2 = 2 , m3 = 3 (1.93)
Vm Vm Vm

We can obtain different Voltage ranges by connecting different value of multiplier resistor in
series with the meter. The number of these resistors is equal to the number of ranges required.

1.17 Potential divider arrangement


The resistance R1, R2 , R3 and R4 is connected in series to obtained the ranges V1,V2 ,V3 and V4

37
CLASS NOTES ON ELECTRICAL MEASUREMENTS & INSTRUMENTATION 2015

Fig. 1.24
Consider for voltage V1, ( R1 + Rm ) I m = V1

V V V
R1 = 1 Rm = 1 Rm = 1 Rm Rm (1.94)
Im V Vm
( m)
Rm

R1 = (m1 1) Rm (1.95)

V
For V2 , ( R2 + R1 + Rm ) I m = V2 R2 = 2 R1 Rm (1.96)
Im

V2
R2 = (m1 1) Rm Rm (1.97)
Vm

Rm
R2 = m2 Rm Rm (m1 1) Rm

= Rm (m2 1 m1 + 1) (1.98)

R2 = (m2 m1) Rm (1.99)

For V3 (R3 + R2 + R1 + Rm )I m = V3

V
R3 = 3 R2 R1 Rm
Im

V
= 3 Rm (m2 m1 ) Rm (m1 1) Rm Rm
Vm
= m3 Rm (m2 m1 ) Rm (m1 1) Rm Rm

R3 = (m3 m2 ) Rm

38
CLASS NOTES ON ELECTRICAL MEASUREMENTS & INSTRUMENTATION 2015

For V4 (R4 + R3 + R2 + R1 + Rm )I m = V4
V
R4 = 4 R3 R2 R1 Rm
Im

V
= 4 Rm (m3 m2 ) Rm (m2 m1) Rm (m1 1) Rm Rm
Vm
R4 = Rm [m4 m3 + m2 m2 + m1 m1 + 1 1]
R4 = (m4 m3 )Rm

Example: 1.1
A PMMC ammeter has the following specification
Coil dimension are 1cm 1cm. Spring constant is 0.15 10 6 N m / rad , Flux density is

1.5 10 3 wb / m 2 .Determine the no. of turns required to produce a deflection of 900 when a current
2mA flows through the coil.

Solution:
At steady state condition Td = TC
BANI = K
K
N=
BAI

A= 1 104 m 2
N m
K= 0.15 10 6
rad

B= 1.5 10 3 wb / m 2

I= 2 10 3 A

= 90 = rad
2
N=785 ans.
Example: 1.2

39
CLASS NOTES ON ELECTRICAL MEASUREMENTS & INSTRUMENTATION 2015

The pointer of a moving coil instrument gives full scale deflection of 20mA. The potential
difference across the meter when carrying 20mA is 400mV.The instrument to be used is 200A
for full scale deflection. Find the shunt resistance required to achieve this, if the instrument to be
used as a voltmeter for full scale reading with 1000V. Find the series resistance to be connected
it?
Solution:
Case-1
Vm =400mV

I m = 20mA
I=200A
V 400
Rm = m = = 20
Im 20

R
I = I m 1 + m
Rsh
20
200 = 20 103 1 +
Rsh

Rsh = 2 10 3

Case-II
V=1000V
R
V = Vm 1 + se
Rm

R
4000 = 400 10 3 1 + se
20

Rse = 49.98k

Example: 1.3
A 150 v moving iron voltmeter is intended for 50HZ, has a resistance of 3k. Find the series
resistance required to extent the range of instrument to 300v. If the 300V instrument is used to
measure a d.c. voltage of 200V. Find the voltage across the meter?
Solution:
Rm = 3k , Vm = 150V , V = 300V

40
CLASS NOTES ON ELECTRICAL MEASUREMENTS & INSTRUMENTATION 2015

R se
V = Vm 1 +
Rm

R
300 = 1501 + se Rse = 3k
3

R
Case-II V = Vm 1 + se
Rm

3
200 = Vm 1 +
3
Vm = 100V Ans

Example: 1.4
What is the value of series resistance to be used to extent 0to 200V range of 20,000/volt
voltmeter to 0 to 2000 volt?

Solution:
Vse = V V = 1800
1 1
I FSD = =
20000 Sensitivity
Vse = Rse iFSD Rse = 36M ans.

Example: 1.5
A moving coil instrument whose resistance is 25 gives a full scale deflection with a current of
1mA. This instrument is to be used with a manganin shunt, to extent its range to 100mA.
Calculate the error caused by a 100C rise in temperature when:
(a) Copper moving coil is connected directly across the manganin shunt.
(b) A 75 ohm manganin resistance is used in series with the instrument moving coil.
The temperature co-efficient of copper is 0.004/0C and that of manganin is 0.000150/C.
Solution:
Case-1
I m = 1mA

Rm = 25

41
CLASS NOTES ON ELECTRICAL MEASUREMENTS & INSTRUMENTATION 2015

I=100mA
R
I = I m 1 + m
Rsh

25 25
100 = 11 + = 99
Rsh Rsh
25
Rsh = = 0.2525
99

Instrument resistance for 100C rise in temperature, Rmt = 25(1 + 0.004 10)

Rt = Ro (1 + t t )

Rm / t =10 = 26

Shunt resistance for 100C, rise in temperature

Rsh / t =10 = 0.2525(1 + 0.00015 10) = 0.2529

Current through the meter for 100mA in the main circuit for 100C rise in temperature
Rm
I = I m 1 +
Rsh t =10 C


26
100 = I mt 1 +
0.2529
Im = 0.963mA
t =10

But normal meter current=1mA


Error due to rise in temperature=(0.963-1)*100=-3.7%
Case-b As voltmeter
Total resistance in the meter circuit= Rm + Rsh = 25 + 75 = 100

R
I = I m 1 + m
Rsh

100
100 = 11 +
Rsh

100
Rsh = = 1.01
100 1

42
CLASS NOTES ON ELECTRICAL MEASUREMENTS & INSTRUMENTATION 2015

Resistance of the instrument circuit for 100C rise in temperature


Rm = 25(1 + 0.004 10) + 75(1 + 0.00015 10) = 101 .11
t =10

Shunt resistance for 100C rise in temperature


Rsh = 1.01(1 + 0.00015 10) = 1.0115
t =10

R
I = I m 1 + m
Rsh
101.11
100 = I m 1 +
1.0115

I m t = 10 = 0.9905mA

Error =(0.9905-1)*100=-0.95%

Example: 1.6
The coil of a 600V M.I meter has an inductance of 1 henery. It gives correct reading at 50HZ and
requires 100mA. For its full scale deflection, what is % error in the meter when connected to
200V D.C. by comparing with 200V A.C?

Solution:
Vm = 600V , I m = 100mA
Case-I A.C.
V 600
Zm = m = = 6000
I m 0 .1

X L = 2fL = 314

Rm = Z m 2 X L2 = (6000) 2 (314) 2 = 5990

V 200
I AC = AC = = 33.33mA
Z 6000
Case-II D.C
V 200
I DC = DC = = 33.39mA
Rm 5990

43
CLASS NOTES ON ELECTRICAL MEASUREMENTS & INSTRUMENTATION 2015

I I AC 33.39 33.33
Error= DC 100 = 100 = 0.18%
I AC 33.33

Example: 1.7
A 250V M.I. voltmeter has coil resistance of 500, coil inductance 0f 1.04 H and series
resistance of 2k. The meter reads correctively at 250V D.C. What will be the value of
capacitance to be used for shunting the series resistance to make the meter read correctly at
50HZ? What is the reading of voltmeter on A.C. without capacitance?

L
Solution: C = 0.41
( RS ) 2
1.04
= 0.41 = 0.1F
(2 103 ) 2

For A.C Z = ( Rm + RSe ) 2 + X L2

Z = (500 + 2000) 2 + (314) 2 = 2520


With D.C
Rtotal = 2500
For 2500 250V
250
1
2500

250
2520 2520 = 248V
2500
Example: 1.8
The relationship between inductance of moving iron ammeter, the current and the position of
pointer is as follows:
Reading (A) 1.2 1.4 1.6 1.8
Deflection (degree) 36.5 49.5 61.5 74.5
Inductance ( H ) 575.2 576.5 577.8 578.8
Calculate the deflecting torque and the spring constant when the current is 1.5A?
Solution:
For current I=1.5A, =55.5 degree=0.96865 rad

44
CLASS NOTES ON ELECTRICAL MEASUREMENTS & INSTRUMENTATION 2015

dL 577.65 576.5
= = 0.11H / deg ree = 6.3H / rad
d 60 49.5
1 2 dL 1
Deflecting torque , Td = I = (1.5) 2 6.3 10 6 = 7.09 10 6 N m
2 d 2

T 7.09 10 6 N m
Spring constant, K = d = = 7.319 10 6
0.968 rad

Fig. 1.25
Example: 1.9
For a certain dynamometer ammeter the mutual inductance M varies with deflection as

M = 6 cos( + 30 )mH .Find the deflecting torque produced by a direct current of 50mA
corresponding to a deflection of 600.
Solution:

dM dM
Td = I1I 2 = I2
d d

M = 6 cos( + 30 )
dM
= 6 sin( + 30)mH
d
dM
= 60 = 6 sin 90 = 6mH / deg
d
dM
Td = I 2 = (50 10 3 ) 2 6 10 3 = 15 10 6 N m
d

45
CLASS NOTES ON ELECTRICAL MEASUREMENTS & INSTRUMENTATION 2015

Example: 1.10
The inductance of a moving iron ammeter with a full scale deflection of 900 at 1.5A, is given by

the expression L = 200 + 40 4 2 3H , where is deflection in radian from the zero


position. Estimate the angular deflection of the pointer for a current of 1.0A.

Solution:

L = 200 + 40 4 2 3 H
dL 2
= 40 8 3 H / rad
d = 90

dL
= 40 8 3( ) 2 H / rad = 20 H / rad
d = 90 2 2
1 2 dL
= I
2 K d

1 (1.5) 2
= 20 10 6
2 2 K

K=Spring constant=14.32 10 6 N m / rad


1 2 dL
For I=1A, = I
2 K d

=
1

(1) 2
2 14.32 10 6
(
40 8 3 2 )
3 + 36.64 2 40 = 0

= 1.008rad ,57.8
Example: 1.11

The inductance of a moving iron instrument is given by L = 10 + 5 2 3 H , where is

the deflection in radian from zero position. The spring constant is 12 10 6 N m / rad . Estimate
the deflection for a current of 5A.
Solution:

46
CLASS NOTES ON ELECTRICAL MEASUREMENTS & INSTRUMENTATION 2015

dL H
= (5 2 )
d rad
1 2 dL
= I
2 K d

1 (5) 2
= (5 2 ) 10 6
2 12 10 6

= 1.69rad ,96.8

Example: 1.12
The following figure gives the relation between deflection and inductance of a moving iron
instrument.
Deflection (degree) 20 30 40 50 60 70 80 90
Inductance ( H ) 335 345 355.5 366.5 376.5 385 391.2 396.5
Find the current and the torque to give a deflection of (a) 300 (b) 800 . Given that control spring

constant is 0.4 10 6 N m / deg ree


Solution:
1 2 dL
= I
2 K d

(a) For = 30
The curve is linear
dL 355.5 335
= = 1.075H / deg ree = 58.7 H / rad
d = 30 40 20

47
CLASS NOTES ON ELECTRICAL MEASUREMENTS & INSTRUMENTATION 2015

Fig. 1.26
Example: 1.13
In an electrostatic voltmeter the full scale deflection is obtained when the moving plate turns
through 900. The torsional constant is 10 10 6 N m / rad . The relation between the angle of
deflection and capacitance between the fixed and moving plates is given by
Deflection (degree) 0 10 20 30 40 50 60 70 80 90
Capacitance (PF) 81.4 121 156 189.2 220 246 272 294 316 334
Find the voltage applied to the instrument when the deflection is 900?
Solution:

Fig. 1.27

48
CLASS NOTES ON ELECTRICAL MEASUREMENTS & INSTRUMENTATION 2015

dC bc 370 250
= tan = = = 1.82 PF / deg ree = 104.2 PF / rad
d ab 110 44
N m
Spring constant K = 10 10 6 = 0.1745 10 6 N m / deg ree
rad

1 2 dC 2 K
= V V =
2K d dC
d

2 0.1745 10 6 90
V= = 549volt
104.2 10 12
Example: 1.14
Design a multi range d.c. mille ammeter using a basic movement with an internal resistance
Rm = 50 and a full scale deflection current I m = 1mA . The ranges required are 0-10mA; 0-50mA;

0-100mA and 0-500mA.


Solution:
Case-I 0-10mA
I 10
Multiplying power m = = = 10
Im 1
Rm 50
Shunt resistance Rsh1 = = = 5.55
m 1 10 1
Case-II 0-50mA
50
m= = 50
1
Rm 50
Rsh 2 = = = 1.03
m 1 50 1
100
Case-III 0-100mA, m = = 100
1
Rm 50
Rsh3 = = = 0.506
m 1 100 1
500
Case-IV 0-500mA, m = = 500
1
Rm 50
Rsh 4 = = = 0.1
m 1 500 1
Example: 1.15

49
CLASS NOTES ON ELECTRICAL MEASUREMENTS & INSTRUMENTATION 2015

A moving coil voltmeter with a resistance of 20 gives a full scale deflection of 1200, when a
potential difference of 100mV is applied across it. The moving coil has dimension of
30mm*25mm and is wounded with 100 turns. The control spring constant is

0.375 10 6 N m / deg ree. Find the flux density, in the air gap. Find also the diameter of copper
wire of coil winding if 30% of instrument resistance is due to coil winding. The specific

resistance for copper= 1.7 10 8 m .

Solution:
Data given
Vm = 100mV

Rm = 20

= 120
N=100

K = 0.375 10 6 N m / deg ree

RC = 30%ofRm

= 1.7 108 m
V
I m = m = 5 10 3 A
Rm

Td = BANI , TC = K = 0.375 10 6 120 = 45 10 6 N m

Td 45 10 6
B= = = 0.12 wb / m 2
6
ANI 30 25 10 100 5 10 3

RC = 0.3 20 = 6
Length of mean turn path =2(a+b) =2(55)=110mm

l
RC = N
A

N (lt ) 100 1.7 10 8 110 103


A= =
RC 6

50
CLASS NOTES ON ELECTRICAL MEASUREMENTS & INSTRUMENTATION 2015

= 3.116 108 m 2
= 31.16 103 mm 2
2
A= d d = 0.2mm
4
Example: 1.16
A moving coil instrument gives a full scale deflection of 10mA, when the potential difference
across its terminal is 100mV. Calculate

(1) The shunt resistance for a full scale deflection corresponding to 100A
(2) The resistance for full scale reading with 1000V.
Calculate the power dissipation in each case?

Solution:

Data given
I m = 10mA
Vm = 100mV
I = 100 A
Rm
I = I m 1 +
Rsh

10
100 = 10 103 1 +
Rsh
Rsh = 1.001 103

Rse = ??,V = 1000V


V 100
Rm = m = = 10
Im 10

R
V = Vm 1 + se
Rm

R
1000 = 100 10 3 1 + se
10
Rse = 99.99 K

Example: 1.17

51
CLASS NOTES ON ELECTRICAL MEASUREMENTS & INSTRUMENTATION 2015

Design an Aryton shunt to provide an ammeter with current ranges of 1A,5A,10A and 20A. A
basic meter with an internal resistance of 50w and a full scale deflection current of 1mA is to be
used.
Solution: Data given

I
m1 = 1 = 1000 A
Im
I1 = 1A I
3
m2 = 2 = 5000 A
I m = 1 10 A I2 = 5A Im
Rm = 50 I 3 = 10 A I
m3 = 3 = 10000 A
I 4 = 20 A Im
I
m4 = 4 = 20000 A
Im

Rm 50
Rsh1 = = = 0.05
m1 1 1000 1
Rm 50
Rsh 2 = = = 0.01
m2 1 5000 1
Rm 50
Rsh3 = = = 0.005
m3 1 10000 1
Rm 50
Rsh 4 = = = 0.0025
m4 1 20000 1
The resistances of the various section of the universal shunt are

R1 = Rsh1 Rsh 2 = 0.05 0.01 = 0.04


R2 = Rsh 2 Rsh3 = 0.01 0.005 = 0.005
R3 = Rsh3 Rsh 4 = 0.005 0.025 = 0.0025
R4 = Rsh 4 = 0.0025

Example: 1.18
A basic d Arsonval meter movement with an internal resistance Rm = 100 and a full scale
current of I m = 1mA is to be converted in to a multi range d.c. voltmeter with ranges of 0-10V, 0-
50V, 0-250V, 0-500V. Find the values of various resistances using the potential divider
arrangement.
Solution:
Data given

52
CLASS NOTES ON ELECTRICAL MEASUREMENTS & INSTRUMENTATION 2015

V 10
m1 = 1 = = 100
Vm 100 10 3
Rm = 100
V 50
I m = 1mA m2 = 2 = = 500
Vm 100 10 3
Vm = I m Rm
V 250
Vm = 100 1 103 m3 = 3 = = 2500
Vm 100 10 3
Vm = 100mV V 500
m4 = 4 = = 5000
Vm 100 10 3

R1 = (m1 1) Rm = (100 1) 100 = 9900


R2 = (m2 m1 ) Rm = (500 100) 100 = 40 K
R3 = (m3 m2 ) Rm = (2500 500) 100 = 200 K
R4 = (m4 m3 ) Rm = (5000 2500) 100 = 250 K

53
CLASS NOTES ON ELECTRICAL MEASUREMENTS & INSTRUMENTATION 2015

AC BRIDGES

2.1 General form of A.C. bridge

AC bridge are similar to D.C. bridge in topology(way of connecting).It consists of four arm
AB,BC,CD and DA .Generally the impedance to be measured is connected between A and B.
A detector is connected between B and D. The detector is used as null deflection instrument.
Some of the arms are variable element. By varying these elements, the potential values at B and
D can be made equal. This is called balancing of the bridge.

Fig. 2.1 General form of A.C. bridge


At the balance condition, the current through detector is zero.
. .
I1 = I 3
. .
I2 = I4
. .
I1 I3
= (2.1)
. .
I2 I4

54
CLASS NOTES ON ELECTRICAL MEASUREMENTS & INSTRUMENTATION 2015

At balance condition,

Voltage drop across AB=voltage drop across AD.


. .
E1 = E 2
. . . .
I1 Z1 = I 2 Z 2 (2.2)
Similarly, Voltage drop across BC=voltage drop across DC
. .
E3 = E4

. . . .
I 3 Z3 = I 4 Z 4 (2.3)

. .
I1 Z2
From Eqn. (2.2), we have = (2.4)
. .
I2 Z1

. .
I3 Z4
From Eqn. (2.3), we have = (2.5)
. .
I4 Z3

From equation -2.1, it can be seen that, equation -2.4 and equation-2.5 are equal.

. .
Z2 Z4
=
. .
Z1 Z3

. . . .
Z1 Z 4 = Z 2 Z 3

Products of impedances of opposite arms are equal.

Z1 1 Z 4 4 = Z 2 2 Z 3 3

Z1 Z 4 1 + 4 = Z 2 Z 3 2 + 3

Z1 Z 4 = Z 2 Z 3

1 + 4 = 2 + 3

55
CLASS NOTES ON ELECTRICAL MEASUREMENTS & INSTRUMENTATION 2015

For balance condition, magnitude on either side must be equal.

Angle on either side must be equal.

Summary
For balance condition,
. . . .
I1 = I 3 , I 2 = I 4
Z1 Z 4 = Z 2 Z 3

1 + 4 = 2 + 3
. . . .
E1 = E 2 & E3 = E4

2.2 Types of detector


The following types of instruments are used as detector in A.C. bridge.

Vibration galvanometer

Head phones (speaker)

Tuned amplifier

2.2.1 Vibration galvanometer


Between the point B and D a vibration galvanometer is connected to indicate the bridge
balance condition. This A.C. galvanometer which works on the principle of resonance. The
A.C. galvanometer shows a dot, if the bridge is unbalanced.
2.2.2 Head phones
Two speakers are connected in parallel in this system. If the bridge is unbalanced, the
speaker produced more sound energy. If the bridge is balanced, the speaker do not produced
any sound energy.

2.2.3 Tuned amplifier

If the bridge is unbalanced the output of tuned amplifier is high. If the bridge is balanced,
output of amplifier is zero.

56
CLASS NOTES ON ELECTRICAL MEASUREMENTS & INSTRUMENTATION 2015

2.3 Measurements of inductance

2.3.1 Maxwells inductance bridge

The choke for which R1 and L1 have to measure connected between the points A and
B. In this method the unknown inductance is measured by comparing it with the standard
inductance.

Fig. 2.2 Maxwells inductance bridge

L2 is adjusted, until the detector indicates zero current.


Let R1= unknown resistance
L1= unknown inductance of the choke.
L2= known standard inductance
R1,R2,R4= known resistances.

57
CLASS NOTES ON ELECTRICAL MEASUREMENTS & INSTRUMENTATION 2015

Fig 2.3 Phasor diagram of Maxwells inductance bridge


. . . .
At balance condition, Z 1 Z 4 = Z 2 Z 3

( R1 + jXL1 ) R4 = ( R2 + jXL2 ) R3

( R1 + jwL1 ) R4 = ( R2 + jwL2 ) R3

R1R4 + jwL1R4 = R2 R3 + jwL2 R3

Comparing real part,

R1R4 = R2 R3

R R
R1 = 2 3 (2.6)
R4

Comparing the imaginary parts,

wL1R4 = wL2 R3

L R
L1 = 2 3 (2.7)
R4

WL1 WL2 R3 R4
Q-factor of choke, Q = =
R1 R4 R2 R3

WL2
Q= (2.8)
R2

58
CLASS NOTES ON ELECTRICAL MEASUREMENTS & INSTRUMENTATION 2015

Advantages

Expression for R1 and L1 are simple.


Equations area simple
They do not depend on the frequency (as w is cancelled)
R1 and L1 are independent of each other.

Disadvantages

Variable inductor is costly.


Variable inductor is bulky.

2.3.2 Maxwells inductance capacitance bridge

Unknown inductance is measured by comparing it with standard capacitance. In this bridge,


balance condition is achieved by varying C4.

Fig 2.4 Maxwells inductance capacitance bridge

59
CLASS NOTES ON ELECTRICAL MEASUREMENTS & INSTRUMENTATION 2015

At balance condition, Z1Z4=Z3Z2 (2.9)

1
R4
1 jwC 4
Z 4 = R4 || =
jwC 4 1
R4 +
jwC4

R4 R4
Z4 = = (2.10)
jwR4 C 4 + 1 1 + jwR4C 4

Substituting the value of Z4 from eqn. (2.10) in eqn. (2.9) we get

R4
( R1 + jwL1 ) = R2 R3
1 + jwR4 C 4

Fig 2.5 Phasor diagram of Maxwells inductance capacitance bridge

( R1 + jwL1 ) R4 = R2 R3 (1 + jwR4C 4 )

R1R4 + jwL1R4 = R2 R3 + jwC4 R4 R2 R3

Comparing real parts,

R1R4 = R2 R3

60
CLASS NOTES ON ELECTRICAL MEASUREMENTS & INSTRUMENTATION 2015

R R
R1 = 2 3 (2.11)
R4

Comparing imaginary part,

wL1R4 = wC 4 R4 R2 R3

L1 = C 4 R2 R3 (2.12)

Q-factor of choke,

WL1 R
Q= = w C 4 R2 R3 4
R1 R2 R3

Q = wC4 R4 (2.13)

Advantages

Equation of L1 and R1 are simple.


They are independent of frequency.
They are independent of each other.
Standard capacitor is much smaller in size than standard inductor.

Disadvantages

Standard variable capacitance is costly.


It can be used for measurements of Q-factor in the ranges of 1 to 10.
It cannot be used for measurements of choke with Q-factors more than 10.
We know that Q =wC4R4

For measuring chokes with higher value of Q-factor, the value of C4 and R4 should be
higher. Higher values of standard resistance are very expensive. Therefore this bridge cannot be
used for higher value of Q-factor measurements.

61
CLASS NOTES ON ELECTRICAL MEASUREMENTS & INSTRUMENTATION 2015

2.3.3 Hays bridge

Fig 2.6 Hays bridge

.
E1 = I1R1 + jI1 X 1
. . .
E = E1 + E 3
. . I4
E 4 = I 4 R4 +
jwC 4
.
E 3 = I 3 R3

1 1 + jwR4C4
Z 4 = R4 + =
jwC4 jwC4

62
CLASS NOTES ON ELECTRICAL MEASUREMENTS & INSTRUMENTATION 2015

Fig 2.7 Phasor diagram of Hays bridge

At balance condition, Z1Z4=Z3Z2

1 + jwR4C 4
( R1 + jwL1 )( ) = R2 R3
jwC 4

( R1 + jwL1 )(1 + jwR4C 4 ) = jwR2 C 4 R3

R1 + jwC 4 R4 R1 + jwL1 + j 2 w 2 L1C 4 R4 = jwC4 R2 R3

( R1 w2 L1C4 R4 ) + j ( wC4 R4 R1 + wL1 ) = jwC4 R2 R3

Comparing the real term,

R1 w 2 L1C 4 R4 = 0

R1 = w 2 L1C4 R4 (2.14)

63
CLASS NOTES ON ELECTRICAL MEASUREMENTS & INSTRUMENTATION 2015

Comparing the imaginary terms,

wC4 R4 R1 + wL1 = wC4 R2 R3

C 4 R4 R1 + L1 = C 4 R2 R3

L1 = C4 R2 R3 C4 R4 R1 (2.15)

Substituting the value of R1 fro eqn. 2.14 into eqn. 2.15, we have,

L1 = C 4 R2 R3 C 4 R4 w 2 L1C 4 R4

L1 = C4 R2 R3 w 2 L1C4 2 R4 2

L1 (1 + w 2 L1C 4 2 R4 2 ) = C 4 R2 R3

C4 R2 R3
L1 = (2.16)
1 + w2 L1C4 2 R4 2

Substituting the value of L1 in eqn. 2.14 , we have

w 2C4 2 R2 R3 R4
R1 = (2.17)
1 + w 2C4 2 R4 2

wL1 w C4 R2 R3 1 + w2C4 2 R4 2
Q= =
R1 1 + w2C4 2 R4 2 w2C4 2 R4 R2 R3

1
Q= (2.18)
wC4 R4

64
CLASS NOTES ON ELECTRICAL MEASUREMENTS & INSTRUMENTATION 2015

Advantages

Fixed capacitor is cheaper than variable capacitor.


This bridge is best suitable for measuring high value of Q-factor.

Disadvantages

Equations of L1and R1 are complicated.


Measurements of R1 and L1 require the value of frequency.
This bridge cannot be used for measuring low Q- factor.

2.3.4 Owens bridge

Fig 2.8 Owens bridge

E1 = I1R1 + jI1 X1
I4 leads E4 by 900

65
CLASS NOTES ON ELECTRICAL MEASUREMENTS & INSTRUMENTATION 2015

. . .
E = E1 + E 3
. I2
E 2 = I 2 R2 +
jwC 2

Fig 2.9 Phasor diagram of Owens bridge

. . . .
Balance condition, Z 1 Z 4 = Z 2 Z 3

1 jwC 2 R2 + 1
Z 2 = R2 + =
jwC2 jwC 2

1 (1 + jwR2C 2 ) R3
( R1 + jwL1 ) =
jwC 4 jwC 2

C 2 ( R1 + jwL1 ) = R3C4 (1 + jwR2C 2 )

R1C 2 + jwL1C2 = R3C4 + jwR2C 2 R3C4

Comparing real terms,

R1C2 = R3C4

66
CLASS NOTES ON ELECTRICAL MEASUREMENTS & INSTRUMENTATION 2015

RC
R1 = 3 4
C2

Comparing imaginary terms,

wL1C 2 = wR2 C2 R3C 4

L1 = R2 R3C4

WL1 wR2 R3C4C 2


Q- factor = =
R1 R3C 4

Q = wR2C2

Advantages
Expression for R1 and L1 are simple.
R1 and L1 are independent of Frequency.
Disadvantages
The Circuits used two capacitors.
Variable capacitor is costly.
Q-factor range is restricted.

67
CLASS NOTES ON ELECTRICAL MEASUREMENTS & INSTRUMENTATION 2015

2.3.5 Andersons bridge

Fig 2.10 Andersons bridge

.
E1 = I1 ( R1 + r1 ) + jI1 X 1
E3 = EC
. .
E 4 = I C r + EC

I 2 = I 4 + IC

E2+ E4 = E

E1 + E 3 = E

68
CLASS NOTES ON ELECTRICAL MEASUREMENTS & INSTRUMENTATION 2015

Fig 2.11 Phasor diagram of Andersons bridge

Step-1 Take I1 as references vector .Draw I1R11 in phase with I1


1
R11 = ( R1 + r1 ) , I1 X1 is r to I1R1

E1 = I1R11 + jI1 X 1
Step-2 I1 = I 3 , E3 is in phase with I 3 , From the circuit ,

E3 = EC , I C leads EC by 900

Step-3 E4 = I C r + EC

Step-4 Draw I 4 in phase with E4 , By KCL , I 2 = I 4 + I C
Step-5 Draw E2 in phase with I2

Step-6 By KVL , E1 + E3 = E or E2 + E4 = E

69
CLASS NOTES ON ELECTRICAL MEASUREMENTS & INSTRUMENTATION 2015

Fig 2.12 Equivalent delta to star conversion for the loop MON

R4 r jwCR 4 r
Z7 = =
1 1 + jwC ( R4 + r )
R4 + r +
jwc

1
R4
jwC R4
Z6 = =
1 1 + jwC ( R4 + r )
R4 + r +
jwc
R4 jwCR4 r
( R11 + jwL1 ) = R3 ( R2 + )
1 + jwC ( R4 + r ) 1 + jwC ( R4 + r )

( R11 + jwL1 ) R4 R (1 + jwC ( R4 + r )) + jwCrR4


= R3 2
1 + jwC ( R4 + r ) 1 + jwC ( R4 + r )

R11R4 + jwL1R4 = R2 R3 + jCwR2 R3 (r + R4 ) + jwCrR4 R3

70
CLASS NOTES ON ELECTRICAL MEASUREMENTS & INSTRUMENTATION 2015

Fig 2.13 Simplified diagram of Andersons bridge

Comparing real term,

R11R4 = R2 R3

( R1 + r1 ) R4 = R2 R3
R R
R1 = 2 3 r1
R4
Comparing the imaginary term,

wL1R4 = wCR2 R3 (r + R4 ) + wcrR3 R4


R RC
L1 = 2 3 (r + R4 ) + R3rC
R4

R
L1 = R3C 2 (r + R4 ) + r
R4
Advantages

Variable capacitor is not required.


Inductance can be measured accurately.
R1 and L1 are independent of frequency.
Accuracy is better than other bridges.

71
CLASS NOTES ON ELECTRICAL MEASUREMENTS & INSTRUMENTATION 2015

Disadvantages

Expression for R1 and L1 are complicated.


This is not in the standard form A.C. bridge.

2.4 Measurement of capacitance and loss angle. (Dissipation factor)

2.4.1 Dissipation factors (D)

A practical capacitor is represented as the series combination of small resistance and


ideal capacitance.

From the vector diagram, it can be seen that the angle between voltage and current is slightly less
than 900. The angle is called loss angle.

Fig 2.14 Condensor or capacitor

Fig 2.15 Representation of a practical capacitor

72
CLASS NOTES ON ELECTRICAL MEASUREMENTS & INSTRUMENTATION 2015

Fig 2.16 Vector diagram for a practical capacitor

A dissipation factor is defined as tan .


IR R
tan = = = wCR
IX C X C

D = wCR
1
D=
Q

sin
D = tan = For small value of in radians
cos 1
D Loss Angle ( must be in radian)
2.4.2 Desautys Bridge
C1= Unknown capacitance

At balance condition,

1 1
R4 = R3
jwC1 jwC 2

R4 R3
=
C1 C 2

R C
C1 = 4 2
R3

73
CLASS NOTES ON ELECTRICAL MEASUREMENTS & INSTRUMENTATION 2015

Fig 2.17 Desautys bridge

Fig 2.18 Phasor diagram of Desautys bridge

74
CLASS NOTES ON ELECTRICAL MEASUREMENTS & INSTRUMENTATION 2015

2.4.3 Modified desautys bridge

Fig 2.19 Modified Desautys bridge

Fig 2.20 Phasor diagram of Modified Desautys bridge

75
CLASS NOTES ON ELECTRICAL MEASUREMENTS & INSTRUMENTATION 2015

R11 = ( R1 + r1 )

R12 = ( R2 + r2 )
1 1
At balance condition, ( R11 + ) R4 = R3 ( R12 + )
jwC1 jwC 2

R4 R3
R11R4 + = R3 R12 + )
jwC1 jwC 2

Comparing the real term, R11R4 = R3 R12

R R1
R11 = 3 2
R4
( R2 + r2 ) R3
R + r1 =
R4
Comparing imaginary term,
R4 R
= 3
wC1 wC 2

R C
C1 = 4 2
R3
Dissipation factor D=wC1r1
Advantages
r1 and c1 are independent of frequency.
They are independent of each other.
Source need not be pure sine wave.

2.4.4 Schering bridge

E1 = I1r1 jI1 X 4

C2 = C4= Standard capacitor (Internal resistance=0)

C4= Variable capacitance.

C1= Unknown capacitance.

r1= Unknown series equivalent resistance of the capacitor.

76
CLASS NOTES ON ELECTRICAL MEASUREMENTS & INSTRUMENTATION 2015

R3=R4= Known resistor.

Fig 2.21 Schering bridge

1 jwC1r1 + 1
Z1 = r1 + =
jwC1 jwC1

1
R4
jwC 4 R4
Z4 = =
R4 +
1 1 + jwC 4 R4
jwC 4

77
CLASS NOTES ON ELECTRICAL MEASUREMENTS & INSTRUMENTATION 2015

Fig 2.22 Phasor diagram of Schering bridge

. . . .
At balance condition, Z 1 Z 4 = Z 2 Z 3
1 + jwC1r1 R4 R3
=
jwC1 1 + jwC 4 R4 jwC 2

(1 + jwC1r1 ) R4C 2 = R3C1 (1 + jwC 4 r4 )

R2C2 + jwC1r1R4C2 = R3C1 + jwC 4 R4 R3C1

Comparing the real part,

R C
C1 = 4 2
R3

Comparing the imaginary part,

wC1r1R4C2 = wC4 R3 R4C1

C R
r1 = 4 3
C2

Dissipation factor of capacitor,

78
CLASS NOTES ON ELECTRICAL MEASUREMENTS & INSTRUMENTATION 2015

R C C R
D = wC1r1 = w 4 2 4 3
R3 C2

D = wC4 R4

Advantages

In this type of bridge, the value of capacitance can be measured accurately.


It can measure capacitance value over a wide range.
It can measure dissipation factor accurately.

Disadvantages

It requires two capacitors.


Variable standard capacitor is costly.

2.5 Measurements of frequency

2.5.1 Weins bridge

Weins bridge is popularly used for measurements of frequency of frequency. In this bridge, the
value of all parameters are known. The source whose frequency has to measure is connected as
shown in the figure.

1 jwC1r1 + 1
Z1 = r1 + =
jwC1 jwC1

R2
Z2 =
1 + jwC 2 R2
. . . .
At balance condition, Z 1 Z 4 = Z 2 Z 3
jwC1r1 + 1 R2
R4 = R3
jwC1 1 + jwC 2 R2

(1 + jwC1r1 )(1 + jwC 2 R2 ) R4 = R2 R3 jwC1

[1 + jwC R 2
2 2 + jwC1r1 w C1C 2 r1R2 = jwC1]R2 R3
R4

79
CLASS NOTES ON ELECTRICAL MEASUREMENTS & INSTRUMENTATION 2015

Fig 2.23 Weins bridge

Fig 2.24 Phasor diagram of Weins bridge

80
CLASS NOTES ON ELECTRICAL MEASUREMENTS & INSTRUMENTATION 2015

Comparing real term,

1 w 2C1C 2 r1R2 = 0

w 2C1C 2 r1R2 = 1
1
w2 =
C1C2 r1R2
1 1
w= , f =
C1C 2 r1 R2 2 C1C 2 r1 R2

NOTE

The above bridge can be used for measurements of capacitance. In such case, r1 and C1 are
unknown and frequency is known. By equating real terms, we will get R1 and C1. Similarly by
equating imaginary term, we will get another equation in terms of r1 and C1. It is only used for
measurements of Audio frequency.
A.F=20 HZ to 20 KHZ
R.F=>> 20 KHZ
Comparing imaginary term,
R R
wC 2 R2 + wC1r1 = wC1 2 3
R4
CR R
C2 R2 + C1r1 = 1 2 3 ..(2.19)
R4

1
C1 =
2
w C2 r1R2
Substituting in eqn. (2.19), we have
r1 R R
C2 R2 + = 2 3 C1
w2C2 r1R2 R4

R4
Multiplying in both sides, we have
R2 R3

R4 1 R
C2 R2 + 4 = C1
R2 R3 w2C2 R2 R2 R3

81
CLASS NOTES ON ELECTRICAL MEASUREMENTS & INSTRUMENTATION 2015

C R R4
C1 = 2 4 +
R3 w2C2 R22 R3

w 2C1r1C 2 R2 = 1

1 1
r1 = =
2
w C 2 R2C1 C R R4
w 2C 2 R2 2 4 +
R3 w 2C 2 R22 R3

1
=
w2C 2 R R
2 2 4 + R4

R3 R2 R3

1
r1 =
R3 2 2 1
w C2 R2 +
R4 R2


R 1
r1 = 3
R4 ( w 2C 2 R + 1 )
2 2
R2

2.5.2 High Voltage Schering Bridge

Fig 2.25 High Voltage Schering bridge

82
CLASS NOTES ON ELECTRICAL MEASUREMENTS & INSTRUMENTATION 2015

(1) The high voltage supply is obtained from a transformer usually at 50 HZ.

2.6 Wagner earthing device:

Fig 2.26 Wagner Earthing device

Wagner earthing consists of R and C in series. The stray capacitance at node B and D are
CB, CD respectively. These Stray capacitances produced error in the measurements of L and
C. These error will predominant at high frequency. The error due to this capacitance can be
eliminated using wagner earthing arm.

Close the change over switch to the position (1) and obtained balanced. Now change the
switch to position (2) and obtained balance. This process has to repeat until balance is achieved
in both the position. In this condition the potential difference across each capacitor is zero.
Current drawn by this is zero. Therefore they do not have any effect on the measurements.

What are the sources of error in the bridge measurements?

Error due to stray capacitance and inductance.


Due to external field.
Leakage error: poor insulation between various parts of bridge can produced this error.
Eddy current error.
Frequency error.

83
CLASS NOTES ON ELECTRICAL MEASUREMENTS & INSTRUMENTATION 2015

Waveform error (due to harmonics)


Residual error: small inductance and small capacitance of the resistor produce this error.

Precaution

The load inductance is eliminated by twisting the connecting the connecting lead.
A0 r
In the case of capacitive bridge, the connecting lead are kept apart.(Q C = )
d
In the case of inductive bridge, the various arm are magnetically screen.
In the case of capacitive bridge, the various arm are electro statically screen to reduced
the stray capacitance between various arm.
To avoid the problem of spike, an inter bridge transformer is used in between the source
and bridge.
The stray capacitance between the ends of detector to the ground, cause difficulty in
balancing as well as error in measurements. To avoid this problem, we use wagner
earthing device.

2.7 Ballastic galvanometer

This is a sophisticated instrument. This works on the principle of PMMC meter. The only
difference is the type of suspension is used for this meter. Lamp and glass scale method is used
to obtain the deflection. A small mirror is attached to the moving system. Phosphorous bronze
wire is used for suspension.

When the D.C. voltage is applied to the terminals of moving coil, current flows through it. When
a current carrying coil kept in the magnetic field, produced by permanent magnet, it experiences
a force. The coil deflects and mirror deflects. The light spot on the glass scale also move. This
deflection is proportional to the current through the coil.

Q
i= , Q = it = idt
t

Q , deflection Charge

84
CLASS NOTES ON ELECTRICAL MEASUREMENTS & INSTRUMENTATION 2015

Fig 2.27 Ballastic galvanometer

2.8 Measurements of flux and flux density (Method of reversal)

D.C. voltage is applied to the electromagnet through a variable resistance R1 and a reversing
switch. The voltage applied to the toroid can be reversed by changing the switch from position 2
to position 1. Let the switch be in position 2 initially. A constant current flows through the
toroid and a constant flux is established in the core of the magnet.

A search coil of few turns is provided on the toroid. The B.G. is connected to the search
coil through a current limiting resistance. When it is required to measure the flux, the switch is
changed from position 2 to position 1. Hence the flux reduced to zero and it starts increasing
in the reverse direction. The flux goes from + to - , in time t second. An emf is induced in
the search coil, science the flux changes with time. This emf circulates a current through R2 and
B.G. The meter deflects. The switch is normally closed. It is opened when it is required to take
the reading.

85
CLASS NOTES ON ELECTRICAL MEASUREMENTS & INSTRUMENTATION 2015

2.8.1 Plotting the BH curve

The curve drawn with the current on the X-axis and the flux on the Y-axis, is called
magnetization characteristics. The shape of B-H curve is similar to shape of magnetization
characteristics. The residual magnetism present in the specimen can be removed as follows.

Fig 2.28 BH curve

Fig 2.29 Magnetization characteristics

86
CLASS NOTES ON ELECTRICAL MEASUREMENTS & INSTRUMENTATION 2015

Close the switch S2 to protect the galvanometer, from high current. Change the switch
S1 from position 1 to 2 and vice versa for several times.
To start with the resistance R1 is kept at maximum resistance position. For a particular value of
current, the deflection of B.G. is noted. This process is repeated for various value of current. For

each deflection flux can be calculated.( B = )
A
Magnetic field intensity value for various current can be calculated.().The B-H curve can be
plotted by using the value of B and H.

2.8.2 Measurements of iron loss:


Let RP= pressure coil resistance
RS = resistance of coil S1
E= voltage reading= Voltage induced in S2
I= current in the pressure coil
VP= Voltage applied to wattmeter pressure coil.
W= reading of wattmeter corresponding voltage V
W1= reading of wattmeter corresponding voltage E
W V W1 E E W
= W1 =
W1 EP W V V
W1=Total loss=Iron loss+ Cupper loss.
The above circuit is similar to no load test of transformer.

In the case of no load test the reading of wattmeter is approximately equal to iron loss. Iron loss
depends on the emf induced in the winding. Science emf is directly proportional to flux. The
voltage applied to the pressure coil is V. The corresponding of wattmeter is W. The iron loss
WE
corresponding E is E= . The reading of the wattmeter includes the losses in the pressure
V
coil and copper loss of the winding S1. These loses have to be subtracted to get the actual iron
loss.

87
CLASS NOTES ON ELECTRICAL MEASUREMENTS & INSTRUMENTATION 2015

2.9 Galvanometers

D-Arsonval Galvanometer

Vibration Galvanometer

Ballistic C

2.9.1 D-arsonval galvanometer (d.c. galvanometer)

Fig 2.30 D-Arsonval Galvanometer

Galvanometer is a special type of ammeter used for measuring A or mA. This is a


sophisticated instruments. This works on the principle of PMMC meter. The only difference is
the type of suspension used for this meter. It uses a sophisticated suspension called taut
suspension, so that moving system has negligible weight.

Lamp and glass scale method is used to obtain the deflection. A small mirror is attached
to the moving system. Phosphors bronze is used for suspension.

88
CLASS NOTES ON ELECTRICAL MEASUREMENTS & INSTRUMENTATION 2015

When D.C. voltage is applied to the terminal of moving coil, current flows through it.
When current carrying coil is kept in the magnetic field produced by P.M. , it experiences a
force. The light spot on the glass scale also move. This deflection is proportional to the current
through the coil. This instrument can be used only with D.C. like PMMC meter.

The deflecting Torque,


TD=BINA
TD=GI, Where G=BAN
TC=KS =S
At balance, TC=TD S =GI
GI
=
S
Where G= Displacements constant of Galvanometer
S=Spring constant

2.9.2 Vibration Galvanometer (A.C. Galvanometer )

The construction of this galvanometer is similar to the PMMC instrument except for the moving
system. The moving coil is suspended using two ivory bridge pieces. The tension of the system
can be varied by rotating the screw provided at the top suspension. The natural frequency can be
varied by varying the tension wire of the screw or varying the distance between ivory bridge
piece.

When A.C. current is passed through coil an alternating torque or vibration is produced. This
vibration is maximum if the natural frequency of moving system coincide with supply frequency.
Vibration is maximum, science resonance takes place. When the coil is vibrating , the mirror
oscillates and the dot moves back and front. This appears as a line on the glass scale. Vibration
galvanometer is used for null deflection of a dot appears on the scale. If the bridge is unbalanced,
a line appears on the scale

89
CLASS NOTES ON ELECTRICAL MEASUREMENTS & INSTRUMENTATION 2015

Fig 2.31 Vibration Galvanometer

Example 2.2-In a low- Voltage Schering bridge designed for the measurement of
permittivity, the branch ab consists of two electrodes between which the specimen under
test may be inserted, arm bc is a non-reactive resistor R3 in parallel with a standard
capacitor C3, arm CD is a non-reactive resistor R4 in parallel with a standard capacitor C4,
arm da is a standard air capacitor of capacitance C2. Without the specimen between the
electrode, balance is obtained with following values , C3=C4=120 pF, C2=150 pF,
R3=R4=5000.With the specimen inserted, these values become C3=200 pF,C4=1000
pF,C2=900 pF and R3=R4=5000. In such test w=5000 rad/sec. Find the relative
permittivity of the specimen?

capacitance measured with given medium


Sol: Relative permittivity( r ) =
capacitance measured with air medium

90
CLASS NOTES ON ELECTRICAL MEASUREMENTS & INSTRUMENTATION 2015

Fig 2.32 Schering bridge

R
C1 = C2 ( 4 )
R3
Let capacitance value C0, when without specimen dielectric.
Let the capacitance value CS when with the specimen dielectric.
R 5000
C0 = C 2 ( 4 ) = 150 = 150 pF
R3 5000
R 5000
C S = C 2 ( 4 ) = 900 = 900 pF
R3 5000
C S 900
r = = =6
C0 150

Example 2.3- A specimen of iron stamping weighting 10 kg and having a area of 16.8 cm2 is
tested by an episten square. Each of the two winding S1 and S2 have 515 turns. A.C. voltage
of 50 HZ frequency is given to the primary. The current in the primary is 0.35 A. A
voltmeter connected to S2 indicates 250 V. Resistance of S1 and S2 each equal to 40 .
Resistance of pressure coil is 80 k. Calculate maximum flux density in the specimen and
iron loss/kg if the wattmeter indicates 80 watt?

91
CLASS NOTES ON ELECTRICAL MEASUREMENTS & INSTRUMENTATION 2015

Soln- E = 4.44 f m N

E
Bm = = 1.3wb / m 2
4.44 fAN

RS E2
Iron loss= W (1 + )
RP ( RS + RP )

40 250 2
= 80(1 + ) = 79.26 watt
80 10 3 (40 + 80 103 )

Iron loss/ kg=79.26/10=7.926 w/kg.

92

Anda mungkin juga menyukai